PR 3 BK

Lakukan tugas rumah & ujian kamu dengan baik sekarang menggunakan Quizwiz!

Tanith and Brian work at the same law firm. Tanith is an associate and Brian is a partner, but Brian is not Tanith's direct supervisor. One day, Tanith and Brian run into each other in a courtroom hallway. Tanith mentions to Brian that she was in the middle of a trial and she had just convinced a witness to "bend the truth a little—or a lot." Brian shakes his head in disapproval but otherwise does nothing. Tanith's client wins at trial. Is Brian subject to discipline for Tanith's misconduct? A. No, because he is not Tanith's direct supervisor B. No, because he did not order or ratify Tanith's misconduct C. Yes, because he is jointly and severally liable for the misconduct of everyone at the firm D. Yes, because he failed to take reasonable remedial action

D. A lawyer is subject to discipline for a disciplinary violation committed by a second lawyer if: (i) the first lawyer ordered the second lawyer's misconduct or knew about it and ratified it; or (ii) the first lawyer is a partner or manager or has direct supervisory responsibility over the second lawyer, and knows about the misconduct at a time when its consequences can be avoided or mitigated and fails to take reasonable remedial action. (D) is correct because Brian had a duty to take reasonable remedial action (e.g., advising Tanith to strike the testimony, notifying Tanith's supervisor, etc.). (A) is wrong because every partner has this duty. It's immaterial that Brian isn't Tanith's direct supervisor. (B) is wrong because a partner need not order or ratify the misconduct; he may also be responsible if he knows about the misconduct when its consequences can be avoided or mitigated and fails to take reasonable remedial action. That's precisely what happened here. (C) is wrong. The term "joint and several liability" refers to a partner's civil liability for the firm's obligations; it doesn't refer to whether a partner is subject to discipline. Furthermore, a partner is not ethically responsible for all misconduct by firm employees.

A law firm is defending a client in a personal injury case. Andrea, a senior associate, is in charge of the case. Andrea believes that the plaintiff is exaggerating his injury, so she hires Ed, a rookie private investigator, on a contract basis to "check out the plaintiff and get to the bottom of this." Instead of conducting covert surveillance as Andrea expected, Ed confronts the plaintiff at his home and tells him to drop the lawsuit. The rules of professional conduct prohibit a lawyer from communicating with a represented party without their counsel's consent. Is Andrea subject to discipline? A. No, because she is not a partner of the firm B. No, because she did not order or ratify the misconduct, and had no opportunity to take reasonable remedial action C. No, because Ed is not an employee of her firm D. Yes, because she failed to make reasonable efforts to assure that Ed's conduct would comply with her own professional obligations E. Yes, because a supervising lawyer is always responsible for the conduct of persons working on a matter

D. The partners and managers in a law firm have to make reasonable efforts to assure that the conduct of nonlawyers who are associated with the firm is compatible with the obligations of the profession. Additionally, any lawyer who directly supervises the work of a nonlawyer has the same duty with respect to that nonlawyer. Therefore, (D) is correct because Andrea is Ed's direct supervisor in the matter—she hired him and is in charge of the case. She had a duty to make reasonable efforts to assure that Ed's conduct would comply with the rules of professional conduct. For instance, she should have made sure he understood the bounds of what she wanted him to do and that, as her agent, he could not communicate with the plaintiff directly.

Which of the following is a PROPER restriction on a lawyer's right to practice law? A. Leon takes on a new associate, Adrian. Adrian's employment agreement provides that if Adrian leaves the firm on his own accord, he will not practice law within a 10-mile radius for six months following his departure B. Leon settles a client's lawsuit against a pharmaceutical company. As part of the settlement, Leon promises not to bring any more claims against the company C. Leon takes on a new partner, Julissa. The partnership agreement provides that if Julissa leaves the firm and takes any of the firm's clients with her, Julissa must pay the firm 50% of her billings for that client for two years following her departure D. Leon sells his family law practice to Samaira and agrees to stop practicing family law in the state

D. There are two types of prohibited restrictions on a lawyer's right to practice law. First, a lawyer can't make or offer a partnership, employment, or similar agreement that restricts a lawyer's right to practice after termination of the relationship (except for an agreement concerning retirement benefits). The agreement in (A) is a noncompete agreement and is clearly improper, even though it's a small geographical restriction and a short period of time. The agreement in (C) is also improper because it effectively restricts Julissa's right to practice law where she chooses; it also restricts the clients' right to select their own lawyer. Second, a lawyer can't make or offer an agreement that includes a restriction on the lawyer's right to practice as part of the settlement of a client controversy. The agreement in (B) violates this rule. When a law practice is sold, the seller generally has to stop practicing law in the local area, so the agreement in (D) is proper.

4-3 Attorney Arlo has hired Clarissa, a third year student at a local law school, to assist him as his clerk. Clarissa is not licensed under any state law or court rule that allows third- year law students to engage in practice under the supervision of a licensed attorney. Arlo has Clarissa perform the following tasks: I. Drafting a release form for personal injury plaintiffs to sign after their cases have been settled. (Arlo himself has the plaintiffs sign the forms.) II. Interviewing witnesses to accidents, and having them sign Clarissa's written version of the interview. III. Reaching settlement agreements with insurance company representatives before suit has been filed. For which, if any, of the above is Arlo subject to discipline?

III only. Reach settlement agreements with insurance company representatives before suit has been filed.

3-1. Attorney Dave is defending Datatec Corporation in an employment discrimination suit in which the plaintiffs are represented by attorney Perclio. Wallner is a Datatec employee, but he is not an officer or shareholder in Datatec, and he is not a party to the lawsuit. Wallner is in charge of Datatec's Personnel Department, and he is responsible for insuring that Datatec's hiring practices comply with the laws against employment discrimination. Wallner is in poor health, so Dave scheduled the taking of Wallner's deposition as a precaution in case Wallner should die before trial. Without seeking Dave's consent, or even telling him, Perello had lunch with Wallner several days before the deposition, and on that occasion Perello pumped Wallner for information relevant to the lawsuit. When Dave learned what had happened, he telephoned Perello and called him a slimy, mud-sucking shyster. Which of the following is most nearly correct?

Perello is subject to discipline because he should not have talked with Wallner about the case without Dave's consent.

4.2Judge Josephine ruled in favor of Plaintiff in a civil action where Defendant was ordered to pay Plaintiff $50,000 in damages. Josephine has since resigned from the bench. Defendant has refused to pay the $50,000, asserting that the verdict was obtained through improper means. Defendant asks Josephine, now in private practice, if she will represent him. Would Josephine be subject to discipline if she represents Defendant?

Yes, because Josephine ruled on this case when she was a judge.

4-14. The law firm of Rice & O'Malley represents P.J. Kilmer, a successful businessperson who has given the firm almost all of his substantial legal business. Kilmer is presently involved in complex litigation in which Kilmer stands to receive a large damages award if the suit is successful. Costello subject to discipline?

Yes, because his statement implies that judge jensen will give him preferential treatment due to their past association.

3-10. The State Bar and the State University are joint sponsors of the State Continuing Legal Education Foundation. The purpose of the Foundation is to provide continuing legal education to lawyers and judges in the state. Its board of directors is composed one-half of members of the legal profession and one-half of university personnel. Judge Anistopolous has been invited to serve on the board of directors. Which of the following are correct? I. She. may serve on the board of directors, as long as it does not interfere with her judicial duties. II. It would not be proper for her to serve on the board of directors, since to do so would involve her in the teaching of law. III. If she serves on the board of directors, it would not be proper for her to accept the modest salary that is paid to other directors. IV. If she serves on the board of directors, she may accept the modest salary that is paid to other directors.

a. I. and IV. only.

3-42. Attorney Acheson represents Dell, who has pleaded guilty to a burglary charge. Dell told Acheson during one of their confidential conversations that this is the fourth time he has been busted for burglary, one other time in this state and two times in the neighboring state. Dell could be facing a mandatory 5-10 years in prison as a repeat offender, but he and Acheson are hoping that his pleading guilty will result in a shorter sentence. Unknown to Acheson or Dell, a glitch in the state computer files resulted in a presentencing report that did not pick up either Dell in-state or out-of-state prior convictions. At sentencing, Judge Johnson states, "Normally, I throw the book at young men like you who have no respect for the property of others. However, since I see you have no prior criminal record, I think you deserve another chance. I sentence you to two years probation and 300 hours community service." Both Acheson and Dell remain silent. Is Acheson subject to discipline?

a. No, because the mistake did not originate with Acheson or Dell.

3-38. Attorney A wants to advertise his legal services in a local newspaper. Which of the following statements would be proper for A to make in the advertisement? I. That he graduated Phi Beta Kappa from Lewis College and now teaches a course in Law and Economics. II. That he is a licensed real estate broker and a member of the National Broker's Association. III. That he speaks Polish and accepts credit cards. IV. That he is a major with the National Guard and a former State Senator.

b. All of the above.

3-49. Attorney A is a friend of B, a real estate broker. B is of the opinion that A is one of the best real estate lawyers in the community, and B recommends her to those persons seeking an attorney to close real estate transactions. Which of the following statements most correctly describes what A may do? I. A may represent clients referred by B, as long as A does not pay B a referral fee. II. A may give' B some of her professional cards to give to those persons seeking a real estate lawyer and accept employment resulting therefrom. III. A may take B and B's wife out to dinner as a way of expressing her appreciation for B's referrals.

b. I. only.

3-36. Attorneys A and B are law partners. They are, however, contemplating incorporation of their law practice. With respect to the proposed incorporation, which of the following statements best describes what A and B may properly do? I. A and B may incorporate their law practice and convey an interest in the corporation to their children. II. A and B may incorporate their law practice and thus avoid liability to clients for malpractice. III. A and B may incorporate their law practice and when they die, a fiduciary representative of their estates may hold the stock in the corporation for a reasonable time during administration. IV. A and B may incorporate their law practice and make their accountant treasurer of the corporation as long as the accountant does not own any interest in the corporation.

b. III. only.

3-47. Paul is a title insurance agent. In serving his customers, he routinely fills in the blanks in standard form documents that are prepared by lawyers. These documents include warranty deeds, quitclaim deeds, mortgages, releases of mortgages, affidavits as to debts and liens, lien waivers, and the like. On occasion, when his customers specifically ask, he advises them about the meaning and legal effect of the technical language used in the forms. Which of the following constitutes the unauthorized practice of law by Paul? I. Filling in the blanks on warranty deeds, quitclaim deeds, and mortgages. II. Filling in the blanks on affidavits as to debts and liens and lien waivers. III. Advising customers, at their request, about the meaning and legal effect of the technical language used in the forms.

b. III. only.

3-16. Attorney Almas was fully licensed to practice in State Green. Her office was located in Endogreen, a small town located on the extreme western border of State Green, on the western edge of a desert area. Almas received a retainer from client Chum, with the agreement that Almas would use funds from the retainer for such things as filing fees when they came due. There was no federally insured bank or savings institution in Endogreen. The nearest such institution in State Green was located in the county seat, which was over 60 miles away from Almas's office and where Almas and Chum resided. Therefore, Almas decided to put Chum's money in his client trust account in a bank in Blue start, a medium-sized city located just across the state line in neighboring State Blue. The account was fully insured by the federal government, hut was not an interestbearing account. Almas is not licensed to practice law in State Blue. Was it proper for Almas to place Chum's money in an account in State Blue?

b. No, because Almas is not licensed to practice in State Blue.

3-19. Newspaper reporter Dabba Ratchet writes a semi-humorous gossip column in which she reports on the vices and victories in the private lives of prominent citizens. Among her favorite targets are judges and lawyers. Over the years, Ms. Ratchet and lawyer Lubner have worked out a tacit arrangement. Every now and then in her column, Ms. Ratchet gives glowing praise to Lubner's legal talents; in return, Lubner calls Ms. Ratchet whenever he learns a juicy tidbit about another lawyer or judge. Is Lubner's conduct proper?

b. No, because a lawyer should not use this means of obtaining favorable publicity.

3-14. Capra hired lawyer Laslo to represent him in a personal injury action against Giant Co. As Capra and Laslo discussed the case, Capra stated, "Of course, I can't predict what the future will hold, but I am ready to go all the way on this. If we have to go to trial to see justice done, so be it. I probably wouldn't agree to a settlement under $500,000, no matter what." Laslo agreed that the claim was worth at least that, but felt they would receive a much higher award if the case went to a jury. Shortly before the trial started, the lawyer for Giant Co. contacted Laslo with a settlement offer of $150,000. Laslo tried to call Capra, but couldn't reach him. After two hours of trying to reach Capra, Laslo called Giant Co. back and rejected the offer. At trial, the jury awarded Capra $1 million. Is Laslo subject to malpractice liability for his actions?

b. No, because the jury award was much greater than the settlement offer.

3-4. During his second year in law school, Peter's wife divorced him, as a result of which he suffered serious emotional imbalance. Ultimately he managed to graduate, and the state bar sent a routine form letter to Professor Schultze, asking her to comment on Peter's fitness to practice law. Professor Schultze is a member of the bar of that state. In response, she gave her candid opinion--that Peter was prone to extreme and irrational tirades against women, and that he was not yet recovered from his emotional trauma. Based partly on this information, the state bar denied Peter's petition for admission. Now, three years later, Peter has asked Professor Schultze to support his re-petition for admission. After carefully checking the facts, Professor Schultze concludes that Peter has regained his emotional balance. Which of the following is most nearly correct?

b. Now, Professor Schultze should support Peter's re-petition, since she believes that he has regained his emotional balance

3-15. During the course of researching a reply brief, attorney Amy discovered a case in the controlling jurisdiction that seemed to be right on one of the key issues involved in the litigation that she was researching. Although much of the dicta in the case seemed to favor Amy's client, one critical sentence in the holding clearly put the court behind the position asserted by Amy's opponent, Bertram. However, Bertram had done a slipshod job of researching the issues and had failed to find the case and to cite it in his brief. Amy decided to cite the case in her reply brief, but she cited the case as favoring her client by quoting much of the dicta and deliberately omitting the key sentence in the holding. Amy filed the reply brief with the court and sent a copy to Bertram, who she knew often bragged, "I haven't actually read a case since my first year of law school." Amy was also aware that this judge's clerks tended to be overworked and so did not always read all the cases cited by attorneys in their briefs. Therefore, she hoped that the negative aspects of the case might slip by unnoticed by the judge and opposing counsel. Is Amy subject to discipline?

b. Yes, because Amy is attempting to mislead the tribunal.

3-8. In a trade secret action against defendant Devere Corp., pending in a United States district court, plaintiff's lawyer Latimer gave timely notice that on July 22 at 9 a.m., he would take the deposition of Devere's Vice President for Manufacturing and Marketing, Sigmoid Fletcher, Ph.D. From earlier discovery in the case, Latimer had good reason to believe that Dr. Fletcher's testimony would prove that Devere had stolen and was using the plaintiff's trade secrets. On the appointed day, defense lawyer Lakk showed up with no witness; she explained that Dr. Fletcher was a very busy man and had been "unavoidably detained" on a trip to one of Devere's factories in Asia. Latimer rescheduled the deposition for August 3, and Lakk promised to have Dr. Fletcher available that day. On August 3, Lakk again showed up with no witness, explaining that Dr. Fletcher had to take his aged mother to the doctor that day. Latimer rescheduled the deposition for August 14, and Lakk promised: "If I don't have Sigmoid Fletcher with me on August 14, I will eat your socks on the courthouse steps at noon." On August 14, Lakk showed up for the deposition in the company of a nine-year-old boy, whom she introduced as Sigmoid Fletcher. Upon questioning by Latimer, Lakk laughingly explained that young Sigmoid was Dr. Fletcher's son, that the boy knew nothing about the case, and that Dr. Fletcher was "unable to attend." At that point, Latimer invited Lakk to accompany him to see Judge Jackson to whom the case was assigned. After hearing the story, Judge Jackson asked Lakk what day and time the witness would assuredly be available, and Lakk said that August 16 would be good. Judge Jackson then entered a formal order that Dr. Fletcher's deposition would be taken that day; Judge Jackson also warned Lakk that she would be sanctioned if she did not present Dr. Fletcher on August 16, and that the issues of trade secret misappropriation and use would be deemed proven in plaintiff's favor. Alas, on August 16, Lakk again showed up alone, explaining that Dr. Fletcher "had to make an unexpected trip out of state." After appropriate notice and hearing, Judge Jackson ordered that the issues of trade secret misappropriation and use would be deemed proven in plaintiff's favor. She also ordered Dr. Fletcher to pay 60% of the plaintiff's expenses and attorneys' fees incurred because of the failed deposition attempts, and she ordered Lakk to pay the other 40% of plaintiff's expenses and attorneys' fees. Was Judge Jackson correct in holding Lakk subject to litigation sanction?

b. Yes, because Lakk violated the court's discovery order by showing up on August 16 without the witness.

3-34. Criminal defendant Daggs is facing trial for the unlawful possession of a large quantity of a controlled substance---specifically, 400 kilograms of cocaine. Daggs originally asserted that he could not afford to pay a lawyer, and the court arranged for him to be represented by a public defender. Later, Daggs was visited in jail by one Wiggens, who identified himself as a skilled criminal defense lawyer who had been hired by Daggs's "friends" to defend him. Wiggens said that Daggs would not have to pay anything for the legal defense if Daggs "played the game straight" and did not implicate the "friends" in the cocaine caper. Daggs consented to the conditions, and in due course Wiggens was substituted in as defense counsel. Shortly before trial, the prosecutor offered Daggs an attractive plea bargain---a mere six months in jail in exchange for a guilty plea, an identification of the persons for whom Daggs was transporting the cocaine, and testimony against those persons. Wiggens solemnly advised Daggs to reject the plea bargain, saying: "Not only would you owe me a big fee, but your friends would dismember you." Daggs did reject the plea bargain. Thereupon the prosecutor moved to disqualify Wiggens as defense counsel on the ground that he was being paid by an unidentified third party and that the third party was unduly interfering with Daggs's constitutional right to effective assistance of counsel. Is Wiggens subject to disqualification?

b. Yes, because both the prosecutor and the judge have a duty to protect the defendant's constitutional right to the effective assistance of legal counsel.

3-31. Attorney Asrrid represented Charles and Wanda, a married couple, for many years, handling such matters as wills, real estate closings, and the occasional lawsuit. Wanda died suddenly, and shortly after her funeral Astrid paid a visit to Charles, wishing to pay her respects as a friend as well as Charles's attorney. Astrid found Charles in an obviously drunken state. He told Astrid that he just "couldn't go on" without his wife and that he planned to commit suicide that evening. He told Astrid directly, "I know you're a friend and you mean well, but I don't want you to do anything to try to stop me." Using a rose to distract the intoxicated Charles, Astrid called the police, who promptly took Charles to the psychological emergency receiving area of University Hospital. Suicide is a felony in the criminal code of State Brown, where Astrid is licensed to practice law. Was Astrid's conduct proper?.

b. Yes, because her client proposed to commit a crime involving serious bodily harm.

3-37. Jones, the driver of a car, and Smith, his passenger, were injured as the result of a collision with a bus. Jones and Smith believe the bus driver was entirely at fault, and they want to bring a negligence action against the bus company. Jones and Smith seek to hire attorney Adams to represent them. Which of the following would be proper conduct by Adams? I. Accept the proffered employment only after informing Smith that he may have a cause of action against Jones and obtaining Jones's and Smith's consent to represent them both against the bus company. II. Withdraw from the common representation if discovery shows that Smith has a claim against Jones. III. Represent Jones and Smith, and also represent the bus company in a solely unrelated matter before the Transportation Board.

c. I. and II. only.

3-41. A basic tenet of the professional responsibility of lawyers is that every person in our society should have ready access to the independent professional services of a lawyer of integrity and competence. In this regard, which of the following statements are correct? I. A lawyer may charge a reasonable fee since this allows the lawyer to serve his client effectively and preserves the integrity and independence of the legal profession. II. A lawyer should not decline representation because a client or a cause is unpopular or community reaction is adverse. III. A lawyer should not decline employment merely because the intensity of his personal feelings may impair his effective representation of a prospective client.

c. I. and II. only.

3-2. Diplock, a juvenile, has been charged with auto theft, a felony, and has hired Lucinda to represent him. Before the case comes to trial, Lucinda confers with Patrick, who has been assigned by the district attorney's office to prosecute the case. Patrick knows that Lucinda plans to run for public office and is always interested in getting as much publicity as possible. She is often referred to in the press as a "hotshot" criminal defense attorney, and almost all of her victories over the state are publicized. Patrick suggests to Lucinda that, since Diplock is a juvenile, the charge might be reduced to "joyriding," a misdemeanor. Lucinda refuses, telling Patrick, "I win by trying cases. You must not have read the newspapers lately--there's going to be a lot of coverage of tiffs ease because Diplock's father is socially prominent, so I'm not about to cop a joyriding plea for tile kid." The proper conduct for Patrick is to: I. Ask the court to dismiss the auto theft charge and prosecute Diplock for joyriding. II. Prosecute the auto theft charge. III. Send an investigator to talk to Diplock about the offer to lower charges.

c. I. and II., but not III.

3-50. According to the Code of Judicial Conduct, which of the following activities are improper for a judge? I. Teaching a course in advanced trial practice at a seminar for practicing attorneys, for a fee. II. Soliciting funds from the public to support the National Council on Penal Reform. III. Writing an article for a national magazine on fishing in the Great Lakes region. IV. Serving as the weekend manager at a local antique store.

c. II. and IV. only.

3-8. While she was attending law school at night, Adela served as a clerk for Judge Jones of the county court. During the course of her employment by Judge Jones, the case of Lyndon v. Baines came before Judge Jones. The case was very complicated and Adela did a lot of research on the case for Judge Jones, submitting a number of memoranda on issues in the case. Judge Jones always carefully supervised his clerks, and this case was no exception. He was exceptionally pleased with the care and high quality of Adela's work. Shortly after Judge Jones handed down his final judgment on Lyndon v. Baines, Adela was admitted to the bar and she accepted employment with Nixon, Dixon & Yates, a prominent local law firm. A few days later, Baines, who was the loser in the Lyndon v. Baines case, appeared at the offices of Nixon, Dixon & Yates. He told the interviewing attorney, "My lawyer at the trial was terrible. I hired him because he was cheap, and I guess I got what I paid for. I'd like your firm to handle the appeal." If Nixon, Dixon & Yates accepts Baines as a client, and Adela's supervising attorney asks Adela to handle the appeal, would it he proper for Adela to do so?

c. No, because Adela was previously personally and substantially involved in work on the case.

3-39. The Department of Children's Services ("DCS"), a state agency, has removed Clara's children from her home and is attempting to terminate her parental rights. Clara retains lawyer Laura to fight DCS's actions. Prior to the first heating on the matter, a lawyer from DCS contacted Ben, Clara's brother, to set up an interview. The lawyer told Ben that he wanted to talk to him about Clara, and that the interview would last for about one hour. Ben, who is childless, had never had any contact with DCS before, and did not know what his duties were. He told the lawyer he would get back to him. Ben then called Laura. He told Laura that he does not want to talk to DCS because he is afraid he might inadvertently say something that will hurt Clara's case. Ben asked Laura how he should proceed. Laura replied, "My advice to you is to simply refuse the interview. You do not have to talk to them, and your sister will be better off if you don't. If they really want to talk to you, they can get a subpoena." Is Laura subject to discipline?

c. No, because Ben is Clara's brother and he did not want to hurt her case.

3-7. Judge Johnnie heard through the "courthouse grapevine" that the District Attorney was investigating corrupt practices in the courts and that the investigation focused on some as yet unascertained time in the past. The "rumor mill" also indicated that several judges and former judges were likely to be indicted for taking bribes to "fix" cases and to generate business for certain attorneys. Johnnie was alarmed at this news, and he telephoned Abe, a retired attorney, arranging to meet Abs for cocktails and dinner. At the restaurant, Johnnie slipped the maitre d' a 10-dollar bill to secure seating at a secluded corner booth. Over dinner, Johnnie told Abe that he had accepted bribes in the past and that he did not know what he should do in light of the D.A.'s investigation. Abe advised Johnnie to do nothing. Johnnie picked up the $120 dinner tab, and Abe thanked him for the fine meal. A month later, indictments were handed down against two sitting judges and three former judges. Johnnie was not among them, and it turned out that the period covered by the District Attorney's investigation was prior to Johnnie's election to the bench. Six months after the indictments were announced, a member of the state appellate court died, and the Governor announced that he was appointing Johnnie to serve the remaining three years of the justice's unexpired term. Must Abe report his knowledge of corruption in office by Johnnie?

c. No, because Johnnie's disclosures to Abe dealt with past crimes.

3-32. Attorney Alice and attorney Baxter were brother and sister. They were licensed to practice in the Province of Manitoba, Canada. For five years they practiced together in a partnership in Winnipeg, Manitoba. Alice then moved to International Falls, Minnesota, where she passed the Minnesota Bar Examination and was duly licensed to practice law in the state of Minnesota. She opened up a law office in International Falls and had letterhead stationery printed which read "Alice and Baxter, Partners, Attorneys at Law." Is it proper for Alice to use such letterhead?

c. No, because the letterhead indicates Alice is aiding the unauthorized practice of law.

3-12. The city of Auburn is located in State A. The city of Burnett is located right across the river in State B. Many people who live in Burnett work and shop in Auburn, and vice versa. Lawyer Alford is admitted to practice in State A, but not State B, and Alford has his law office in Auburn. When he is asked to represent a criminal defendant in the Burnett Superior Court, he customarily refers the case to lawyer Bemis. Bemis has his law office in Burnett. He is admitted to practice in State B, but not State A. Likewise, when Bemis is asked to represent a client in the Auburn Municipal Court, he refers the case to Alford. Each customarily sends the other a small referral fee. The precise amount of the fee varies in accordance with the gravity and complexity of the case. In no case does the referral fee increase the amount paid by the client. Bemis and Alford have evolved their mutual arrangement in order to provide better service to their respective clients. Is the arrangement proper?

c. No, because, on the facts given, the referral fee could not be proportionate to the services rendered by the referring lawyer, and there is no indication of joint representation.

3-27. Lawyer Munoz was employed to represent Claus Schnauffier at his trial for treason. Schnauffier was charged with smuggling top secret military information to a foreign government. Munoz had reason to suspect that some of the prosecution's witnesses were paid liars. Therefore, with Schnauffier's consent, Munoz hired Dr. Clara Belle to assist him in the defense. Dr. Belle is a psychologist whose specialty is the behavior of liars. Munoz agreed with Schnauffier that Munoz would advance Dr. Belle's fee as a part of the expenses of litigation. At the trial, Dr. Belle sat with Munoz at the counsel table. She watched the witnesses testify, and she advised Munoz when she believed that a witness was lying and ought to be pursued on cross-examination. Most of the time, Munoz followed Dr. Belle's advice, but sometimes he did not. Ultimately, Schnauffier was convicted and sent to prison for 20 years. Which of the following propositions are correct? I. Munoz is subject to discipline for allowing a third party to interject herself into the relationship between him and his client. II. Munoz is subject to discipline for advancing Dr. Belle's fee as a part of the expenses of litigation. III. Munoz is subject to discipline for allowing a nonlawyer to sit at the counsel table and to participate in his legal representation of a criminal defendant.

c. None of the above is correct.

3-17. Defendant Devereaux was charged with first degree murder. It is claimed that Devereaux captured his victim, dragged her into dense woods, and stabbed her with a knife. Devereaux has pleaded not guilty. Pamela is the prosecutor, and Denise is Devereaux's defense attorney. During the prosecution's case-in-chief, one of Pamela's witnesses testified that he had seen Devereaux's car near the scene of the crime shortly before the murder. This courthouse has no private room set aside for conferences between defendants and their counsel. Thus, at the next recess, Denise and Devereanx held a hurried, whispered conference in the hallway, as follows: Denise: "Were you driving around that area at the time?" Devereaux: "Yes." Denise: "Why didn't you tell me that before?" Devereaux: "I didn't realize that anybody saw me." Unbeknownst to Denise and Devereaux, Pamela was standing nearby and overheard their whispers. Which of the following are correct? I. Pamela should seek the court's permission to withdraw as trial counsel and testify to what she heard. II. Pamela should ignore what she heard and proceed with the case in the normal manner. III. Denise should seek the court's permission to withdraw as trial counsel and inform the trial judge in chambers what Devereaux said. IV. Denise should proceed with the case in the normal manner, even though Devereaux's statement may cause her to doubt his innocence.

c. Only II. and IV.

3-22. Attorney Patricia limits her law practice to the representation of plaintiffs in actions for medical malpractice. She has developed a standard employment contract to use with all clients who desire a contingent fee arrangement. The contract requires the client to pay in advance the first $500 of litigation expenses, and states that Patricia's fee shall be 20% of any amount recovered without going to trial, 30% of any amount recovered if the case goes to trial, and 35% of any amount recovered if the case is appealed. Client Nesbitt signed Patricia's standard form contract. After putting in only 10 hours' work on Nesbitt's case, Patricia was able to work out a settlement agreement in which Nesbitt received $10,000 from the defendant' s malpractice insurance carder. Nesbitt was delighted with the settlement, but he is unhappy with having to pay 20% of it ($2,000) m Patricia for so few hours' work. With respect to the clause of Patricia's standard form contract that requires the client to pay in advance the first $500 of litigation expenses, which of the following is most nearly correct?

c. The clause is proper, unless $500 is a clearly excessive estimate of the amount of litigation expenses that will be incurred in the ordinary medical malpractice case.

3-28. Hal and Wilma, a married couple, experienced continuing marital difficulties and, being unable to resolve their problems, decided to divorce. Wilma retained attorney Ambrose, and Ambrose filed papers on Hal. Shortly after he was served, Hal telephoned Ambrose and suggested that "all this can be resolved amicably.'' Ambrose asked Hal if he was represented by counsel. Hal replied that he was not, to which Ambrose said, "I strongly advise you to retain an attorney to represent you and safeguard your interests." Hal insisted, "I don't need a lawyer." A few weeks later, after ascertaining that Hal still had not retained counsel, Ambrose sent Hal a proposed "Division of Marital Property." There were spaces at the bottom for both Hal and Wilma to sign, but the copy sent to Hal was unsigned by Wilma. In his cover letter to Hal, Ambrose indicated that Hal could sign the settlement agreement, which Ambrose characterized as "fair and equitable." However, Ambrose also stated in the letter that, "I am not your attorney and therefore cannot advise you whether or not you should sign this, and I strongly urge that you obtain independent counsel to review these papers before you sign them. However, Hal, the choice remains yours, and I cannot compel you to hire an attorney to protect your interests." Were Ambrose's actions in handling Hal and Wilma's divorce proper?

c. Yes, because Ambrose did not give Hal legal advice.

3-45. Attorney Allison, a member of the state bar, and Judge Jack, who sits on the district court, set up "The Profitable Probate School" ("PPS"). The announced purpose of the school was to educate laypersons as to probate procedures, to enable them to file their own papers and avoid the expenses of obtaining counsel for the probate process. The tuition was a modest $50 per student, which barely covered the expenses of books and forms handed out to the students. Despite the "Profitable" in PPS's name, Allison and Jack donated their time and received no remuneration from the school. However, each student was required to sign a paper that stated, "The $50 tuition fee establishes an attorney client relationship between an attorney teaching at PPS and all students. PPS and attorneys teaching therein are not liable for any damages that might be incurred by students as a result of pro se filings made in accordance with or at variance from instructions given by PPS." The statement was signed by Allison and Jack as "officers of the school." Is Allison subject to discipline?

c. Yes, because an attorney may not limit her malpractice liability by contract.

3-46. Attorney Anita is a fully licensed member of the state bar, but she is a rather junior associate with the prominent firm of Alpha, Beta, and Gamma, which handles many security matters. Anita interviewed Curry, a client who brought an exceedingly complex securities matter to the firm. Although Anita was highly competent, a couple of points in Curry's complex case were beyond her range of experience. Therefore, she consulted Gamma, one of the firm's senior partners, regarding these complicated issues. Gamma gave her excellent advice on how to handle the matters based upon his 30 years of experience in securities law. Anita continued to handle Curry's case and brought it to a successful conclusion. Was Anita's conduct proper?

c. Yes, because she consulted with a lawyer in her own firm who had no reason to be screened from the case.

3-25. Lawyer Lewellen represented defendant Devonshire in a felony case that charged him with intentional evasion of over $9 million in state income tax. Devonshire allegedly earned the unreported income from selling child pornography. The state's criminal procedure law requires a unanimous jury verdict to convict in a felony case. Midway through the prosecutor's case-in-chief, Lewellen instructed one of Devonshire's employees, Snagg, to attend the trial daily, to sit in the spectator section as close as possible to the defense counsel table, and to "put the evil-eye on Juror No. 4, the little guy with the pink nose." Snagg, a wicked-looking man with a jagged scar down the side of his face, attended every court session and stared constantly with cold, squinting eyes at Juror No. 4. One day, when no one but Juror No. 4 was looking, Snagg pointed his hand like a pistol directly at Juror No. 4's head, and pretended to shoot by moving his thumb downward. Then Snagg smiled an evil smile and continued to stare. Juror No. 4 was too frightened to tell anyone, but he endured the torment day after day. Finally the case was submitted to the jury, and Juror No. 4 joined the rest of the jurors in returning a prompt, unanimous verdict of guilty. Is Lewellen subject to criminal liability for what Snagg did?

c. Yes, but only if it is proven that Lewellen induced Snagg to intimidate Juror No. 4 for the purpose of influencing the jury verdict.

3-6. The state in which attorney Lawrence practices levies an annual tax on trusts for the benefit of minors. Tax returns must be filed, and the taxes must be paid, by March 15th; late filing results in an automatic penalty of 15%. In mid January, trustee Morris retained attorney Lawrence to prepare and file a tax return. Morris heard nothing from Lawrence during February, and he became seriously alarmed when the first week of March passed with no apparent action from Lawrence. He called Lawrence repeatedly during late February and early March, but the secretary always said that Lawrence was "in conference," "in court," or "in deposition." Lawrence never returned any of Morris's phone calls. On March 10th, Morris fired Lawrence and hired Donita to do the necessary work. She was able to complete the tax return and get it filed on time. Is Lawrence subject to discipline?

c. Yes, even if he would have been able to complete the necessary work between March 10th and March 15th.

3-20. After a major airplane crash in the vicinity of Biggston, in which 122 passengers and crew were killed, Biggston's attorneys swarmed like locusts to get a "piece of the action" and the potentially huge contingent fees that were likely to arise from the case. Interested in fees himself, but also rather disgusted at the performance of some of his colleagues of the bar, attorney Axe/ placed an ad in the Biggston Law Journal, a weekly newspaper whose readership was almost entirely lawyers. The ad suggested that any attorneys representing plaintiffs in the airline crash matter contact him in order to consolidate lawsuits against the airline. The ad ended with a statement, "This should materially benefit everyone in that it should reduce court costs and encourage prompt settlements for our clients." The ad was signed by Axel and indicated his office address and telephone number. Was it proper for Axel to place such an advertisement?

c. Yes, provided the ad was not misleading.

3-13. Attorney Bob was engaged in a partnership law practice with his sister, Sarah, for almost 10 years. Sarah then decided to run for judge. She was successful in her efforts and was duly sworn in as one of the 15 sitting judges on the district court. According to state venue rules, at least 90% of the cases that Bob usually handles must be filed with the district court. Would it be proper for Bob to continue to handle such cases and appear before the district court?

c. Yes, unless Bob implies that his clients will receive unfair advantages because his sister is a judge.

3-40. Attorney A is representing C. Judgment at the trial court is against C. C wants to appeal the case. The appellate court rules provide a 60-day deadline for the filing of appeals, and no exception is ever granted. During the first 30 days of the period, A was frantically busy in his office on other matters, and he had no chance to file the appeal. Then, during the second 30 days, he went on vacation and simply forgot to file the appeal. The trial court's error was so obvious that the appellate court would undoubtedly have reversed the case and entered judgment in C's favor. Which of the following is most nearly correct?

d. A is both subject to discipline and liable for malpractice.

3-30. Three months from now, Judge Torpor will be up for reelection. His opponent is Vernon Vigoroso, a bright and ambitious young attorney in the prosecutor's office. Due to the press of his judicial duties, Judge Torpor has not paid much attention to the upcoming election. One afternoon an old law school friend, Getta Movin, visits him in his chambers. She convinces him that he must get busy if he hopes to defeat Vigoroso. Getta frequently represents clients in Judge Torpor's court, and he trusts her judgment and ability; thus, he agrees to have her serve as chairperson of his reelection campaign committee. Which of the following would be proper in the context of Judge Torpor'a campaign for reelection? I. Getta contributes $200 of her own money to the campaign fund. II. Getta urges other lawyers who appear in Judge Torpor's court to make contributions to his campaign fund. III. Getta urges other lawyers who appear in Judge Torpor's court to allow their names to be listed in a half-page newspaper advertisement in favor of Judge Torpor's reelection. IV. Getta urges the local newspaper that will run the half-page advertisement to give her a discount from the regular commercial advertising rate because her advertisement concerns a campaign for public office.

d. All of the above.

3-48. L, P's attorney, knows that the statute of limitations on P's claim against D has run. However, the statute of limitations is an affirmative defense that D would waive if he failed to plead it. What should L do?

d. File the suit if P is willing to incur the legal fees and court costs.

3-21. Sales manager Morton testified before a federal grand jury that was investigating pricefixing in the automobile tire industry. Ultimately the grand jury indicted Morton for price-fixing, a felony under the Sherman Act. After his indictment, Morton sought to hire attorney Agnes to represent him at his criminal trial. Morton is a middle class business executive with enough savings to pay for private counsel. He told Agnes in confidence that he had lied to the grand jury about several meetings he had had with competitors. Further, he told her that he wanted to plead not guilty to the criminal charge, and he told her that he intended to testify as he did before the grand jury. Which of the following would be proper for Agnes to do in this situation? I. Decline to represent Morton. II. Agree to represent Morton, and represent him as best she can at the criminal trial. III. Inform Morton that unless he pleads guilty to the criminal charge, she will tell the prosecutor about his false testimony before the grand jury. IV. Decline to represent Morton, and inform the prosecutor about his false testimony before the grand jury.

d. I. only.

3-44. In which of the following situations is the attorney-client privilege applicable? I. Client C telephones her accountant and has him deliver tax returns and work papers to attorney A, who has been engaged to represent C in a tax case. II. Client C hires lawyer L to represent him in a relatively simple tax case. L employs his son A, a newly certified accountant, to help him with the case. At L's request, C discusses certain tax accounting facets of the case with A. III. Client C discloses confidential information to attorney A with the intention of hiring A to represent him. A, however, decides not to take the case.

d. II. and III. only.

3-11. Assistant District Attorney Krytzer has been assigned to prosecute Abdala for petty theft and attempted sale of stolen property. Abdala's arrest was the result of information provided by O'Leary, a local pawnbroker. O'Leary has himself been in trouble with the law on prior occasions, but he is not suspected of any present crime. Abdala is represented by Public Defender Fuchinello. Krytzer wants to interview O'Leary for possible use as a prosecution witness. Which of the following is most nearly correct?

d. Krytzer may interview O'Leary without Fuchinello's consent.

3-33. Lewis, an attorney, was formerly employed by the Environmental Protection Agency ("EPA") as government counsel in the New York office. In this capacity, Lewis acted as chief counsel in several suits brought by the EPA involving chemical dumping into public waterways. Two years after leaving the employ of the EPA, Lewis was retained to represent Acme Chemical Corporation in a suit brought by the EPA alleging violations of certain EPA regulations regarding the dumping of chemical wastes. While with the EPA, Lewis was never directly involved in a case concerning Acme Chemical Corporation. The EPA Washington office had exclusive responsibility for the drafting, promulgation, and enforcement of the regulations in question. Which of the following statements is correct?

d. Lewis may represent Acme, whether or not the EPA consents.

3-24. Before Judge Jerkins was elected to the bench, she and her law partner, Perkins, purchased a piece of property to be held in cotenancy by Perkins and Jerkins. After Judge Jerkins was elected to the bench, Judge Jerkins agreed to pay Perkins an annual fee to manage the property, since Judge Jerkins's time would be severely limited by her judicial duties. Judge Jerkins and Perkins meet every three months to discuss the status of the property. Perkins sometimes appears as an attorney in Judge Jerkins's courtroom. Was it proper for Judge Jerkins to make this arrangement with Perkins?

d. No, because Perkins appears in cases before Judge Jerkins's court.

3-18. Attorney April represents Chuck, plaintiff in a personal injury suit arising out of an automobile collision. Chuck asserts that he had the right-of-way to enter the intersection where the accident occurred. The defendant claims otherwise, but Chuck tells April that there was a witness present who would be able to verify Chuck's version of the accident. April obtains a copy of the police report on the accident, but the name of the witness is not contained in the report. April contemplates running an ad in the newspaper or hiring an investigator to find the witness, but April concludes that Chuck's testimony ought to be strong enough to win the case. The case comes to trial, and the jury finds for the defendant. Is April subject to discipline for failure to try to find the witness?

d. No, if she reasonably believed that Chuck's testimony would be sufficient.

3-29. Walters is insured under an auto liability policy issued by Farmers Insurance Company. The policy requires Farmers to provide a lawyer to defend Waiters, and it requires Walters to cooperate in the defense. Walters had an accident and was sued. In a sworn statement to Farmers' insurance investigator, Walters told a story that showed he was clearly not at fault. Based on that story, Farmers rejected plaintiff's offer to settle the case for a modest sum. Farmers hired attorney Chen to represent Walters at the trial of the case. Shortly before trial, Walters told Chen in confidence that he had lied to the investigator, and he recounted facts that showed he was clearly at fault in the accident. Chen realized that under the applicable state law, Walters's falsehood was a breach of the "cooperate in the defense" clause, and that it relieved Farmers of any further duties to Walters. At this juncture, what should Chen do?

d. Promptly seek the court's permission to withdraw from the matter, without revealing Walters's confidential statement to anyone.

3-23. Charlene was involved in an aviation mishap. The airline company has admitted liability and has settled with 10 other persons involved in the mishap for amounts ranging between $120,000 and $150,000. Charlene's injuries are very similar to those suffered by the persons with whom the airline has settled. Charlene received a settlement offer of $135,000 from the airline company. Upon receiving the offer, Charlene decided to employ counsel to determine if the offer was a fair amount, and generally to read over the settlement papers and the release that the airline asked Charlene to sign to get the $135,000. Charlene went to the offices of attorney Alp, bringing the settlement papers and release with her. She asked Alp what his hourly fee for reading the papers would be. Alp responded that Charlene had a personal injury case and that Alp's standard fee for personal injury cases was 30% of any settlement or judgment received by the plaintiff. Is Alp subject to discipline?

d. Yes, because Alp's fee bears no rational relationship to the time and effort required to perform the work requested by the client.

3-43. Lawyer Lucy represents client Charlie, who is a writer and producer of Broadway stage plays. This morning Charlie telephoned Lucy and said: "Great news, Lucy! Anthony Hopper, the famous actor, has agreed to star in my new stage play, Guardian Angel. With Anthony Hopper in the lead, the play is certain to be a long-running blockbuster. Don't tell a soul because it's not public information yet. Please draft up the necessary legal papers and meet Hopper and me for dinner at 6 p.m. at the Franklin Club." After drafting the legal papers as Charlie requested, Lucy telephoned her friend Mitchell, who owns one of the best theaters on Broadway. Lucy said: "Mitch darling, I have a tip for you, but it will cost you $5,000. Are you buying?" Mitchell agreed to pay Lucy $5,000 for the tip, at which point she said: "Make a deal with Charlie to run his new play, Guardian Angel, in your theater. Set the rent so low Charlie can't refuse---even at a low rent you will make millions because Guardian Angel is going to be a long-running blockbuster. I can't tell you how I know that, but trust me." Mitchell promptly made the deal with Charlie, and he paid Lucy the $5,000. Is Lucy subject to civil liability to Charlie for selling the tip to Mitchell?

d. Yes, because Lucy profited by $5,000 from trading on Charlie's confidential information. She can be ordered to disgorge her profit to Charlie.

3-35. Cesar comes to attorney Abigail with an invention he wants to patent. Cesar explains that he and a competitor have been racing one another to come up with the ideal cleaning solution. If Cesar's competitor were to find out that Cesar was at the patent stage, and worse, if he found out Cesar's formula, Cesar would be mined. Abigail, a trained and certified patent attorney, agrees to represent Cesar in the patent process. The invention involves complex chemical formulae, and Abigail's particular area of expertise is electronic devices. However, having worked with inventions of all types, she has no doubt that she can properly shepherd the solution through the patent process. In putting together the necessary paperwork, Abigail asks Barbara and David, who are associates in her firm and hold chemistry degrees, to help her out on the project. In due time, Cesar's product receives a patent. Cesar's total bill for legal fees was $60,000, which was reasonable for the work done. When Abigail received Cesar's final payment, she decided to give Barbara and David each a $10,000 bonus from the fee. Are Abigail's actions proper?

d. Yes, because with the aid of her associates, she was competent to handle the matter.

3-5. Attorney Alex has a high-profile divorce law practice in Centerville. Because of his heavy case load, Alex often appears before the four chancery judges of the county court. One of the chancery judges, Jamal, is getting married, and he sends a wedding invitation to Alex. Alex wishes to send Jamal, as a wedding gift, an imported Italian machine that makes espresso and cappuccino coffee because Alex knows that Jamal loves fine coffee. The coffee machine sells for $200 at Centerville's best cooking equipment store. Would it be proper for Alex to send the coffee machine to Jamal?

d. Yes, but only if Alex and Jamal are friends and only if the gift is not excessive for the occasion and the relationship.

3-26. Attorney Aria placed an advertisement in the Clarion, a newspaper of general circulation published in Old Salem, the city in which Aria practiced. The ad contained Arla's name, office address, telephone number, and the following additional information: (i) "Attorney Arla is a graduate of State University Law School"; (ii) "Attorney Aria has an M.B.A. from the Graduate School of Business of Laffercurve University''; and (iii) "Attorney Arla is the only lawyer in town who speaks fluent Spanish." Is Arla's advertisement proper?

d. Yes, if all statements contained therein are true.

3-3. Diplock, a juvenile, has been charged with auto theft, a felony, and has hired Lucinda to represent him. Before the case comes to trial, Lucinda confers with Patrick, who has been assigned by the district attorney's office to prosecute the case. Patrick knows that Lucinda plans to run for public office and is always interested in getting as much publicity as possible. She is often referred to in the press as a "hotshot" criminal defense attorney, and almost all of her victories over the state are publicized. Patrick suggests to Lucinda that, since Diplock is a juvenile, the charge might be reduced to "joyriding," a misdemeanor. Lucinda refuses, telling Patrick, "I win by trying cases. You must not have read the newspapers lately--there's going to be a lot of coverage of tiffs ease because Diplock's father is socially prominent, so I'm not about to cop a joyriding plea for tile kid." If Diplock is convicted of auto theft, would it be proper for Patrick to report Lucinda to the disciplinary authorities?

d. Yes, if he knows that Lucinda never conveyed the offer of a lesser charge to Diplock.

2-6. Client Crowell made a preliminary contact with lawyer Lear to see if she wanted to hire Lear to defend her in a tort case that had been assigned to Judge Johnson. Lear told her that the initial consultation was free of charge. After listening to Crowell's brief outline of the case, Lear told her: I know how to get a favorable decision from Judge Johnson. He will be running for reelection 18 months from now, and he will need money for his campaign. You should send him a $2,000 campaign contribution now, with a nice note wishing him well in his bid for reelection. Johnson's opponent in the election will be a local lawyer, Willard Wampler. Wampler is an honest fellow, but I know that his two brothers are associated with organized crime. I can write a guest editorial for the local paper, praising Johnson's .judicial record and implying that Wampler is a crook. With your contribution and my letter, I think we can count on Judge Johnson to reach a wise decision in your case. Crowell hired Lear and sent Judge Johnson the $2,000. Lear wrote the guest editorial, and it was published in the local paper. For which of the following is Lear subject to discipline? I. Saying that he knew how to get a favorable ruling from Judge Johnson. II. Advising Crowell to send Judge Johnson a campaign contribution. III. Writing the guest editorial. IV. Accepting the case following free legal advice.

(A) I., II., AND III. only

1-28. Lawyer Lorenz agreed to represent wife Withers on an hourly fee basis in securing a divorce from her husband Hullar. Hullar is represented in the matter by attorney Atwell. Despite repeated warnings by Lorenz, Withers kept pestering Lorenz with telephone calls and office visits concerning inconsequential details and trifling personal complaints. When Withers was unable to contact Lorenz on the phone or in person, she would telephone Atwell, her husband's attorney, and try to put her questions and complaints to him. Atwell always refused to talk to Withers. Lorenz repeatedly told Withers not to contact Atwell, but to no avail. Finally, Lorenz told Withers that she would withdraw unless Withers changed her ways, but Withers did not do so. Lorenz withdrew and sent Withers a fee bill for the total number of hours she had spent on the case. Withers refused to pay the bill, and after futile efforts to settle the matter, Lorenz sued her to collect the fee. Which of the following propositions are true? I. It was proper for Lorenz to withdraw. II. It was proper for Atwell to refuse to talk with Withers on the phone. III. It was proper for Lorenz to bill Withers for the total amount of time she spent on the case. IV. It was proper for Lorenz to sue Withers to collect the unpaid fee.

(A) I., II., III., and IV.

4-1.Attorney Aggie is prosecuting a complex tax case. After two government witnesses testified, the defendant, Daft, took the stand in his own defense. Daft asserted that he always complied with all tax rules and regulations. He also testified that the two government witnesses had deliberately falsified Daft's Internal Revenue Service records because the witnesses belonged to a Satanic cult which bore a long-standing grudge against Daft. In Aggie's closing argument, she made the following statements relative to the defendant's testimony: I. "Defendant's testimony is clearly in conflict with the testimony of two government witnesses." II. "Of the persons who have given testimony, who has the better reason to lie, the defendant or the government's witnesses?" III. "If you believe the testimony of the defendant, you will also believe the Moon is made of green cheese!" For which, if any, of these statements would Aggie be subject to discipline?

(A) III Only

1-42. Lawyer Lawrence is a partner in the firm of Lawrence and Loeb. That firm regularly provides legal services to three major banks and two other important lending institutions in the community of Farmdale. Lawyer Lawrence has been invited to become a member of the board of directors of the Farmdale Legal Aid Society, the group that sets overall governing policies for the local Legal Aid office. One of the major issues that will soon face the board of directors is whether to amend the Case Intake Guidelines to allow the Legal Aid office to represent clients in disputes with banks and other lending agencies. Which of the following statements is most nearly correct?

(A) Lawrence may join the board of directors, but she should refrain from participating in the decision about the Case Intake Guidelines

1-5. Pringle consulted lawyer Louder, hoping to hire Louder to represent him as plaintiff in a medical malpractice action against Dr. Dooley. Without mentioning Dr. Dooley's name, Pringle described the alleged acts of malpractice and said that they happened more than two years ago. Only at that point did Pringle mention Dr. Dooley's name. Louder immediately stopped Pringle and said: "I am sorry, but I cannot represent you in this matter. I am presently representing Dr. Dooley in an entirely unrelated lawsuit, but that means I cannot represent you. You will need to consult another lawyer." That was the end of the conversation. Pringles did nothing further for 15 months, at which point he consulted another lawyer. By that time, the statue of limitations had run on Pringle's claim against Dr. Dooley. Pringles then sued Louder for legal malpractice, alleging that Louder was negligent in not warning him about the statute of limitations. Is Louder subject to civil liability in Pringle's malpractice case?

(A) No, because Louder did what a reasonably prudent lawyer would do in the circumstances--decline to represent Pringle and suggest that he consult other counsel.

1-34.Union Bank and Trust Company maintains a list of Approved Estate and Trust Lawyers as a service to Union customers who seek Union's advice on estate planning matters. When lawyer Lieu opened her trust and estate practice in town, she asked other lawyers how to get on Union's approved list. They explained that Union lists lawyers who regularly name Union in wills and trust agreements they draft for clients who need an institutional executor or trustee. Union is one of the most stable and reputable banks in the state, and its fees for executor and trustee services are competitive with those of similar institutions. In light of what she has been told by the other lawyers, may Lieu seek to have her name included on Union's list?

(A) No, because a tacit condition of being on the list is regularly to name Union as executor or trustee.

2-19. L'Etoille Women's Wear, Inc. is incorporated in and has its principal place of business in State A, and it is represented by a lawyer who is licensed to practice in State A. L'Etoille owns the valuable, federally registered trademark L'Etoille for use on various items of women's wearing apparel, and it licenses the trademark to garment makers in various states for use on items of apparel that are manufactured in accordance with style and quality specifications set by L'Etoille. Lawyer Londrell is licensed to practice only in State B. She represents DonnaDell Clothing Co., which is incorporated in and has its sole place of business in State B. Londrell and the president of DonnaDell traveled to State A, where they negotiated with L'Etoille's lawyer for a license to use its trademark on apparel to be manufactured at DonnaDell's plant in State B. Londrell is not licensed to practice before the United States Patent and Trademark Office. The license agreement between L'Etoille and DonnaDell provides that the agreement is to be construed in accordance with the law of State A, and that any disputes arising under the license will be arbitrated in State A. Is Londrell subject to discipline?

(A) No, because admission to practice in State A was not necessary to negotiate the trademark license there.

2-36. Lawyer LeBrille is admitted to practice in State A. One of her regular clients is Chatsworth Inc., which is incorporated in and has its principal place of business in State A. The president of Chatsworth went to France to negotiate a business contract for Chatsworth that would be governed, in part, by the law of the European Community ("EC"). The president telephoned LeBrille to ask whether a particular provision of the proposed contract would be lawful under EC law. The president needed a quick answer because he had to resume the contract negotiation a few minutes later. LeBrille had studied EC law, but she was not admitted to practice in any nation that is a member of the EC. LeBrille warned the president about the danger of relying on off-the-cuff, unresearched legal advice, but he asked her to do the best she could. She then advised him that the contract provision would be lawful under EC law. The president thanked her, continued the contract negotiation, and signed a contract for Chatsworth that included the questioned provision. As it turned out, LeBrille's advice was mistaken: the provision violated EC law and rendered the contract unenforceable. Is LeBrille subject to discipline ?

(A) No, because she did the best she could in an emergency situation

2-24 The M&S law firm represented United Oil Company in a merger transaction in which United acquired all the assets of Mogul Petroleum Inc. in exchange for a specified amount of capital stock of United... Are Lurner and M&S subject to disqualification?

(A) No, because the merger matter and the discrimination case are unrelated matters and because M&S did not gain confidential information from United that would be material in the discrimination case.

4-30. Wanda is greatly distraught since her late husband Harry provided only a $1,000 bequest for her in his will. Under the terms of his will, the rest of his substantial estate (over $1 million) will go to a fraternal lodge of which Harry was a lifelong member. Wanda consults Anita Attorney. Anita studies the will and determines that there has been an important flaw in its execution. Anita successfully challenges the will's validity, and the probate court rules that Harry's estate will descend by the laws of intestate succession, which means, in this jurisdiction, that Wanda will take all of Harry's estate. Anita charges Wanda a modest fee, which Wanda pays promptly. Wanda, however, is so pleased with Anita's work that she wants to give Anita a substantial gift as a token of her gratitude. Wanda goes to a local car dealership and purchases a new car worth approximately $500 vase. She smilingly presents the car to Anita Attorney. Will Anita be subject to discipline if she accepts the vase?

(A) No, because there is nothing wrong with an attorney accepting a gift from a client.

1-50. Police officer Owens was charged with murder. He is alleged to have savagely beaten and ultimately killed a teenage gang member in the course of an arrest. Neither the police department nor officer Owens's union was willing to provide legal counsel for his defense, and Owens himself lacked funds to hire private counsel. The public defender's office could not represent him due to a conflict of interest from a related case. The trial court therefore appointed lawyer Lee to defend Owens. Lee is only three years out of law school. Lee practices criminal defense, but he has never handled a murder case before. For which of the following reasons may Lee decline the court appointment'? I. Based on what he has read in the newspapers, he sincerely believes that Owens is guilty. II. He has no experience in the defense of a murder case. III. He is of the same race as the teenage victim, and he is in sympathy with the plight of young gang members. IV. Many of his clients are of the same race as the teenage victim, and they will be irate if he defends Owens.

(A) None of the above

2-49. Defendant Dennis asked lawyer Liston to defend him in a criminal case in which Dennis was charged with running a gambling operation. Dennis was known in the community as a wealthy person, but one who seldom kept his word and seldom paid his bills. Liston agreed to do the necessary work for a fiat fee of $5,000, which was reasonable in light of the difficulty of the case and the number of hours required. However, Liston required that Dennis pay $3,000 in advance. When Dennis protested that he did not have that amount available in ready cash, Liston accepted Dennis's full-length fur coat in lieu of the cash advance. The fair market value of the coat is $3,000, and Liston agreed to return it upon payment of his fee. Their agreement was reduced to writing and signed by both parties. After Liston had put in considerable time in preparing the case for trial, Dennis fired Liston for no good reason and refused to pay him anything for the work already done. Assuming the reasonable value of Liston's services to date is $4,000, which of the following statements is correct? I. Liston is subject to discipline for demanding that Dennis pay $3,000 in advance, before any legal services had been rendered. II. Liston is subject to discipline for taking the coat in lieu of cash as an advance on legal fees. III. Liston must promptly return the coat to Dennis upon termination of their relationship, and submit a bill for his services. IV. If Liston returns the coat and sues Dennis to collect his fee, Liston will be entitled to recover the full contract price ($5,000) because Dennis terminated the contract without good cause.

(A) None of the above.

2-38. Lawyer Lacey is on the in-house legal staff of Transcorp, Inc. In that capacity, she works daily with Transcorp's top executive officers. She was assigned to defend Transcorp in a lawsuit brought by West America Bank to collect a $750,000 promissory note. The note was signed on behalf' of Transcorp by Willard Westerman, Transcorp's treasurer and chief financial officer. Transcorp's defense is that Westerman had no authority to sign the note and that the bank knew it. Transcorp has advised Westernlan that it may seek indemnification from him if it is held liable to the bank. Westerman is not represented by counsel. Shortly before Westerman was to have his deposition taken by the bank, Westerman called Lacey and asked her what to expect at the deposition and how to respond to the bank's questions. What should Lacey do'?

(A) Not discuss the matter with Westerman, and, if appropriate, advise him to hire a lawyer to represent him at the deposition.

4-34. Assume that during the questioning, Booker not only admitted to embezzling funds, but also described company-wide accounting practices that could subject Hyrax to civil and criminal liability. In an effort to avoid publicity and liability, Hyrax fired Booker but agreed not to turn the matter over to the police. Hyrax has since rectified its accounting problems. Sometime later, Hyrax was sued by Cable Corp., one of its creditors, for fraud based on the above accounting practices. During the trial, Cable's attorney calls Arnold to the stand to testily, about his conversation with Booker. Hyrax's attorney objects, claiming attorney-client privilege. The objection should be:

(A) Sustained, because Booker talked to Arnold at Potsdam's request and his statements concern accounting.

4-31. Newshound regularly wrote the "Courts and the Law" column that appeared twice each week in the Daily Bugle, a newspaper of general circulation in the Bricktown area. Newshound approached Arlington, an attorney with offices in Bricktown. Newshound told Arlington that Arlington's name would appear frequently in Newshound's column in a favorable light, if Arlington would supply Newshound with "behind the scenes" items about local .judges, lawyers, and important or otherwise interesting cases. Arlington readily agreed to Newshound's proposal, and Arlington began supplying information to Newshound. Arlington's name did appear often in Newshound's column, and Arlington was characterized as a "top legal eagle," an "outstanding trial tactician," and otherwise touted in the newspaper. Is Arlington subject to discipline?

(A) Yes, because Arlington has given consideration for favorable publicity.

4-40. Lawyer Lapin represented defendant Defiance Holding Corp. in a West Dakota civil case brought by three named plaintiffs suing on behalf of a class of similarly situated people. The plaintiffs allege that Defiance owns nearly 100 apartment houses throughout West Dakota and that it refuses to rent apartments to persons of color, in violation of the West Dakota discrimination law. The civil discovery rules of West Dakota do not require voluntary document production--i.e., a litigant does not have to produce a document until the adversary asks for it in a timely, specific document request. Immediately after the complaint was filed, and before any discovery had started, Lapin made a quick but careful investigation of the types of records Defiance keeps concerning tenants and prospective tenants. She found that, starting more than 20 years ago, the manager of each Defiance apartment house filled out a paper "application form" for each person who wanted to rent an apartment. The application form called for information about the person's age, sex, marital status, race, religion, current and past employment, and approximate yearly income. About eight years ago, Defiance started entering information from the application forms into a computer system, but the computer version omits information about the person's race and religion. The computer system makes it unnecessary to keep and store the paper application forms. Nonetheless, through inertia and corporate ineptitude, Defiance has carefully preserved all of the paper application forms for the past eight years. Lapin advised her client as follows: "I suggest that you get rid of those old paper application forms before the plaintiffs get around to requesting document production. The paper forms are of no use to you, and they could prove embarrassing if the plaintiffs get hold of them." Defiance did as Lapin suggested. Is Lapin subject to criminal liability for suggesting the destruction of the paper application forms?

(A) Yes, because Lapin knew that the paper application forms were relevant and would probably be requested by the plaintiffs.

4-44. Arlington is an attorney engaged in private practice in the city of New Novgorod. Arlington has many friends who belong to the Ancient Society of Sultans, a fraternal and charitable organization with chapters throughout the state. Arlington is not a member of the Society, but knows a number of its officers socially and has performed legal work for them on matters unrelated to the Society. The officers of the Society are sometimes consulted by members who have legal problems. The officers, being very pleased with the quality of Arlington's work, often refer such members to Arlington. Arlington has never asked the officers for such referrals, but is, of course, very pleased since he has earned substantial fees from these referrals. The Society is presently organized as an unincorporated association, but the leaders are interested in incorporating the Society under the State Nonprofit Corporation Act. One of the officers asks Arlington what his fee would be for incorporating the Society. Arlington tells the officer that he is very grateful for the client referrals from the Society and, as a token of his appreciation, he will not charge a fee for the incorporation work. Is Arlington subject to discipline?

(A) Yes, because an attorney must not give something of value in return for client referrals.

1-38. Lawyer Ladner represents defendant Dewey in a criminal case. Dewey is charged with vehicular homicide, a felony... Is Ladner subject to criminal liability for inducing Battista to testify falsely?

(A) Yes, because both Battista and Ladner knew that Battista did not remember what color the light was.

1-2. Lawyer Long, three years out of law school, had never set foot in a courtroom. Long was on the board of directors of the Community Nursery School ("CNS"), a nonprofit preschool for underprivileged children. One of the CNS teachers was charged with felony child abuse for allegedly molesting three of the CNS pupils. After conducting its own careful investigation, the CNS board of directors concluded that the criminal charge was totally unfounded, and the board resolved to provide defense counsel for the teacher. Long volunteered to do the work without a fee. A few days before the trial was to begin, Long got cold feet and became convinced that he was incompetent to serve as the teacher's trial counsel. He asked the trial judge for permission to withdraw. After thoroughly questioning Long about his preparation for trial, the judge said: Mr. Long, I understand your anxiety, but you are perfectly competent to handle this case. Your motion to withdraw is denied; I will postpone the trial for seven days to allow you to complete your preparation. Now go do your job. Instead of doing what the judge ordered, Long advised the teacher that he would not defend her. He handed her all of the files in the case and advised her to retain another lawyer. Is Long subject to discipline?

(A) Yes, because he abandoned his client in direct violation of the trial judge's order.

4-38. Lawyer Larry happened upon an accident scene and stopped his car to see if there was anything he could do to help. Several police officers were on the scene, and Larry told one of them that he was a lawyer and asked if he could do anything to assist the accident victims. The police officer told Larry, "One of the victims is a physician and he claims that he knows that he's going to die from his injuries and he keeps moaning about wanting a will." Larry went over to talk to the physician, who lay on a stretcher. The physician, Peter, begged Larry to write a will for~ him on the spot. Larry at first demurred, explaining to Peter that he had only been sworn into the state bar two weeks before, he had never written a will for a client, and he had received a "D" in his only law school class covering the subject. Peter said, "I know the police officers will be glad to act as witnesses and I'll keep it simple; I just don't want those greedy worms in my mother's family to get their hands on my estate." After listening to five minutes more of Peter's pleading, Larry agreed to write the will for Peter. He wrote the will on the blank backside of an accident report. Peter signed the will and two police officers witnessed it. Larry told Peter, "Of course, there's no fee for this." Peter died two hours later. Was Larry's conduct proper?

(A) Yes, because he acted in a humane manner appropriate to an emergency situation.

1-3. During a brief recess in jury deliberations in a criminal case, juror Jimmerson telephoned a friend of hers, attorney Aulet, and asked for help in understanding a legal concept that was puzzling the jury--the meaning of "beyond a reasonable doubt." Aulet explained the term as best he could, given the circumstances and shortness of time. Is Aulet subject to discipline?

(A) Yes, because he communicated with a juror about a pending case.

2-9. Probate attorney Adamson was representing the executor of decedent Denman's estate. The executor removed the contents of Denman's safe deposit box and brought them to Adamson to be inventoried and appraised. The items included Denman's collection of valuable antique gold coins. Adamson put the coin collection into a heavy brown envelope and labeled it as part of the Denman estate. Because he intended to start preparing the inventory immediately after lunch, Adamson put the brown envelope and Denman's other belongings into the file drawer of his desk; he then left for lunch without locking the file drawer. Adamson's secretary saw the coins and saw what Adamson did with them. During the lunch hour, the secretary took the envelope of coins and disappeared, never to be seen again. Is Adamson subject to discipline'?

(A) Yes, because he did not put the coins in a safe place

2-10. Paralegal Platen works for the law firm of Dahlers & Sentz. Her direct supervisor is partner Dahlers, whose practice is limited to international trade law. Partner Sentz is the firm's leading trial lawyer, both in commercial and personal injury cases. On her way to work one morning, Platten saw a pedestrian run down in a crosswalk by a speeding car. Platten rendered first aid, and while she was waiting with the pedestrian for the ambulance, Plattcn said: "Here, call the number on this card and talk to attorney Seymour Sentz; he's really good, and he can help you recover money for the injury you have suffered." When she got to work, she told partner Dahlers what she had done. Dahlers admonished Platten not to hand out the firm's cards in such situations, but he did not discuss the matter with partner Sentz. Is Dahlers subject to discipline?

(A) Yes, because he failed to warn Sentz not to take the pedestrian's case.

2-39. Attorney Anderson is licensed to practice in State Red, but he is not engaged in the active practice of law. Anderson and Benson, a nonattorney partner, operate a temporary placement service lot legal secretaries in State Green, which borders State Red. Anderson is not licensed to practice law in State Green, nor does he hold himself out to be so licensed. An investigation by State Green authorities results in the discovery that Anderson and his partner have intentionally filed improper state business tax returns. Is Anderson subject to discipline in State Red?

(A) Yes, because his actions in State Green constitute fraud.

4-33. Booker is employed as an accountant for Hyrax Corporation. Several months ago, Hyrax's president, Potsdam, noticed that there were some discrepancies in the company's books, and that some funds seemed to be missing. Unbeknownst to Booker, Potsdam has been checking Booker's work after hours and he is convinced that Booker has been embezzling funds from Hyrax. Potsdam, on behalf of Hyrax, retains the services of Arnold Attorney to determine if Hyrax has a case against Booker. Arnold agrees with Potsdam that there is a strong indication that Booker has been embezzling funds. In fact, Arnold has already determined that tomorrow he will file a civil suit against Booker to recover Hyrax's money and go to the prosecutor's office to sign an embezzlement complaint against Booker. Arnold, however, tells Potsdam that the more evidence they can get against Booker, the stronger their case will be. Potsdam suggests that Arnold interview Booker before he presses charges, as Booker may make some remarks that would implicate him in the embezzlement. Arnold readily agrees to this. Potsdam tells Booker that Arnold is investigating some problems in account record keeping to make sure Hyrax's procedures comply with all applicable laws and regulations. He asks Booker to explain how his operation works. Arnold and Booker go to a private office, where Arnold interrogates Booker for approximately two hours. During the course of the interrogation, Booker becomes suspicious of the line of questioning, and asks if he is in any trouble. Arnold tells him not to worry, as the amount of money involved is so small that the action may not be worth pursuing. Is Arnold subject to discipline for questioning Booker?

(A) Yes, because of his statements urging Booker not to worry.

1-1. Judge Jackman is a full-time judge in State A. Her father lives in a retirement home in State B. Judge Jackman's father told her on the telephone that several of his friends in the retirement home had employed attorney Abbott to write wills for them, that in each will attorney Abbott had included a bequest to himself, and that each bequest was approximately .5% of the estimated total value of the person's probable estate. The friends told Judge Jackman's father that they did not really want to leave Abbott anything; they had assumed it was merely a matter of routine, a part of the lawyer's compensation for drafting the will. Attorney Abbott is admitted to practice in State B, but not in State A. Judge Jackman did not talk personally with any of her father's friends, but she believes that her father's rendition of the story is entirely accurate. Would it be proper for Judge Jackman to communicate directly with attorney Abbott about the matter, and if that does not satisfy her, to communicate with the attorney disciplinary authority in State B about the matter'?

(A) Yes, because she has received information indicating a substantial likelihood that Abbott has violated a legal ethics rule.

2-41. In an effort to prevent homosexual persons from moving to Elmville, the Elmville City Council passed a zoning ordinance that prohibits the use of any dwelling house as a residence for two or more adults of the same sex unless they are related to each other. Violation of the ordinance is a misdemeanor and carries a fine of $10,000. Carlo owns several rental houses in Elmville, and he was outraged when the City Council passed the ordinance. He sought the legal advice of attorney Ahmad. Ahmad advised him that the ordinance could probably be overturned as a violation of rights guaranteed by the state constitution, but that one would have to obtain legal standing to challenge the ordinance. Ahmad advised that one way to obtain legal standing would be for a landlord to bring an appropriate action for declaratory judgment, and another way would be to simply violate the ordinance and raise the constitutional challenge as a defense to its enforcement. After receiving this advice, Carlo promptly rented one of his houses to two gay men. Carlo was cited for violation of the ordinance. Was Ahmad's conduct proper?

(A) Yes, because violating the ordinance was one of the few ways to gain legal standing to challenge the constitutionality of the law

4-5. Attorney Arlette represented Ted Tingus in a wrongful discharge action against Apex Petroleum Corporation. Apex had fired Tingus from his job as a "service station field representative.'' Such representatives are responsible for visiting and inspecting Apex service stations to ensure that the station operators are obeying the terms of their leases; following Apex's rules about retail gasoline pricing; keeping full stocks of Apex brand tires, batteries, and accessories; not carrying competitors' tires, batteries, and accessories; and maintaining the stations in a clean and at attractive condition. Apex claimed that it fired Tingus for poor job performance, but Tingus claimed that he was fired because he refused to enforce certain Apex policies that he believed were violations of federal and state antitrust laws. In the course of preparing the case for trial, Arlette and Tingus talked in confidence at great length about how Apex expects its field representatives to enforce its allegedly anticompetitive roles against service station operators. Shortly before trial, Apex settled with Tingus for a generous sum. Tingus did not, however, return to his job at Apex, nor has he communicated with Arlette since the case ended. Now, three years later, a former Apex service station operator named Operman has hired Arlette to represent him in a federal antitrust action to collect treble damages from Apex for subjecting him to the very same anticompetitive policies that cost Tingus his job. Artette is handling Operman's case on a contingent fee basis. In preparing Operman's case for trial, Arlette was able to save many hundreds of hours of discovery and research work because of the information she learned in confidence from Tingus about the way Apex treats its service station operators. Thanks to what Tingus taught her about Apex's business practices, Arlette was ready for the Operman trial in record time--which panicked the lawyers for Apex and ultimately resulted in an extraordinarily generous settlement for Operman. Arlette, of course, took her share of the settlement under the contingent fee arrangement and thus profited personally from Tingus's information. Is Arlette subject to civil liability to Tingus for using his confidential information for her own monetary benefit?

(A) no, because Arlene's recycling of Tingus' information did not harm Tingus; she simply took advantage in her law practice of her earlier experience and accumulated knowledge.

1-10. Two years ago, Weaver obtained a divorce from her husband Hubbard in State A. The court awarded Weaver custody of the three children and ordered Hubbard to pay Weaver $3,000 per month in child support and alimony payments. The lawyer who represented Weaver in the divorce proceedings died. Hubbard failed to make the $3,000 payments for 17 months in a row. Weaver ran out of money and in desperation hired attorney Avilla to represent her in a proceeding to collect the past due payments from Hubbard. State A has no law or court rule that requires the loser to pay the winner's attorney fees in domestic relations matters. Because Weaver had no money to pay Avilla a regular fee, Avilla agreed to do the work on a contingent fee basis for 10% of whatever amount Weaver was ultimately able to recover. Avilla won an award for Weaver of the entire amount due ($51,000), and by tracking down and attaching Hubbard's secret bank account, he got the full amount paid to Weaver. He then sent Weaver a bill for his share, $5,100. Is Avilla subject to discipline?

(A)No, unless $5,100 is an unreasonably high fee for type work Avilla did.

1-17. Consolidated Insurance Corporation offers a legal services insurance policy. In return for a yearly premium, an insured person will be reimbursed by Consolidated for a specified amount of legal services during the year. The insured selects a lawyer from a list of "Authorized Providers" supplied by Consolidated. Any lawyer who agrees to follow a maximum lee schedule set by Consolidated can become an "Authorized Provider." Consolidated solicits insurance sales by in-person and live telephone contact with potential insurance buyers. Will attorney Alvarado be subject to discipline if he becomes an "Authorized Provider" and receives clients through Consolidated's insurance plan?

(A)No, unless Consolidated solicits persons whom it knows are in need of legal services in a particular matter covered by its insurance plan.

1-14Attorney Adams is defending defendant Dutcher at his trial for armed robbery of a liquor store. Dutcher tells Adams in confidence that at the time in question, he was sitting at home watching television with his aged mother, and that his mother can confirm his alibi. Adams interviews the mother, who solemnly confirms Dutcher's story. After talking with her, Adams strongly suspects that she is lying to protect Dutcher. Adams does not know for sure that Dutcher and his mother are lying, but every instinct tells him that they are. Adams has warned both of them about the dangers of perjury, but both have insisted that they want to testify to the alibi at trial. May Adams call Dutcher, or his mother, or both, as trial witnesses?

(A)Yes, as to both Dutcher and his mother.

2-25Lawyers Abner, Baker and Clark formed a law partnership; each contributed $100,000 in capital to get the firm started. Their partnership agreement provided that when a partner dies, the firm will make certain payments to the dead partner's named beneficiary. Sometime later, Clark died, leaving his daughter Clara, a doctor, as his sole beneficiary. Under the partnership agreement, the firm plans to make the following payments to Clara: $100,000, which represents Clark's share of the firm's assets, as measured by his capital contribution; $45,000, which represents Clark's share of fees that had been earned but not collected from clients at Clark's death; and a $125,000 death benefit, representing a percentage of Clark's earnings the year prior to his death and payable in monthly installments. Which of the following payments may the firm make to Clara?

(B) $270,000, which includes Clark's share of the firm's assets, Clark's share of uncollected fees, and the death benefit.

1-41. Client Corrales lives in State A and is a regular client of attorney Amundson, who is admitted to practice only in State A. When Corrales was on vacation in distant State B, she was injured in a car accident caused by a resident of State B. Corrales hired Amundson to represent her in a civil action against the State B driver. For masons of jurisdiction and venue, the case had to be filed and tried in State B. The written fee agreement between Corrales and Amundson provided that: (1) Amundson would assume full responsibility for the case as lead lawyer; (2) Corrales would pay Amundson 40% of the net recovery after deduction of litigation expenses; (3) Amundson would associate State B lawyer Linz to serve as trial counsel in State B; (4) Linz would assume responsibility only for his work as trial counsel; and (5) Amundson would pay Linz an appropriate portion of the 40% contingent fee. Corrales ultimately recovered $1.2 million. After deduction of $200,000 in litigation expenses, Corrales paid Amundson $400,000 (40% of $1 million). By measure of both quality and quantity, Linz did 10% of the work in the case, and Amundson did the other 90%. Assuming that $400,000 is a reasonable fee for all the work done, which of the following amounts would be proper for Amundson to pay Linz?

(B) $40,000 (10% of the total), because Linz did 10% of the work.

4-50. Larry Litiger has been practicing law for two years. The following statements were broadcast in a recent radio ad: I. "Larry Litiger specializes in personal injury and divorce cases. He is a certified trial specialist." II. "Larry Litiger comes from a long line of lawyers. In fact, his father and grandfather are both judges." III. "Larry has never lost a jury trial, and 99% of his clients end up receiving some form of payment." IV. "Litiger succeeds where others fail. Don't find out the hard way, call Litiger first." Assuming that all of the above statements are true and that Litiger complied with all the procedural requirements for running a radio advertisement, which of the statements are proper?

(B) I. only.

2-48. Judge Jones is a loyal member of the alumni association of Heathmoor, the women's college from which she was graduated. The 25th reunion of her graduating class is coming up next June, and she has been asked to participate in some activities designed to raise money for a gift from the class to the college scholarship fund. Which of the following activities would be proper for Judge Jones to do? I. Make a substantial personal donation to the class gift fund. II. Telephone other members of her graduating class and urge them to make a donation to the class gift fund. III. Serve on the scholarship fund committee, which devises the various fund-raising strategies. IV. Be the guest of honor at a dinner to raise funds for the class sift. V. Attend a fund-raising dinner for the class gift.

(B) I., III., and V. only

4-43. Lawyer LePage represents Prudence, the plaintiff in a sexual harassment case against defendant Dartmore Industries, Inc. Dartmore is represented by its regular corporate counsel, Clem. Prudence, who works on an assembly line, alleges that she was repeatedly harassed by the foreman on her work shift, Frank Farmer. Further, she alleges that the plant manager, Marianne Martin, was aware of Farmer's misconduct and did nothing to stop it. Prudence tells LePage that two of her co-workers on the assembly line, Will Whorley and Wendy Winston, each witnessed harassment incidents, but neither Whorley nor Winston reported the incidents to supervisory personnel. Whorley quit working for Dartmore at about the time Prudence filed her lawsuit. LePage wants to do some fact investigation before he starts discovery in the case. Which of the following best states proper conduct for LePage in interviewing the potential witnesses?

(B) LePage may freely interview Whorley and Winston, but he must obtain Clem's consent to interview Farmer and Martin.

1-43. Twelve-year-old Jimmy was badly injured when he was struck by a dump truck owned by Damassa Construction Corp. and driven by Damassa's employee Edmonds. Jimmy and his parents sued Damassa and Edmonds. The first count of their complaint alleges that Edmonds drove negligently while acting within the scope of his duties for Damassa, and that Damassa is therefore liable for Jimmy's injuries. The second count alleges that Edmonds drove negligently while on a frolic of his own, and that Edmonds is therefore liable for Jimmy's injuries. Damassa hired lawyer Lazarro to defend both Damassa and Edmonds. Lazarro conducted a careful investigation of the facts and concluded that Edmonds was in no way negligent; he was driving slowly and carefully when Jimmy suddenly ran out into traffic from between two parked cars. Lazarro further concluded that Edmonds was acting within the scope of his duties when the accident happened. Lazarro concluded that he could win the case because of the lack of negligence, and that he could effectively represent both Edmonds and Damassa. He then carefully explained the potential conflicts of interest to both of them and obtained their written consent to the joint representation. After exhaustive discovery proceedings, Lazarro remained convinced that Edmonds was not negligent, but he nonetheless explained the potential conflicts to Edmonds and Damassa a second time and again obtained their written consent to the joint representation. Three weeks before the case was scheduled for trial, counsel for the plaintiffs moved to disqualify Lazarro due to a conflict of interest between Edmonds and Damassa. Should the trial judge disqualify Lazarro?

(B) No, because Edmonds and Damassa consented to the joint representation after full disclosure.

2-30. Lawyer Li represented Weaver in a court proceeding to raise the alimony and child support payments set in the decree that divorced Weaver from her ex-husband, Hyde. Hyde stubbornly refused to get a lawyer in the marten The evening before the court hearing, Hyde telephoned Li at home and asked Li to explain the legal standard the judge would apply to Weaver's request for increased payments. Li responded: Mr. Hyde, I am not your lawyer, and I cannot give you legal advice. I think that you ought to get a lawyer in this matter, and if you need time to do that, I will ask the judge to postpone the hearing for a couple of weeks. Hyde said he didn't want a lawyer, and then asked Li whether Weaver and the children really needed more money to live on. Li responded: Mr. Hyde, I have no personal interest to serve here--I am simply trying to do what is best for you, and your ex-wile, and your kids. Now if you really want my opinion, I'd say yes, you should pay the extra money because they do need it to live on. Hyde thanked Li and hung up. Was Li's handling of the matter proper?

(B) No, because Li pretended to be disinterested and advised Hyde to pay the extra money.

4-20. Pilate, a local assistant attorney, has just finished prosecuting a case against Dudley Defendant, who was accused of committing a serious felony. Pilate believed he had a strong case, but Justinian, the judge trying the case, ruled to acquit the defendant,. Justinian is running for reelection in a contested campaign. The Judicial election will occur in two months. Immediately after Dudley's trial is over, Pilate makes the following statement to reporters assembled on the courthouse steps. Statement proper?

(B) No, because Pilate has brought the administration of justice into disrepute.

4-21. Cato Client brings an extremely complicated case to Art Attorney. This case is so complex that it will require the undivided attention of a qualified attorney for three months to do the necessary research, draw up the proper papers, and take the case trial. Cato, realizing the complexity of the case, makes the following offer to Art. Cato promises to pay Art $30,000 in advance for three months if Art agrees to take no other cases during the period of time, it being determined that Art's usual income from fees for a three-month period was $30,000. Which of the following best describes proper conduct for Art?

(B) Retain an amount that represents fair compensation for work actually performed on the case.

1-45. Lawyer Lubner is defending client Marine Supply, Inc. ii1 a civil action brought by the State Attorney General under the State Corrupt Practices Act of 1931. That statute makes it a civil offense for any person or business entity to bribe or give a kickback to a state official. The statute authorizes fines of up to $100,000 per transaction for any violation. Marine Supply has a strict corporate policy that prohibits its employees from bribing or giving kickbacks to anyone. Employees who violate the policy are subject to immediate discharge and are required to indemnify Marine Supply for any loss it suffers as a consequence of the violation. The Attorney General has noticed the depositions of dozens of Marine Supply employees, including one Dan Dowling. Prior to his recent retirement, Dowling was the sales manager of Marine Supply. Lubner met with Dowling to prepare him for his deposition. At the outset of the interview, Lubner agreed to represent Dowling without charge, and Lubner told Dowling that anything said between them would be confidential. During the interview, Lubner asked Dowling whether he had ever bribed any State A officials. Dowling responded: "Certainly. All our competitors were doing it too, and l had to do it to sell anything to the state." What course of action should Lubner pursue at this point?

(B) Withdraw from the case and keep Dowling's statement in confidence.

1-44. Client Carlin hired attorney Adler to put together a complex real estate syndicate. In connection with that work, Carlin disclosed to Adler a great deal of confidential information about Carfin's financial affairs. When the task was about half completed, Adler's wife was killed in a car accident, and his family's house burned down, all in the same week. Adler was so emotionally and physically drained that he felt he could not competently continue with the work for Carlin. Carlin refused to allow Adler to withdraw, stating: "Listen, Adler, you are the only person I trust to handle this matter. I know you are distraught, but hard work may help take your mind off the disasters that have befallen you." Adler begged Carlin to allow him to turn the files over to Adler's law partner, Pamer, an excellent real estate lawyer who was completely trustworthy and perfectly competent to handle the matter. Carlin refused to allow his files to be turned over to any other lawyer and insisted that Adler himself promptly complete the work. What should Adler do?

(B) Withdraw, turn Carlin's files over to Carlin, and offer to help Carlin find a new lawyer.

4-45. Attorney Ace placed an advertisement that ran daily in the classified section of the Pikeville News Journal, a newspaper of general circulation, widely read in the Pikeville area where Ace practiced. Besides stating Ace's office address and telephone number, and identifying Ace as a licensed attorney, the ad included the following statement: DIVORCES - LOW RATES!!! Just $100, plus costs for uncontested divorces According to bar association surveys, the "low average" fee in the Pikeville area for an uncontested divorce is $125, plus costs. Is Ace's advertising proper?

(B) Yes, because Ace's rates really are low.

4-18 Preacher is a law school graduate but is not a licensed member of the bar. After Preacher graduated from law school, he felt called to ministry, received a degree in divinity, and was formally ordained as a minister of his faith. Preacher is now the past of a local church, where Alfred Attorney is a member of the congregation. Preacher has been disturbed about the high rates of divorce and the break-down in American family life. If Alfred agrees to do this, is he subject to discipline?

(B) Yes, because Alfred is assisting in the unauthorized practice of law.

2-42. Attorney Aronson has just opened an office in a town where he does not know many people and has few contacts. Aronson is attempting to build up a clientele for his general civil and criminal law practice. Aronson has just heard on the radio that a group of several hundred townspeople have been arrested and are being held at the county jail for conducting a noisy demonstration outside the local high school to protest an impending strike by the teachers. The radio reports that the townspeople are charged with unlawful assembly, resisting arrest, and disorderly conduct. Aronson believes that the arrests of these demonstrators is a politically motivated trick, and that the demonstrators have been deprived of their freedom of expression in violation of the First Amendment. Aronson goes down to the county jail and offers his legal services, free of charge, to any of the arrested demonstrators who want legal assistance. Are Aronson's actions proper?

(B) Yes, because Aronson was offering his services free of charge

2-23. Attorney Ashe is a partner in a four-partner law firm. Client Corbett entrusted $40,000 to Ashe, instructing Ashe to hold it in safekeeping for a few days and then use it as a down payment on a piece of lakefront property... Is Ashe subject to liability to Corbett for mishandling the money?

(B) Yes, because Ashe mishandled the money and is therefore civilly liable to Corbett for breach of his fiduciary duty.

2-45. Attorney Alpha is a candidate in a contested partisan election to fill a vacancy on the district court bench. Attorney Beta has known Alpha since law school days, and the two attorneys often appear in the same court. Beta thinks that Alpha is an intelligent and competent attorney. However, Beta also believes that Alpha has always been "an arrogant hothead," and that a person with that kind of personality lacks the temperament to serve on the bench, where Alpha's tendencies toward arrogance might grow even worse. Newsmann, a reporter for The Daily News, a local general circulation newspaper, stood outside the courthouse one afternoon asking attorneys for their opinions of various candidates for the bench. When Beta was questioned, he told the reporter, "I'm supporting attorney Gamma for the district court vacancy, because I think that Gamma's opponent, Alpha, lacks the proper judicial temperament." Beta's comments were published the next day in The Daily News. Was Beta's conduct proper?

(B) Yes, because Beta honestly believes that Alpha lacks judicial temperament.

2-37. Entertainment lawyer Labrie has for many years represented country music star Spangles Truhart. One evening, Labrie and Truhart were having a quiet business dinner together at a well-known restaurant. A brutish drunk, Duke Sirosis, lurched up to their table and in a loud voice began a vulgar and defamatory tirade against Truhart. Everyone in the restaurant heard the vile names Sirosis called Truhart. While all of the defamatory comments about Truhart involved her personal life, about which Labrie had no real knowledge, he felt that they could not possibly be true. At Truhart's request, Labrie commenced a slander suit against Sirosis. In his answer to the complaint, Sirosis admitted making the allegedly slanderous statements, and as an affirmative defense, he alleged that the statements were entirely truthful. When the case comes to trial, would it be proper for Labrie to act as Truhart's trial counsel?

(B) Yes, because Labrte is not a necessary witness.

4-35. Lawyer Lydia represents defendant Dave, who is being prosecuted in a jury trial for an armed robbery and attempted murder that occurred on June 15. Dave has pleaded not guilty to the charges, but Lydia knows that Dave is the perpetrator and that the crime occurred at approximately 10 p.m. Vivian, the victim, testifies that she is certain that the crime occurred at 12 a.m. midnight. Dave has an airtight alibi for midnight. At 11:40 p.m. he was arrested on a drunk driving charge, and he was in police custody until 6 a.m. on June 16. On cross-examination, Lydia does nothing to challenge Vivian's recollection of the time of the attack. Also, as the trial unfolds, Lydia does not introduce ally evidence at her disposal that would help establish the time of the attack at 10 p.m. Lydia calls as a witness Officer Tatum, who testifies that Dave was in fact in custody at midnight on the night in question. Dave does not testify and is acquitted. Are Lydia's actions proper?

(B) Yes, because Lydia did not present false evidence.

1-23 Judge Jettelson sits on a United States Court of Appeals. He and two other Court of Appeals judges heard a diversity of citizenship case in which they were required to interpret a statute of State A concerning the marital communications privilege. Judge Jettelson's two colleagues wrote the majority opinion, in which they concluded that the statute gives only the witness-spouse the right to claim the privilege. Judge Jettelson wrote a vigorous and scholarly dissent, arguing that the statute gives both spouses the right to claim the privilege. Later, a State A senator legislator introduced a bill to amend the statute to reflect Judge Jettelson's position. The State Senate Justice Committee invited Judge Jettelson to testify about the public policy reasons for giving both spouses the right to claim the privilege. May Judge Jettelson testify?

(B) Yes, because a judge may engage in activities designed to improve the law

4-7. Hemoglobin, a licensed attorney, represents the plaintiff, Portia, in a personal injury case. Portia has authorized Hemoglobin to settle the case for $2,000. She further tells Hemoglobin that if he receives $2,000 from the defendant, he may keep $750 of it as his fee, and that he should pay Dr. Arzt, the physician who examined Portia, $250 for his examination and treatment of her injuries. Is Hemoglobin subject to discipline?

(B) Yes, because he did not promptly pay his client the money due her.

4-15. In April 2000, plaintiff PorTex, Inc, sued defendant Datasafe Corp, in United States District Court for Infringement of ProTex's copyright on a computer software program that protects computer data from being destroyed by so-called computer viruses. Pro-Tex's complaint alleges that Datasafe infringed the copyright by copying ProTex's anti-virus program in January 2000. Was Judge Juno correct in holding Littleman subject to litigation sanction?

(B) Yes, because in both June and September 2000, Littleman either knew or ought to have known that the January 2000 sources code was vital evidence.

2-17. From 1996 through 1999, attorney Arnett was a partner in the firm of Able & Aman. During that period, Amett represented client Cobb in obtaining a business loan from a bank. Cobb disclosed to Arnett a great deal of confidential information about his business and his personal assets. No other lawyer in Able & Aman gained access to that confidential information. In late 1999, Amett died. In 2000, Carlton asked attorney Able (the senior partner of Able & Amah) to represent him in a civil suit for serious personal injuries Carlton suffered when he was run over by a delivery track driven by one of Cobb's employees. Would it be proper for Able to represent Carlton?

(B) Yes, because neither Able nor any other lawyer in Able &Aman gained access to Cobb's confidential information.

2-3.Farmer Farner asked attorney Anderson to represent him in an eminent domain proceeding in which the state sought to obtain a right-of way across Farner's farm. Anderson had not handled an eminent domain case before, but she planned to make herself competent through diligent research and study. As it turned out, Anderson did not have enough time to do what she had planned, so she associated an eminent domain specialist, lawyer Ling, as her cocounsel in the case. Anderson did not consult Farner about associating Ling. Ling did about 90% of the work in the case, and Anderson did the other 10%. Together they secured a very favorable result for Farner, and Anderson sent Farner a fee bill for a reasonable amount. Farner paid the bill, and Anderson remitted 90% of the proceeds to Ling. Is Anderson subject to discipline?

(B) Yes, because she did not consult Farner about associating Ling.

4-25. Lawyer Linda placed an advertisement in The Clarion, a newspaper of general circulation published daily in the community in which Linda practiced. The ad was to run every Tuesday and Thursday for a six-month period. The ad listed Linda's office address and office phone and properly identified her as an attorney. The ad also included the statement" after 5 pm, call 932-4585, which was Linda's home number. Ad proper?

(B) Yes, because the information supplied will make it easier to contact her and make her services more accessible to more people .

1-36.Lawyer Laden regularly represents Electratec, Inc., a manufacturer of electric kitchen appliances. One morning the president of Electratec called Laden and said excitedly: Did you read in this morning's paper about the woman who got electrocuted when she opened the door of her dishwasher? The paper said the washer was three years old, and I'm pretty sure that it was one of ours. I found our quality control records from that period, and some of our washers left the plant without proper testing. Those records should have been shredded after two years, but somehow this batch was overlooked. I'm going to send them to the shredder now, unless you tell me that I can't. Must Laden advise the president to keep the records?

(B) Yes, because the records have potential evidentiary value if the company gets sued.

1-47. Casper, a Hollywood movie producer who was charged under a criminal statute for unfair trade practices, now faces a civil claim under the same statute. Casper retains attorney Adams to represent him in both suits. Adams is a nationally known defense attorney who has represented many famous people. Most recently, he defended a celebrity in a notorious murder case that held the country rapt for several weeks. Adams explains to Casper that the representation is very complex and would take a majority of his time for several months. Given Adams' s steep hourly rate, Casper's legal fees would likely be around $1 million. Casper is short on cash and makes the following proposal: If Adams will represent him in both the civil and criminal suits, Casper will produce a movie based on Adams's most famous past cases, told from the lawyer's viewpoint. Adams would have complete creative control and would be entitled to all of the movie's profits, which could be anything from $0 to $100 million dollars. Casper had his personal attorney draw up a proposal to this effect, and Casper submitted it to Adams. Assuming that Adams receives any consent necessary from his former clients who might be portrayed in the movie, is this proposed arrangement proper?

(B) Yes, but only if the ultimate amount paid to Adams is not excessive in light of the work done.

4-29. Attorney Axelson has been retained by Sludge Corporation. Axelson has done legal work for Sludge for many years and is on very friendly terms with its officers and directors. Axelson is also Chair of the State Bar Association's Committee on Corporate Law. A bill has been introduced into the state legislature that would allow corporate boards of directors to vote by telephone, thus eliminating the necessity for the directors to be physically present at meetings. The State Bar Committee on Corporate Law has been studying the bill and is about to have a meeting where a vote will be taken on whether to recommend to the legislature that the bill be passed. The committee's recommendation will probably carry a great deal of weight with the legislators, and in fact may well be determinative of whether the bill is enacted into law. Axelson meets the president and the treasurer of Sludge after work lot cocktails at the Snob Club, a private club to which Axelson and the officers of Sludge belong. Although the occasion is primarily social, the president of Sludge tells Axelson that he is very interested in the work of the State Bar Committee on Corporate Law. The president tells Axelson that he and the other officers of Sludge strongly favor the bill pending in the legislature, as telephone voting would be much more efficient for Sludge and would save all the time and trouble of gathering the directors together/hr meetings. He urges Axelson to argue in favor of the bill in the committee meeting and to vote in favor of recommending that the bill pass. Is it proper for Axelson to support the corporation bill in the State Bar Committee?

(B) Yes, if he conscientiously believes the bill is in the public interest and if he discloses to the committee that one of his corporate clients will benefit if the bill passes.

1-25. Attorney Atley was representing plaintiff Putkis at the bench trial of a civil action before Judge Joiner. Alley loaned judge money and suit was settled... Is Atley subject to criminal liability for lending money to the judge?

(B) Yes, if it is proven that, in making the loan, Atley intended to induce Judge Joiner to decide the case in favor of Putkins

4-28. Athos and Porthos are law partners. For many years their firIi1 bas represented Cardinal Industries, a local manufacturing firm. Cardinal gives tile bulk of its legal business to the firm. Such business is handled by Athos. Porthos has never done any work for Cardinal, but he knows that Cardinal is a major client of the firm. Along with other work for Cardinal, Athos is doing some collection work for Cardinal. A number of Cardinal's customers have not paid their bills, and Athos is in the process of obtaining judgments against them. These particular judgments are all default judgments, as none of the customers have filed answers to the complaints within the time limit stated. Thus, the judgments will be handled in a routine manner by the court with virtually automatic rulings in favor of Cardinal. In the meantime, Porthos has been retained by Dartagnan Products, Inc. Porthos has been asked by Dartagnan to draw up a number of contracts. Athos files the papers/'or default judgments against Cardinal's delinquent customers. Among these customers is The Aramis Company. After the papers have been filed, Athos discovers that Aramis is an unincorporated division of Dartagnan Products. Athos tells Porthos. Porthos in turn tells the appropriate officer of Dartagnan that he will have to withdraw from representing Dartagnan because of a conflict of interest. After explaining the problem fully to the Dartagnan officer, they part on very cordial terms with Porthos being given permission to withdraw. Athos proceeds with his cases against Cardinal's delinquent customers. Is Athos subject to discipline?

(B) Yes, unless his client consents after full disclosure.

1-15. The Bar of State Alpha has established an Interest on Lawyers' Trust Accounts ("IOLTA") program, whereby lawyers deposit client trust funds into special client trust accounts that pay interest to the Bar of State Alpha, which then uses the money to help fund legal services for poor people. The State Alpha IOLTA program requires lawyers to deposit a particular client's funds in an IOLTA account unless the funds would earn more than $50 in interest during the time they are entrusted to the lawyer. If the client's funds would earn more than $50 in interest during that time, the lawyer must deposit them in a separate interest-bearing trust account and pay the interest to the client. State Alpha lawyer Longfisher settled a personal injury case brought by her client Choy. The defendant sent Longfisher a check for $9,000. Because she was leaving that day for a one month vacation in Fiji, Longfisher instructed her assistant to deposit the check in Longfisher's IOLTA account. The assistant is authorized to make deposits to and withdrawals from the account. Longfisher did not tell her assistant to notify Choy that the check had arrived. When Longfisher returned a month later, she notified Choy that the check had been received, and Choy came to Longfisher's office that same day to collect the $9,000. At the prevailing rate of interest, the $9,000 would have earned $40 during the month that Longfisher was gone. Was Longfisher's handling of the matter proper?

(B)No, because she should have instructed her assistant to notify Choy promptly that the check had arrived.

2-2.Justice Jacobs was on the Supreme Court of State Beta. State Beta's Supreme Court Rules provide that in capital punishment cases, any one justice of the supreme court is empowered to grant a stay of execution pending appeal to the supreme court. Justice Jacobs granted such a stay in the case of People of State Beta v. Dillon, on the ground that Dillon had been denied the effective assistance of counsel at his trial. A few months later, Justice Jacobs retired from the supreme court and went back to private law practice. In due course, the supreme court heard the appeal in the Dillon case, rejected Dillon's effective assistance of counsel contention, and affirmed the death penalty. Acting as an indigent in propria persona, Dillon then commenced a federal habeas corpus proceeding in the United States District Court for the Eastern District of State Beta, and asked that court to appoint a private lawyer to represent him in the habeas corpus proceeding. The district court appointed Jacobs to represent Dillon. A key issue in the habeas corpus proceeding is whether Dillon was deprived of the effective assistance of counsel at his trial. May Jacobs represent Dillon without getting the consent of all parties to the habeas corpus proceeding?

(B)No, because when Jacobs was a supreme court justice he granted a stay of execution to Dillon.

1-37. Lawyer Leavitt practices environmental law. He also happens to be one of the nation's leading experts on the environmental effects of filling wetlands. The legislature of State A has scheduled hearings on a bill to prohibit the filling of wetlands surrounding Clearwater Bay. One of Leavitt's regular clients is Bay View Development Company, which owns development rights to some of the wetlands in question. Bay View wants to fill its wetlands so that it can build low-cost housing for underprivileged families. Bay View hired Leavitt to appear as a witness at the legislative hearings and to testify in opposition to the ban on wetland filling. Leavitt appeared as a witness, identified himself as an expert on wetlands, and testified vigorously against the proposed legislation. Was Leavitt s conduct proper?

(B)No, unless he informed the legislators that he was appearing in a representative capacity.

1-7. Judge Jeffery serves on a State A trial court that has nine other judges. Her husband, Horace, is a life insurance salesman for the Amalgamated Life Insurance Company. Amalgamated is occasionally a litigant in the court on which Judge Jeffery sits. Every year Amalgamated runs a national sales contest in which the person who sells the most life insurance during the year receives a valuable prize. Horace has just learned that he is the winner this year. The prize is an all-expense-paid vacation in Europe for two people. May Judge Jeffery urge Horace to accept the prize and take her on the European vacation?

(B)Yes, unless giving Horace the prize could reasonably be perceived as an attempt to influence Judge Jeffery in the performance of her judicial duties.

2-26. Client Chandler, a concerned environmentalist, hired lawyer Lipscomb to obtain preliminary and permanent injunctions against a highway construction project that would require draining and filling certain wetlands inhabited by migratory waterfowl. Lipscomb is the nation's leading expert in wetland preservation law, and he charges $400 per hour for his services. Chandler agreed to pay him at that rate. She gave him a $40,000 advance on attorney fees and a $5,000 advance to cover future litigation expenses. Lipscomb deposited the entire $45,000 in his client trust account. Lipscomb then spent 80 hours preparing and filing a complaint and preparing and arguing a motion for a preliminary injunction. He paid a court filing fee of $50, plus $1,950 in witness fees to wetlands experts who testified at the hearing on the preliminary injunction motion. The judge denied the preliminary injunction motion. Lipscomb sent Chandler a bill for $32,000 in attorney fees and $2,000 in litigation expenses, and he told her he would deduct those sums from the advances she had given him unless he heard from her to the contrary within 15 days. In light of the loss of the preliminary injunction motion, Chandler was outraged at the size of Lipscomb's fee; she immediately fired him and demanded the prompt refund of her entire $45,000. Which of the following amounts must Lipscomb promptly refund to Chandler?

(C) $11,000

2-44. Attorney Angstrom was appointed by the court to defend client Cheever at Cheever's criminal trial for second degree murder. Angstrom started interviewing potential witnesses. When she interviewed Cheever's landlord, the landlord said that on the night of the murder, Cheever came home very late and was wearing a shirt covered with blood. The landlord died before trial without speaking to state authorities. Which of the following best states what Angstrom should do with respect to the information she has learned from the landlord?

(C) Angstrom should keep the information in confidence unless Cheever authorizes her to reveal it, even though the death of the landlord has made it impossible for the prosecutor to obtain the information other than from Angstrom.

4-23. Attorney Alondra and client Cyprian agree that Alondra will represent Cyprian for a contingent fee of 25% of any eventual settlement or judgement in Cyprian's personal injury action. The case comes to trial and the judgement awarded is $20,000. The day after the trial, Cyprian calls Alondra and tells her, "I just talked to my brother-in-law's cousin; he's a file clerk for an insurance company and he says that any good at attorney would've gotten $40,000 for the kind of injury I had. It would be proper for Alondra to:

(C) I, and II, but not III

4-27. Shortly after the county grand jury handed down an indictment for armed robbery against fugitive Drooles, District Attorney Daly met with the working press outside the door to the grand jury room. Daly told the press, "you all know that I'm limited as to what I can say about pending cases, so I only have three statements to make to you. Which of the above was proper?

(C) I, and III. only

2-20. Attorney Abrahamson recently opened his solo law practice in Crystal Springs. His practice is fairly evenly divided between civil litigation and criminal defense. The Crystal Springs Superior Court has just appointed him to represent defendants Denton and Drews, who will be tried jointly for their alleged kidnapping and brutal murder of nine Crystal Springs school children. For which of the following reasons may Abrahamson decline the appointment? I. He believes that to represent Denton and Drews will take so much time away from his newly opened practice as to impose an unreasonable financial burden on him. 11. He believes that Denton coerced Drews into helping kidnap and kill the children. III. He believes many of his potential clients in Crystal Springs will be outraged if he defends Denton and Drews. IV. He believes that confidential information he received when representing one of the prosecution's key witnesses will be useful in impeaching that witness's credibility.

(C) I., II., and IV. only.

2-14. Bar applicant Lingenfelter is applying to become a member of the Bar of State A. The application questionnaire asks whether she has ever used any narcotic in violation of State A law. When she was a high school student in State A, Lingenfelter occasionally smoked marijuana, which is a minor misdemeanor under State A law. The statute of limitations has long since run on those offenses. Lingenfelter is convinced that she could not validly be kept out of the State A bar for those offenses. She therefore believes that the question is irrelevant and an invasion of her privacy in violation of Stale A's Constitution. She fears, however, that challenging the question could brand her as a troublemaker and delay her admission to the bar. Which of the following would be proper? I. Answer the question in the negative, without saying more. II. Answer the question in the affirmative, but explain the circumstances. III. Decline to answer the question, citing the invasion of privacy provision of the State A Constitution. IV. Decline to answer the question, citing the federal constitutional privilege against self- incrimination.

(C) II. and III. only

1-46. Criminal defendant DeVries exercised his Fifth Amendment privilege against self- incrimination and elected not to testify on his own behalf at his trial. Prosecutor Prichard presented compelling evidence of DeVries's guilt. In her closing argument to the jury, Prichard made the following statements: I. DeVries knows where he was on that fatal night. I have presented the testimony of three witnesses that he was with the victim. Did DeVries deny it? No! He sat there saying nothing. II. What are you to make of defense witness Fergus Grutz? You heard evidence that Grutz has twice been convicted of perjury. Could there be better proof that Grutz is a liar'? III. You may wonder why I cross-examined defense witness Emma Schlarp so vigorously. When you've been a prosecutor as long as I have, you can tell who is truthful and who is not. Was Emma Schlarp telling you the truth? I don't think so, do you? IV. Is DeVries guilty? That's what you have to decide, but I hope you will conclude that the evidence points only one way: guilt beyond a reasonable doubt. Which of Prichard's statements were proper?

(C) II. and IV. only.

1-4. Duffy graduated from law school, but he never took the bar examination and was never admitted to practice. He works as an investigator and paralegal for the law firm of Schnell & Gao, a professional corporation. Which of the following statements are true? I. On Duffy's recommendation, one of Duffy's friends retained lawyer Gao to represent her as plaintiff in a personal injury case. It would be proper for Schnell & Gao to pay Duffy 10% of its fee in the case as compensation for the referral. II. Lawyer Schnell frequently assigns Duffy to draft wills for Schnell's estate planning clients. Schnell supervises Duffy's work, revises Duffy's drafts, and is ultimately responsible for the final product. Schnell is subject to discipline for assisting a nonlawyer to engage in the unauthorized practice of law. III. Schnell & Gao established a retirement plan that is funded partly by legal fees earned by the Finn's lawyers. The firm may include Duffy as a beneficiary of the retirement plan. IV. Schnell & Gao established a stock option plan to compensate its personnel for hard work. The options allow recipients to purchase shares of stock in the law firm at a reduced price. It would be proper for Duffy to acquire stock in the firm through the stock option plan.

(C) III. only.

2-47. Attorneys Agnes and Barnaby have been law partners for six years. They share a suite of offices in a large downtown office building and use the same secretarial staff. Unfortunately, Barnaby had difficulties with the Internal Revenue Service, and was found guilty of intentionally understating his income for one tax year. Barnaby paid the taxes due, plus interest, penalties, and a $5,000 fine. He did not receive a prison sentence. The Disciplinary Committee of the State Bar Association suspended Barnaby from the practice of law for one year. Agnes took over Barnaby's clients when his suspension went into effect. Shortly before his suspension, Barnaby had negotiated a $30,000 personal injury settlement on behalf of his client, Charlotte. Two weeks after the settlement was reached, the defendant's insurer sent a $30,000 check to the law offices. By this time Barnaby's suspension had gone into effect. Agnes placed the check in the proper account and confirmed the amount of the fee with Charlotte. Agnes then promptly forwarded a $20,000 check to Charlotte and a $10,000 check to Barnaby, the latter check representing Barnaby's one third contingent fee. Is Agnes subject to discipline?

(C) No, because Barnaby earned the fee prior to his suspension.

4-41. Jilllan has just been elected a judge of the circuit court. She has been assigned to the probate division. Prior to Jillian's elevation to the bench, Jilllan was a partner in the law firm of Judkins & Jarvis. During her last week with the firm, Jilllan filed a number of very routine, uncontested probate motions. At the time, Jilllan had no idea that she would be assigned to the probate division. These routine probate motions have been assigned to her courtroom by a lottery system of random assignment that the circuit court regularly employs to assign cases. Is it proper for Judge Jillian to rule on these motions?

(C) No, because Judge Jilllan has a conflict of interest.

2-34. United Consumers Bank operates a "Professional Referral Hotline" for its depositors. Any United depositor who needs to find a physician, lawyer, accountant, dentist, or the like, can telephone the hotline and obtain a free referral from lists of professionals compiled by United. The lists are limited to professionals who maintain an average balance of at least $10,000 in a time deposit account at United, but the professional does not pay a fee to United for receiving a particular referral. Lawyer Lomax keeps $10,000 on deposit with United for the express purpose of being included on its lawyer referral list. Is this arrangement proper?

(C) No, because Lomax is required to keep $10,000 on deposit to be included on the list.

4-17. Client Celia comes to attorney Adnan seeking representation in bringing a breach of contract action. Celia tells Adnan that she originally retained lawyer Louis about a year ago, but that as far as she knew he had not even filed the papers. According to Celia, Louis never returned her calls, and when she went to his office to find her calls, and when she went to his office to find out the status of her case, he was drunk and out of status of her case, he was drunk and verbally abusive. Celia told her friends and family about Louis's treatment of her, and when her brother fared no better in getting information from him, he suggested that she contact Adnan. Were Adnan's actions proper?

(C) No, because Louis' actions indicate that he is not currently fit to practice law.

4-19. Windy was a named partner in the law firm of Blowhard, Windy & Crooke. After being approached by officials of his political party, Windy decided to run for Governor of State North. Windy won both the primary and the general election and was recently sworn in as the state's new Governor. Is Blowhard's decision proper?

(C) No, because Windy is not actually practicing with the firm.

4-8. Attorney Aoki represents client Carson, the plaintiff in a personal injury suit arising out of a tour bus accident in Hawaii. Nearly all of the eyewitnesses were tourists who have no returned home to the mainland. Without notifying the defense attorney, Aoki has interviewed most of the witnesses by phone, By far the most compelling witness, and the one most favorable to Carson, is Willa... Is Aoki subject to discipline?

(C) No, because a lawyer may pay a witness's reasonable expenses and lost wages.

2-28. Client Cristin sought the advice of lawyer Leona on a difficult and sensitive family problem. Cristin suspected that her husband had been molesting Daisy, Cristin's 12-year- old daughter by a prior marriage. Cristin asked Leona what she should do. Leona advised Cristin that all three members of the family should consult Frances, a licensed family counselor who specializes in precisely this sort of problem. Fearing that if Cristin were aware of the law she would not seek counseling, Leona purposely failed to tell Cristin that a new state statute requires family counselors to report to the district attorney all instances of suspected child abuse. Cristin and her family consulted Frances, and Frances reported the matter to the district attorney, as she was required to do by law. The district attorney commenced criminal proceedings against Cristin's husband, much against the wishes of both Cristin and Daisy. Were Leona's actions proper?

(C) No, because a lawyer should fully advise a client of relevant information.

4-9. Charlene went to the law office of Leopard and told Leopard that she wished to divorce her hard-drinking, abusive husband, Buster. However, since she was a housewife, she told Leopard that she could not afford to pay big Lawyer's fee. Contingency fee... However, personal satisfaction aside, was Leopard's conduct proper?

(C) No, because contingent fee arrangements are inappropriate in divorce cases.

4-24. Attorney Alex has been retained by Cal Carbon, a wealthy and socially prominent resident of Salt City, where Alex maintains his offices. Cal has held a number of local political offices and is presently engaged in a hotly contested primary election contest for the seat in the United States Congress for Salt City and environs. Is Alex subject to discipline for negotiating with Sarah?

(C) No, because he advised her to retain counsel.

2-50. Lawyer Leroy is interested in obtaining legal business from the United Smelter and Mining Employees Union ("USMEU"), which has many organized workers in the state. As a result of a recent mine fire and explosion in which several miners were killed, USMEU has succeeded in persuading the appropriate state agency to bring an administrative action against the mining company for failing to install smoke detectors, which might have saved some lives in the mine disaster. Although Leroy is in no way involved in the case, he sees this as an opportunity to get future business from the union by showing USMEU that he is strongly on their side in the mine disaster case. Leroy telephones a popular call-in radio show, and makes the following statement: My name is Leroy and I am an attorney. I'd just like to say that I am shocked and appalled at the callousness of the mining company that caused the recent disaster in which so many miners were killed. From what I have read, it appears to me that the mining company was willful and wanton in its failure to install smoke detectors. I am behind the union 100%; I hope the company will not be allowed to escape the consequences of this despicable conduct. Without Leroy's knowledge or consent, his statement was later printed in several newspapers in the state. Is Leroy subject to discipline for his conduct?

(C) No, because he did not make any false or misleading claims about himself or his services.

4-42. Mr. Smith and Ms. Jones wish to trade parcels of real estate. Smith presently owns Puceacre and Jones owns Limeacre. Jones retains Alan Attorney as counsel. Jones wishes to complete the deal as cheaply as possible. She asks Alan to draw up papers for the transfer of property and asks him to order a title search and survey of Puceacre. Alan recommended a similar search and survey for Limeacre as well, but Jones replied that she wished to save money and did not need a search and survey for Limeacre. Jones and Smith exchange warranty deeds. Smith takes possession of Limeacre and Jones takes possession of Puceacre. A year later Smith contracts to sell Limeacre. The prospective purchaser of Limeacre orders a title search and survey. He then discovers that there are defects that will substantially reduce the value of the property. Smith eventually sells Limeacre, but at a price much lower than he could have commanded had the defects not been present. Smith sues Jones for damages and receives an award of $10,000. Jones feels that Alan is responsible for this and sues him to recover the $10,000. Is Alan subject to liability for malpractice?

(C) No, because he was following his client's instructions.

2-22. Lawyer Loomis is the head of the in-house law department of Darlington KiddieWear Corp., which has its principal place of business in State A. Under the law of State A, it is a felony to manufacture or sell children's sleepwear that is not fire retardant. The president of Darlington informed Loomis in confidence that Darlington is stuck with a whole warehouse full of children's pajama fabric that does not meet State A's fire standards, and that to avoid financial disaster Darlington will use the fabric to make children's sleepwear and take its chances on legal liability. Loomis was unable to convince the president to change his mind; she then raised the issue with Darlington's board of directors, which ratified the president's decision. Will Loomis be subject to discipline if she resigns as house counsel and reports the matter to the appropriate State A law enforcement authorities?

(C) No, because she is entitled to reveal this type of confidential information.

2-31. Lawyer Lockwood represents defendant Downs in a drug smuggling case. Downs is in pretrial custody in a distant city and cannot be reached by telephone. One key issue in Downs's case is on the cutting edge of search and seizure law, and Lockwood believes that he needs help to deal with the issue competently. Lockwood's former law professor is a nationally known expert on search and seizure law. Lockwood calls his professor to ask lot his help, and also asks that the professor keep this information confidential. To frame the issue accurately, Lockwood tells the law professor some information that Downs revealed to Lockwood in confidence. Lockwood does not tell the professor tile name of his client. Is Lockwood subject to discipline r0r disclosing Downs's confidential information to the professor?

(C) No, because the disclosure was necessary to effectively carry out the representation.

2-32. For many years, tax attorney Aguere has handled all of the tax work for client Carrara, a famous post-modem sculptor Carrara's large sculptures cost hundreds of thousands of dollars each and are sold mostly to wealthy collectors and museums. Carrara also, however, produces the occasional small, modestly priced work. Aguero greatly admires Carrara's talent and yearns for one of the small sculptures to display in his office. One evening Carrara invited Aguere to his studio to discuss some tax returns that had to be filed the next day. In the studio, Aguere saw a small sculpture that would be perfect for his office. At the close of their tax discussion, Aguere told Carrara how much he admired the small sculpture and offered to buy it for $10,000, its approximate fair market value. Carrara told Aguere that it was not for sale. In due course, Aguere sent Cartara a $750 fee bill for the tax work. A few days later, the small sculpture was delivered to Aguero's office with the following note from Carrara: My dear Aguere: I hope this small piece of my work will satisfy your recent lee bill. I want you to have it as a token of my gratitude for the excellent tax advice you have given me all these years. I hope you will enjoy having it in your office. Your friend, Carrara. May Aguere accept the small sculpture from Carrara?

(C) No, because the gift did not require Aguere to draft an instrument.

2-11. For several years, attorney Aston worked at the United States Department of Labor as part of a small group of attorneys whose responsibilities included compiling certain corporate safety records and monitoring compliance with federal regulations. Under a federal statute, factories of a certain type and size must report all work related accidents to Aston's office. Aston's duties included compiling an annual report containing the accident statistics of all of the reporting companies. The report is used internally and in discussions with companies, but it is not distributed to the general public. A person may obtain a copy of the report, but must file a formal request under the Freedom of Information Act procedures adopted by the Labor Department. During the last three years, Chemco has bad more accidents than any of the other reporting companies. Six months ago, Aston left the Labor Department and took a job with a private law firm. Charles comes to Aston seeking representation in a suit against Chemco for injuries he sustained last month while working at one of Chemco's factories. Although unsure as to whether he should take the case, Aston, who is just starting out in private practice and cannot afford to turn clients away, agrees to represent Charles. Is Aston subject to discipline?

(C) No, because the information is available by formal request under the Freedom of Information Act.

4-37. Lawyer Lou LaCosta represented Carter Corp. in some business negotiations with Alconn Construction Inc. Only four persons were present during the negotiations: (i) Carter Corp.'s president, Cora Carter;, (ii) Carter Corp.'s business lawyer, Lou LaCosta; (iii) Alconn's chief executive officer, A1 Corm; and (iv) Alconn's attorney, Arlo Askew. During the negotiations, Cora Carter and LaCosta clearly heard A1 Corm make a certain representation that was vital to the success of the negotiation. Based on A1 Corm's representation, the parties reached an oral, handshake agreement to pursue a certain business opportunity as a joint venture. Six months later, after Carter Corp. had invested $11 million in the joint venture, Carter Corp. discovered that Alconn's representation was false and that A1 Corm undoubtedly knew it was false when he made it. Due to the false representation, the joint venture failed, and Carter Corp. lost its $11 million. Carter Corp. sued Alconn Construction Inc. and A1 Corm in federal court for intentional misrepresentation. Caner Corp. selected one of LaCosta's law partners, Liddy Gator, as its trial counsel. The defendants denied making the representation. Carter Corp.'s final pretrial statement listed Cora Carter and LaCosta as witnesses for Carter Corp., stating that they would testify that they heard Al Corm make the representation. Counsel for the defendants then moved to disqualify Liddy Gator as trial counsel due to LaCosta's role as a witness for Carter Corp. Is Liddy Gator subject to disqualification?

(C) No, because there is no conflict of interest presented by LaCosta's role as a witness for Carter Corp., and because a lawyer is ordinarily allowed to serve as trial counsel in a case where her law partner will testify on behalf of her client.

4-6. Attorney Archer is representing Davis in the civil case of Preston v. Davis, which arose out of a business deal gone sour. One evening after court was out of session, but with the trial set to resume the next day, Archer attended a $5,000 per person charity fund-raising dinner. Talked to Preston not about case. Is Archer subject to discipline?

(C) No, because they discussed the charity, the weather and sports

4-36. Tom Tuttle is; the trustee of a trust for the care and support of his deceased sister's minor children. Tuttle wishes to sell some of the trust property to pay for the schooling of one of the children, who has special needs. Tuttle hires lawyer Lemke to file the appropriate papers to get court approval for the sale. In the course of the conversations between Lemke and Tuttle, Tuttle discloses that he has committed several breaches of trust in the past, including borrowing trust funds to pay for his home improvements and gambling trust funds at the race track. Most of the money has been repaid with reason able interest, and Tuttle tells Lemke that he has learned a few things about being a trustee, and will be much more careful about his handling of trust funds in the future. Lemke urges Tuttle to tell the court of his wrongdoing and resign as trustee, but Tuttle refuses. Lemke proceeds to represent Tuttle in the proceeding seeking court approval for the sale of trust assets. Lemke fills out all of the court papers truthfully and does not in any way slate anything false or misleading lo the court. The court does not inquire about the management of the trust or any dissipation of trust assets, and neither Lemke nor Tuttle volunteer the information. Is Lemke subject to discipline?

(C) No, because tile information was confidential.

2-5. Solo practitioners Arias and Armer share office space. Each of them has organized her practice as a professional corporation. The sign on their office door reads: Arlene Arias, PC. Personal Injury Law/Alice Armer, PC General Practice. Arias and Armer frequently consult with each other about their respective cases, and they often refer clients to one another. Sometimes they work on cases together under a fee-sharing arrangement. When one of them is out of the office, the other responds to client inquiries to the extent that she is able; to facilitate that practice, each attorney has physical access to the other's client files. Plaintiff Puente hired Arias to sue McDougal's Bakery for personal injuries he sustained when he bit into a piece of' glass in a dinner roll baked by McDougal's. McDougal's liability insurance carrier, American Assurance Associates, asked Armer to serve as defense counsel in the case. May Armer take the case ?

(C) No, even if Arias and Armer believe that they can effectively represent their respective clients, and even if Puente, McDougal's, and American consent after lull disclosure.

2-8. Lawyer Lederlee was assigned by the court to defend an indigent person, former college English teacher Deniew, at her trial for the murder of her husband. The jury convicted Deniew, and she was sentenced to 40 years in prison. Lederlee's court appointment expired at the end of the trial, but he promised Deniew that he would represent her without cost in taking an appeal from her conviction. Lederlee advanced $350 on Deniew's behalf to cover the expenses of the appeal, knowing that Deniew would probably not be able to pay him back. While the appeal was pending, Deniew wrote the manuscript for a book about life in a women's prison. She hired Lederlee to negotiate a contract with a publisher to have the book published, and in return for the contract work, she promised to pay Lederlee 30% of the royalties from her book. Is Lederlee subject to discipline?

(C) No, unless 30% of the book royalties is unreasonably high for the contract negotiation work.

4-22. Judge Jonathan, considered a great personal injury litigator when he was in private practice, is trying a very complicated commercial law case. He has carefully listened to the opposing attorney's arguments and has read the briefs several times. He has found neither the oral arguments nor the briefs to be very enlightening. Sheba, a former law partner of Judge Jonathan, is considered to be one of the leading experts on commercial law in the state. Is it proper for Judge to seek such help from Sheba?

(C) No, unless he gives notice to the parties and allows them time to respond to Sheba's memorandum.

4-49. Uncle, a wealthy landowner in Moocow County, has become very infirm in his old age, even though his mind is still very sharp. His doctor tells Uncle that although Uncle has survived open-heart surgery, he will never be really robust and healthy again. In fact, the doctor says that Uncle should either enter a nursing home or employ a nurse to care for him full-time at his ranch. Uncle, who has always prided himself on his independence, shudders at the thought of being put in a nursing home. He also finds the thought of bringing a stranger into his home to nurse him distasteful. Uncle contacts Niece, his youngest sister's daughter, who is a registered nurse employed as a head nurse at a major research hospital in a large city in another state. Uncle urges Niece to come and live with him at the ranch and take care of him. Uncle, always known in Moocow County for his generosity, tells Niece that he understands she would be making a major sacrifice and that "I aim to make this worth your while. The Doc tells me I don't have more than a year or two left anyway, but I know it's tough for a young, educated, city gal to be stuck out here in Moocow County with nottin' but old men, steers, and no-account cowboys. So if you stay and take care of me until I die, I'll leave you one-third of my ranch in my will." Niece agrees to care for Uncle. Niece arrives in Obelisk, the county seat of Moocow County where Uncle is hospitalized. She is met at the train by Cowhand, one of Uncle's employees, who takes Niece to the hospital. Uncle is then released from the hospital, but before going home he stops by the law offices of Archon, his attorney, to pick up some tax documents Archon has completed for him. Uncle tells Archon, in the presence of both Niece and Cowhand, "The next time I come to town, I'm going to have you draw up a new will for me, leaving one-third of my ranch to Niece, who's come all the way out here to Obelisk to take care of her poor old uncle." Uncle never has another opportunity to go into Obelisk. Niece feels he is too weak to travel very much after his operation, and insists he stay home where she can care for him. Two months later, Uncle has a massive heart attack, followed by a stroke. He goes into a lingering coma and dies after two weeks in the comatose state. Uncle's will is duly admitted to probate, and under its terms Uncle left everything he had to Son, Uncle's only child, who is a highly successful businessman in another state. Son arrives in Obelisk to attend Uncle's funeral and to settle the affairs of his father's estate. Naturally, he consults with Archon. Son mentions to Archon that he thinks it was very good of Niece to come all the way to Obelisk to care for Uncle, and that he is sure that Uncle would have wanted to reward her in some special way, even though there is no mention of Niece in the will. Son tells Archon that he would like to do the "right thing," but has no idea what Uncle might have had in mind. What should Archon do?

(C) Tell Son about Uncle's statement about leaving one-third of the ranch to Niece, but offer no further advice.

2-12. Attorney Alexander and her client Cardone endured a stormy attorney-client relationship until Alexander finally withdrew due to Cardone's repeated refusals to pay Alexander's fee bills. At the end of the relationship, Cardone owed Alexander more than $10,000. Cardone said he would not pay because Alexander's legal services were "defective." In a final effort to avoid having to sue Cardone for the unpaid fees, Alexander proposed a settlement agreement to Cardone. Under the proposed agreement, Alexander would accept $4,000 as full payment, reserving the right to sue Cardone for the other $6,000 if Cardone filed a State Bar disciplinary complaint against Alexander or filed a legal malpractice action against Alexander. Cardone signed the settlement agreement without consulting outside counsel, and Alexander did not suggest that he should consult outside counsel before signing it. Is Alexander subject to discipline for entering into the settlement agreement with Cardone?

(C) Yes, because Alexander did not advise Cardone to seek outside counsel before entering into the settlement agreement.

2-4. Attorney Aoki is defending client Childs in a civil fraud case in which it is relevant to know what advice Childs received in confidence from an independent certified public accountant, Ben Counter. This jurisdiction has no evidentiary privilege for confidential communications between accountants and their clients. Counter telephoned Aoki and asked how he should respond to the plaintiff's lawyer's request to speak with him privately about the case. Reasonably believing that Counter would not be harmed by refusing to talk informally with the plaintiff's lawyer, Aoki responded: "If the plaintiff's lawyer subpoenas you to testify, then you must do so, but I encourage you not to talk to him about the case unless you are under subpoena." Was her advice to Counter proper?

(C) Yes, because Counter acted as Childs's agent in rendering accounting advice to Childs.

2-21 Federal prosecutor Feldman is stationed in Maryland; he is gathering evidence to support federal racketeering charges against a swindler named Phatseaux... Is Feldman subject to criminal liability because of his evidence-gathering technique?

(C) Yes, because Feldman made an interstate threat to accuse Bucks of a crime for the purpose of extracting valuable information that he could use against Phatseaux.

1-33. Lawyer London and her nonlawyer friend Ferguson created a partnership to serve people who want to invest in commercial real estate. Ferguson, a licensed real estate developer, finds promising commercial real estate projects, brings together groups of investors, and works with local planning authorities to gain approval for the projects. London drafts the legal documents for the projects, assists the investors with the legal technicalities, advises the investors on their tax liabilities, and does whatever legal work the investors need in connection with management and operation of the projects. London and Ferguson charge the investors a single fee for their work, and they divide the partnership profits 50%-50%. Is London subject to discipline?

(C) Yes, because Ferguson and London are partners in the business.

1-39.Law professor Pompman was selected as the neutral arbitrator of a boundary line dispute between land owners Owens and Osborne. Pompman decided the matter in favor of Owens. Shortly thereafter, Pompman quit his teaching position and entered private law practice. Osborne brought suit to have the arbitration award set aside. Owens asked Pompman to represent him in the suit. If Pompman takes the case, will he be subject to discipline?

(C) Yes, because his earlier service as neutral arbitrator creates a conflict of interest.

2-7. A series of brutal daylight muggings in downtown Sedatia brought fear to the citizens of that normally placid city. The police captured one Diablo, who was charged with the muggings and, in due course, was ordered to stand trial in Sedatia. Two days before the jury selection began, a local newspaper reporter cornered the prosecutor, District Attorney Axelrod, in the Sedatia Cafe and got her to make the following statement: I'm certain Diablo is the right man; among other things, we have discovered that he was previously convicted three times for brutal muggings in other states. Is Axelrod subject to discipline for making the statement to the reporter?

(C) Yes, because she should have known that the statement would be quite likely to prejudice the trial.

1-30.The Bar of State A has established a peer counseling program whereby lawyers who are addicted to alcohol or other drugs can receive confidential counseling from other lawyers. The Bar of State A's ethics rule on confidential information provides that communications between the counselor lawyer and the counseled lawyer are to be treated just like confidential communications between attorney and client. Lawyer Loontis is addicted to alcohol and is receiving peer counseling under the program from lawyer Lin. Loomis is a large, strong man, and his addiction has made him subject to periodic fits of physical violence. This afternoon, during their peer counseling session, Loontis told Lin: "My client Crothers has refused to pay the fees he owes me; the next time I get drunk enough, I'm going to smash the little creep's face in." From working with Loomis over an extended period, Lin believes that he may really do it. May Lin disclose Loomis's statement to Crothers, the police, and the Bar of State A?

(C) Yes, even if Loomis objects.

1-32. Swimming coach Dershowitz was charged with sexually assaulting a young male student. Dershowitz hired criminal attorney Bailey to conduct his defense... Will Bailey be subject to civil liability in a legal malpractice action brought by Dershowitz for having missed the double jeopardy issue?

(C) Yes, provided that Dershowitz proves by a preponderance of evidence that he did not commit the sexual assault on the student.

4-39. Agnes and Bertram represent two corporations who oppose each other in a civil suit. Agnes has filed a petition with the court, seeking to have Bertram removed as opposing counsel. She claims he has a conflict of interest because he once did certain work for her present client. Bertram calls Agnes and tells her that he thinks the conflict of interest matter can be resolved if she listens to his explanation. Agnes and Bertram meet and begin to negotiate the conflict of interest issue, and during the same meeting discuss a settlement of the lawsuit. Agnes and Bertram agree to meet again and continue their discussions. Immediately after the first meeting, Agnes goes to her office and dictates a letter to Judge Jowl)', who is trying the case. The letter tells Judge Jowly that Agnes and Bertram are attempting to resolve their differences on the conflict of interest matter and are also negotiating a settlement of the underlying lawsuit. Agnes mails the letter to Judge Jowly that afternoon. Is Agnes subject to discipline?

(C) Yes, unless Agnes sends Bertram a copy of the letter.

2-16. Continuously since 1910, the law firm of Hardwicke, and Chandler has practiced under that name in State A. The founders of the firm, Horace Hardwicke and Carlisle Chandler, are long dead. No partner named Chandler now practices with the firm. Horace Hardwicke IV (the great-grandson of the founder) is presently the managing partner of the firm. Five years ago, Hanna Hardwicke, the daughter of Horace Hardwicke IV, became a partner in the firm. Hanna recently left law practice to take a life tenure appointment on the State A Supreme Court. May the firm continue to use the name Hardwicke & Chandler?

(C) Yes, unless the firm name would be misleading.

1-16. After lawyer Laben graduated from law school, she joined the congressional staff of U.S. Senator Senders. In that role, she personally drafted a bill that was ultimately enacted as the Educational Rights of Disabled Americans Act ("ERDAA"), a far-reaching statute that required colleges and universities to make many changes in the facilities they supply to blind, deaf, and physically-impaired students. Shortly thereafter, Laben moved to State A, where she became an associate in the private law firm of Dillard & Domish. Seeking legal advice on how to comply with ERDAA, the University of State A hired partner Dillard and specifically asked Dillard to assign Laben to assist him in doing the work. In light of Laben' s earlier role as the drafter of ERDAA, which of the two lawyers may work on the matter?

(C)Both Dillard and Laben.

1-27. Solo practitioner Proctor is one of only three lawyers in the small town of Sandy Gulch. Proctor is presently defending client Cridley in a criminal action for assault and battery. This morning one of Proctor's regular clients, the Sandy Gulch Gas & Grocery, asked Proctor to sue Cridley to recover $638.64 that is past due on Cridley's gasoline and grocery charge account. Would it be proper for Proctor to represent the Sandy Gulch Gas & Grocery in the charge account case?

(C)No, unless both Cridley and the Sandy Gulch Gas & Grocery consent after full disclosure.

1-22. Attorney Adams is a voting member of the legislation committee of Citizens for Safer Food ("CSF"), a consumer-based law reform group that drafts and advocates the passage of proposed statutes on food safety. CSF is currently debating a draft statute that sets quality and safety standards for growth hormones administered to chickens, turkeys, and other poultry. Attorney Adams is also engaged in the private practice of patent law. She regularly represents Genetico, Inc., a biotechnology firm. Using the techniques of genetic engineering, Genetico invents, develops, and sells a variety of patented growth hormones. Adams herself has obtained patents on some of these hormones for Generico. If enacted into law, CSF's proposed statute on poultry hormones could substantially affect Genetico's hormone sales. Would it be proper for Adams, as a member of the CSF legislation committee, to participate in the debate on, and to cast her vote on, the proposed statute?

(C)Yes, provided that she informs the legislation committee that she represents an unnamed client whose interests could be materially benefitted by the statute.

1-6. Client Chason hired lawyer Lucero to do the legal work in connection with a complex public securities offering. Lucero agreed to do the work for $160 per hour. Lucero did a great deal of legal research, prepared numerous memoranda of fact and law, and drafted most of the documents needed for the public offering. At that point, Chason became angry with Lucero for no apparent mason and fired him. Chason paid Lucero at the agreed rate for the work Lucero had done, and Chason demanded that Lucero turn over to him the papers that Lucero had prepared, including the legal and fact memoranda and the document drafts. What papers must Lucero turn over to Chason?

(D) All of the papers, even though Chason fired Lucero.

2-15. Judge Jamagian sits on a State A trial court. Every six years, State A trial judges must stand as candidates in a public election to determine whether they will retain their positions. Judge Jamagian will be a retention candidate in the election to be held nine months from now. In that same election, her husband, attorney Ali, will be a candidate for Lieutenant Governor of State A. Which of the following may Judge Jamagian do? I. Establish a campaign committee that will immediately begin soliciting reasonable contributions for Judge Jamagian's campaign. II. Allow her name to be listed on Republican Party election materials, along with the name of her husband and other Republican candidates for elective offices. III. Publicly endorse her husband as a candidate for Lieutenant Governor. IV. Attend political gatherings in the company of her husband, and speak on behalf of both herself and him. V. Personally solicit contributions to her own campaign. VI. Personally solicit contributions to her husband's campaign.

(D) I and II only

4-26. Alexandra is a recent law school graduate who has just been admitted to the State Bar. She returns to her hometown, Sodville, a town of $20,000 in population in the cent of the state, Gastrix, who has practiced out of one-person office in Sodville for many years, asks Alexandra to associate with him. He produces an employment contract that he asks Alexandra to sign. For agreeing to which provision would Alexandra be subject to discipline?

(D) I, II, and III

1-29. Lawyer Lars is defending Castco, Inc. in a suit brought in federal district court in the Second Circuit. One of the issues in the case is whether Castco violated a workplace rule promulgated by the Federal Employment Commission ("FEC"). Castco denies doing the act that allegedly violates the FEC rule. As a fall-back position, Castco argues that even if it did the act, the rule should be interpreted to exclude acts of that kind. For which of the following actions is Lars subject to discipline? I. Failing to turn over incriminating documents that Castco gave him in confidence upon his employment, and which were requested during discovery. II. Failing to cite a case directly on point decided last month by the Ninth Circuit Court of Appeals. III. Failing to notify the opposing side of a witness who can testify that the president of Castco specifically instructed one of her deputies to commit the act in question. IV. Failing to cite a three-week-old FEC decision that Lars found in a computer search and that interprets the FEC rule to include precisely the kind of act Castco allegedly committed.

(D) I. and IV, but not II. and III.

4-46. Attorney Augusta decided to run against incumbent Judge Isadore in the forthcoming election. Judge Isadore was widely regarded by members of the local bar as a "party hack," who had no business being on the bench. The opposition party was very pleased to be able to slate Augusta because she had a high reputation for intelligence, honesty, and overall competence as an attorney. Augusta realizes that she will have to fight an uphill battle to unseat Judge Isadore because her political party is a minority party in the county and most voters know very little about judges and candidates for the judiciary and therefore, voters are likely to vote a straight ticket for judges of their own political party. Augusta wants the public to know that Isadore has been a poor judge, but she also wishes to comply with all ethical rules governing judicial campaigns. Attorney Eve, Augusta's best friend and chief advisor, suggests that Augusta should make the following statements during her campaign: I. "Judge Isadore has had the highest percentage of cases reversed on appeal of any judge in the state over the past two years." II. "Eighteen months ago Judge Isadore was publicly disciplined by the State Judicial Conduct Board." III. "A recent poll taken by the local bar association indicates that a majority of bar association members feel that Judge Isadore lacks the proper judicial temperament." IV. "A recent newspaper article comparing judges of the county states that Judge Isadore has handed out an average sentence of only two and a half years to persons convicted of serious felonies. I won't be soft on crime!" Assume that all the facts cited in the numbered statements are accurate. Which, if any, of the numbered statements would it be proper for Augusta to make in her judicial campaign?

(D) I., II., and III., but not IV.

1-21. For many years lawyer Lacy has done business transactions work for wealthy client Chung. Chung was recently injured in an automobile crash, and she has asked Lacy to represent her as plaintiff in an action against the driver who injured her. Lacy has taken some business cases to trial, but has never handled a personal injury case. Lacy would like to earn a profit from Chung's case. Which of the following would be proper ways for him to do so? I. Take the case and, with Chung's consent, associate a co-counsel who is competent in the field of personal injury law. II. Refer Chung to a competent personal injury lawyer and charge that lawyer a $1,000 forwarding fee. III. Take the case and do the study and re-search needed to handle it competently. IV. Refer Chung to a competent personal injury lawyer and charge Chung a reasonable sum for the time spent in making the referral.

(D) I., III., and IV. only.

5-12. Attorney Quarrels is embittered as he has recently had to expand a great deal of time and money defending himself against a frivolous malpractice suit brought by a disgruntled former client. To forestall such suits in the future, Quarles decides to take extra precautions. As a result, he enters into the following arrangements with clients. For which if any, subject to discipline?

(D) II and III, but not I.

2-40. Client Carson, a self-employed furnace repairman with no assets, was run over in a pedestrian crosswalk by a moving van driven by an employee of Deluxe Transport and Storage Corporation. Carson's injuries were so severe that he could not work, or even seek work, for a period of 18 months after the accident. Carson asked lawyer Lucas to represent him in a personal injury action against Deluxe. At Carson's request, Lucas agreed to represent Carson on a contingent fee basis. Carson requested that Lucas provide him with certain financial assistance during the pendency of the lawsuit. Which of the following may Lucas do? I. Lend Carson $5,000, pursuant to an agreement reviewed by independent counsel, to be used to support Carson's family during the pendency of the suit. II. Lend Carson $10,500, pursuant to an agreement reviewed by independent counsel, to pay for Carson's medical treatment. III. Advance Carson $1250 for court- filing~ fees and deposition-reporter fees, subject to repayment by Carson when the case is concluded. IV. Advance Carson $2,000 to pay the expert witness fee of Dr. Stromberg, a medical expert consulted solely for testimony and not for treatment. Carson promises to repay the money when the case is concluded.

(D) III. and IV. only

1-12. For the past five years, attorney Aries has represented art dealer Corot in the sale of many valuable paintings. One of the major transactions occurred three years ago, when the American Museum of Art paid Corot $23 million for a Post-Impressionist landscape Answer purportedly painted by Vincent Van Gogh in 1890. The American Museum of Art subsequently resold the painting to the Amsterdam Fine Arts Museum for $35 million. Today, Corot asked Aries to do the legal work in connection with the sale of a smaller, less valuable landscape, also a purported Van Gogh. The proposed purchase price is $12 million, and the prospective purchaser is Lavita Lavish, a wealthy television personality who knows nothing about art. During a confidential conversation in Arles's office, Arles said to Corot: "1 assume you have appraisal letters certifying the painting as a genuine Van Gogh?" Corot replied: "Of course I have letters! I forged them myself', just as I did for that bogus Van Gogh you helped me sell to the American Museum of Art three years ago!" When Aries inquired further, Corot told him in confidence that both of the purported Van Gogh paintings were in fact counterfeits created by a clever art student. Which of the following must Arles do at this point'? I. Report Corot to the law enforcement authorities. II. Warn Lavita Lavish about the proposed sale. III. Inform the American Museum of Art of the truth about the first painting. IV. Refuse to represent Corot in the present transaction.

(D) IV only

2-18. Young associate Aster was assisting senior partner Parker in writing the reply brief in an appeal for one of Parker's clients. In doing the legal research, Aster discovered a recent case from the controlling jurisdiction that had not been cited in the adversary's brief. In Aster's opinion, the case was directly opposed to the position of Parker's client. Aster asked Parker about citing it in the reply brief, but Parker explained that, in his view, the case was not directly on point and did not have to be cited. Aster and Parker argued back and forth at some length and finally decided to submit the question to one of the other senior partners in the firm for a fresh view. That partner sided with Parker, and the reply brief was filed without mentioning the case. Should Aster write a short letter to the appellate court and the adversary lawyer, explaining his position and enclosing a copy of the case?

(D) No, because Aster should abide by Parker's resolution of the matter.

1-35. Carla alleges that she was assaulted by Devlin, a very wealthy businessman. Carla contacted lawyer Lazar about representing her in a civil action against Devlin. After several lengthy discussions about the merits of the case, Carla decided to employ attorney Arnold to represent her instead. Devlin was later charged with criminal assault in connection with this incident, and his trial was televised. Lazar watched the trial and was astonished when Carla testified to facts that Lazar knew from their previous discussions to be false. Lazar sent a letter with a messenger over to the court to notify the court that Carla had perjured herself. Were Lazar's actions proper?

(D) No, because Lazar's information was gained during his discussions with Carla

1-24.Attorney Anthony Altamirez has organized his law practice as a professional corporation. Altamirez is the sole shareholder. The sign on the office door states: Anthony Ahamirez, P.C.-- Attorney at Law/Corporate and Business law Torts and Domestic Relations...Altamirez has only one lawyer-employee, Leola Lipkis, who was admitted to practice two years ago. Altamirez pays Lipkis a modest monthly salary plus 60% of the fees collected in cases that Lipkis handles by herself. Altamirez has a general business practice, and when a client needs representation in a tort or domestic relations matter, Altamirez turns the case over to Lipkis. When Altamirez turns a case over to Lipkis, he provides general guidance and is available to answer any questions she may have, but he does not supervise every step she takes. Is Altamirez subject to discipline?

(D) No, because Lipkis is a lawyer-employee of Altamirez.

1-11. Lawyer Lohman (age 34) regularly represented client Cruikshank (age 78) in matters relating to the investment of Cruikshank's considerable wealth. Cruikshank told Lohman that he wanted to put $500,000 into a sound, income-producing investment. Lohman suggested that the two of them pool their money and talent and buy an attractive new apartment house. Lohman would put up $75,000 and do the legal work, and Cruikshank would put up $500,000 and serve as the live-in manager of the apartment house. Cruikshank enthusiastically agreed to the arrangement and told Lohman to draw up the papers. Lohman drafted an agreement between himself and Cruikshank, negotiated the purchase of the apartment house, and drafted a deed from the seller to himself and Cruikshank as joint tenants with right of survivorship. Lohman gave Cruikshank a carefully written explanation of the terms of the transaction, but he forgot to explain the significance of the joint tenancy, i.e., that upon the death of one joint tenant, the property would pass automatically to the other joint tenant. Lohman urged Cruikshank to have an outside lawyer look over the transaction, but Cruikshank said he trusted Lohman and signed all the papers without further ado. Lohman and Cruikshank operated the apartment house successfully for several years, until Cruikshank died at age 83. The executor of Cruikshank's estate sued Lohman to have the apartment house declared part of Cruikshank's estate, but the court concluded that the joint tenancy created a gift to Lohman, effective on Cruikshank's death. Were Lohman' s actions proper?

(D) No, because Lohman drafted the deed that bestowed a substantial gift on himself.

2-33. Public defender Purdum was assigned to represent defendant Dewitt at Dewitt's preliminary hearing on a charge of kidnapping for ransom. Against Purdum's advice, Dewitt testified on his own behalf at the preliminary hearing. Dewitt was bound over for trial. At that point, Dewitt's elder brother provided money to hire a private lawyer to defend Dewitt, and public defender Purdum was discharged. Dewitt testified on his own behalf at the trial, and the jury acquitted him. Later, in connection with his work on another matter, Purdum read the transcript of Dewitt's trial. Based on information Purdum learned while representing Dewitt, Purdum concluded that Dewitt had committed perjury, both at the preliminary hearing and at the trial. May Purdum reveal Dewitt's perjury?

(D) No, because disclosure would violate Purdum's duty of confidentiality.

4-47. Judge Jacques is often referred to behind his back as "Judge Continuance" by lawyers who practice in Bayou County. This soubriquet is well-deserved because Judge Jacques is known to grant continuances whenever requested by an attorney, regardless of the substantiality of the attorney's grounds. He has turned down a continuance request on occasion, but such occasions are so few and far between that local attorneys are shocked when they hear of them. When queried by his colleague Judge Jeanne about his policy on continuances, Judge Jacques told her, "This society has gotten to be litigation crazy. It's sue, sue, sue over every little thing. Most of this stuff can be settled between the parties if they really want to try. If I grant a continuance, it gives the parties that much more time to come to their senses and settle. If they settle, it saves them and the taxpayers the expense of a full-blown trial. This country and its court systems would be a lot saner if more judges tried to promote settlements like I do." Is Judge Jacques's policy of granting continuances to promote settlements proper?

(D) No, because judges have a duty to expedite litigation.

1-26.Universal Steel, Inc., merged with Delta Iron Corp. The attorney general of State A sued Universal and Delta in federal court to enjoin the merger, alleging that it was in violation of the federal antitrust laws. The federal district judge enjoined the merger, and Universal appealed his decision to the United States Court of Appeals for the Fourth Circuit. Universal's lawyer on the appeal is attorney Alvarez. In doing the legal research for the appeal, Alvarez found a recent merger decision rendered by the Federal Trade Commission ("FTC") that is directly adverse to Universal's position. FTC decisions do not control the United States Courts of Appeal, but they are persuasive. The attorney general for State A failed to cite the FTC decision. Must attorney Alvarez disclose it to the court?

(D) No, because the Court of Appeals is not obliged to follow the FTC ruling.

2-46. Stork, Inc. is a large private adoption agency that handles over 65% of all private adoptions in State Blue. Stork provides each set of adoptive parents with a list entitled "Approved List of Lawyers Handling Private Adoptions." Stork's representatives tell prospective adoptive parents that it is in their best interest to obtain counsel who has experience in adoptions. Stork has checked out every lawyer on its list to make sure that the lawyer had experience in private adoptions as well as a reputation for honesty and ethical behavior. Because of Stork's volume of business, State Blue attorneys recognize the advantages of being placed on the "Approved List." After an attorney desiring placement on the list has been screened by Stork, the attorney is required by Stork to sign a form agreement before the lawyer's name is placed on the list. To assure the adoptive parents that any lawyer on the list would follow through with the adoption to its conclusion, the required form agreement contains the statement, "1 agree that under no circumstances will 1 withdraw from any case where I have been retained by parents adopting through Stork." Attorney Ash has handled many private adoptions and is highly regarded as an honest and competent attorney. He would like to be placed on Stork's list. Would it be proper for Ash to have himself included in the list?

(D) No, because the form agreement allows a third party to exercise influence over the lawyer-client relationship.

2-43. Judge Jardon is a full-time trial judge in State A. State A has a statute that prohibits employment discrimination against gays and lesbians. In addition to her judicial work, Judge Jardon is the chief executive officer of a corporation that is closely held by Judge Jardon and her three brothers. The corporation owns and operates a nursing home in State A. Because of strong anti-homosexual religious beliefs on the part of residents, the nursing home does not employ gays and lesbians. Is it proper for Judge Jardon to continue as chief executive officer of the corporation?

(D) No, because the nursing home practices employment discrimination against gays and lesbians.

1-48. Patent attorney Amari focuses her practice on patents that involve genetically engineered medicines. Representatives of Biogenco had a preliminary conversation with Amari about representing Biogeny in a patent infringement action against Turner-Kline Pharmaceuticals, Inc. Amari had never represented either company previously. Biogenco's representatives talked to Amari for more than an hour about Biogenco's patent and about Turner-Klein's supposedly infringing product. This conversation covered... Is Amari subject to disqualification?

(D) No, because the prior conversation between Amari and Biogenco's representatives did not involve confidential information.

4-10. Belltrix, a local attorney, has recently defended David Defendant in a civil action tried before a jury. Defendant lost the case, and Paul Plaintiff was awarded a substantial amount of damages. Bellatrix receives an anonymous, handwritten letter that states: "Joan Juror, who sat on the jury when Paul Plaintiff got that big damages award, was bribed to influence other members of the jury to side with Paul and to award a large sums in damages" Is proper for Bellatrix to hire the private investigator?

(D) No, if the investigation is to be conducted in such a manner as to harass the jurors.

2-35. Inventor Inovacio asked patent lawyer Patton to represent him in obtaining a U.S. patent on a new computer technique for predicting the growth patterns of tumors in the human body. Patton informed Inovacio that he had never worked on that kind of patent application before, and that he would have to do extensive background research on the patentability of computer techniques. Patton will be able to use the knowledge that he gains through the research to serve other clients who wish to obtain patents for all manner of other computer techniques. Patton offered to do the work for Inovacio for his standard hourly rate, but Inovacio proposed instead to assign Patton a 10% interest in the patent, if and when it was issued. Patton agreed to do the work on that basis, and he and Inovacio entered into an appropriate written fee agreement. Patton did the work; the patent was ultimately issued and proved so valuable that Patton was able to sell his 10% interest for $9.7 million. Is Patton subject to discipline?

(D) No, unless $9.7 million is an unreasonably high fee for the work that Patton did

4-32. Claude walked into the law offices of lawyer Leda. As Leda was not busy at the time, she agreed to talk to Claude right away. Claude told Leda that he had "a criminal problem," and was concerned that he might be indicted soon. He explained the details of his predicament at length to Leda, but after he finished, Leda told Claude, "You certainly do need a lawyer, but l only handle civil matters. I suggest you consult with my friend Swann; he has a large criminal practice and he's pretty good." Claude went on to retain Swann. A few days after her interview with Claude, Leda read a news item announcing Claude's indictment. It quoted the district attorney at some length. After reading the article, Leda became convinced that something Claude had told her during their interview would probably exonerate Claude or, at the very least, lead to a reduction in the charges against him, if the district attorney became aware of the information in Leda's possession. May Leda reveal the information to the district attorney?

(D) No, unless Claude consents to the disclosure.

4-11. Castor and Pollux are law partners. Castor represents Scurvy, and Pollux represents Smythe. Both are criminal defendants whose cases are seemingly unrelated. During the course of an interview with Castor, Scurvy tells him that he was involved in the crime with which Smythe is charged and that he is willing to testify against. Which of the following courses of action is proper for Castor?

(D) Not inform Pollux and withdraw from representation.

1-40. Paul Poller brought a civil action to recover damages for personal injuries he suffered as the victim of alleged police brutality inflicted by defendant police officers Able, Baker, and Carter. The trial was widely reported by the media. The jury returned a verdict in favor of Poller and against the three police officers for $500 million. When Trial Judge Johnston received the verdict, he was shocked by the size of the award. Before dismissing the jurors, Judge Johnston directed the following statements to the jury: When you people were sworn in as jurors in this case, you promised that you would deliver a verdict based on the evidence, and that you would not be swayed by passion or prejudice. You have failed in those duties and made a mockery of justice. You should be ashamed of yourselves. He then dismissed the jury, and the defense lawyers renewed their motion for judgment as a matter of law and, alternatively, moved for a new trial. Judge Johnston announced that he would rule on the motions the following Monday at 10 a.m. in open court. The press reports of the verdict and the judge's comments to the jury created a great public tumult in the city where the case was tried. On the following Monday, the courtroom wag jammed with reporters. Primarily for the purpose of educating the reporters, Judge Johnston first gave a detailed explanation of the legal requirements for granting a renewed motion for judgment as a matter of law and for granting a new trial motion. He then granted the renewed motion for judgment as a matter of law and, alternatively, the motion for a new trial. Were Judge Johnston' s actions proper?

(D) The communication with the reporters was proper, but the statements to the .jury were not.

2-13. Worker Workman sued his employer, Drexel Moving and Storage Co., claiming that he was permanently and totally disabled due to a back injury he suffered on the job. Lawyer Lenhart represented Drexel in the case. Lenhart strongly suspected, but had no proof, that Workman continued his hobby of skydiving after the alleged back injury. In due course, Lenhart met with Workman's lawyer for a settlement discussion. Lenhart told Workman's Lawyer: "We won't give you a dime on this claim; we've got movies of your guy jumping out of an airplane two weeks after his phoney injury." Workman's lawyer excused herself to make a telephone call to Workman. When she asked Workmail whether he had been skydiving after the accident, he admitted that he had. With the consent of their' respective clients, the two lawyers then settled the case for $400. Is Lenhart subject to discipline?

(D) Yes, because Lenhart lied about having movies.

2-27. Attorney Ambrose is admitted to practice only in State A, where he specializes in securities and real estate finance law. In that role, Ambrose advised his client, Corbuster, that the law of State B did not require Corbuster to include information about certain mineral rights in a disclosure statement that Corbuster had to file in State B in order to sell some real estate limited partnership interests to State B citizens. Acting on Ambrose's advice, Corbuster did not disclose the information and did sell partnership interests to State B citizens. Later, Ambrose became a full-time trial court judge in State A Later still, State B brought a criminal action against Corbuster for failing to disclose the mineral rights information in his State B disclosure statement. One of Corbuster's defenses is that he lacked the necessary criminal intent because he was acting in good faith based on the advice of his counsel, Ambrose. Corbuster needs Ambrose's testimony to prove that Ambrose did indeed advise him that he was not required to disclose the mineral rights information. Ambrose, in State A, is beyond the subpoena power of the State B court. May Ambrose voluntarily testify on behalf of Corbuster?

(D) Yes, because his testimony would concern the giving of the advice, not his client's character.

1-31. Wilma is the only living child of widower Warner, age 83. Wamer's main asset is a 51% partnership interest in Mobiland, Ltd., a wealthy real estate syndicate that owns and operates mobile home parks throughout the state. Wilma is married to attorney Atwater. One of Atwater's regular clients, Christopher, asks Atwater to represent him in negotiating the sale of 3,000 acres of roadside property to Mobiland. Mobiland is represented by its own lawyer in the matter. May Atwater represent Christopher?

(D) Yes, but only if Christopher consents after full disclosure of Atwater's connection with Warner.

1-13Two years ago, attorney Azari represented client Claubert in the sale of Blackacre to buyer Boyer. Unbeknownst to Azari, Claubert made some fraudulent statements to Buyer about the value of some mineral deposits on Blackacre. Boyer recently discovered the fraud and is now in Azari's office threatening to immediately file a civil fraud suit against both Claubert and Azari. Boyer accuses Azari of engineering the fraud and helping Claubert carry it out. The only way that Azari can convince Boyer that he had no part in the fraud is to tell Boyer a fact that Claubert disclosed to him in the deepest confidence when he was working on the Blackacre transaction. May Azari disclose the fact without the consent of Claubert?

(D) Yes, even if doing so will subject his client to civil or criminal liability

4-16. Attorney Alpheus worked for two years for the Veteran's Administration. While there, his main function was to investigate claims filed by veterans. During the course of his employment, he once investigated a claim filed by Charles, a Vietnam War veteran. After Alpheus left the Veteran's Administration, the agency denied Charles's claim. Charles comes to Alpheus, who is now engaged in private practice, and asks him to represent him in a suit against the Veteran's Administration for the benefits to which Charles believes he is entitled. Is Alpheus subject to discipline if he accepts Charles's case?

(D) Yes, if Alphas had substantial and personal responsibility for Charle's Veteran's Administration claim.

1-49. Plaintiff Prentice, represented by Lawyer Leer. brought suit in federal district court against Exterminate, Inc., a pest control company, and nine chemical companies for "grave physical and emotional injuries" Prentice suffered after accidentally inhaling cockroach spray emanating from an apartment that had recently been fumigated by Exterminate... Is Leer subject to litigation sanction in the form of an order against Leer personally to pay the $14,500 in attorneys' fees and the $6,750 in litigation costs?

(D) Yes, provided that Calloway can show that Leer either intentionally or recklessly took frivolous positions in order to harass Calloway.

4-13. Attorney Ariadne has an arrangement whereby a local radio station broadcasts four times each day a prerecorded tape advertising her services. Ariadne pays the station its standard rate for "spot advertising" the advertising tape is as follows. Is ad proper?

(D) Yes, there is nothing wrong with Ariadne's advertising.

2-29. When lawyer Locke was an associate in the firm of Bliss & Buford, she did the legal work for one of the firm's clients, Cannon, on a land sale transaction that earned Cannon millions of dollars. In gratitude, Cannon asked Locke whether she had any unfulfilled wishes. Locke told him that she wished she had enough money to start her own solo ]aw practice. Cannon then told her that he would lend her $100,000 to set up her new practice. In return, she would thereafter do all of his legal work at a 5% discount from her normal hourly fee, and she would pay Cannon 10% of the legal fees she earned in solo practice until the $100,000 loan was fully repaid. Locke was delighted. She drafted a complete, detailed agreement between herself and Cannon, and she insisted that he get outside legal advice before signing the agreement. Cannon got the outside advice and signed the agreement, and Locke set up her solo practice accordingly. Is Locke subject to discipline?

(D) Yes, unless Cannon is a lawyer.

1-19. Building contractor Carter and his lawyer Lewis met with landowner Owens to negotiate a contract whereby Carter would construct an office building on land owned by Owens. Carter, Lewis, and Owens were the only persons present at the meeting. Ultimately the three of them worked out a written agreement, and Carter commenced work on the building. It soon became apparent that the building site required far more preparation work than Carter had contemplated when he agreed to the contract price. Carter and Owens got into a dispute about who had to pay for the additional site preparation. One important issue is whether Owens made certain oral representations to Carter during the contract negotiating session that Lewis attended. Carter contends that Owens did make the representations, and Owens contends that he did not. Lewis was present during the entire negotiating session, and she is virtually certain that Owens did not make the representations. Carter stopped work on the building and refused to proceed until Owens paid for the extra site preparation. Owens then sued Carter for specific performance of the construction contract. Carter asked Lewis to represent him as trial counsel. Lewis should:

(D)Decline to serve as trial counsel for Carter, because she can foresee that she will be called as a witness.

1-9. Attorney Anderson received her law degree two years ago from Flatland College of Law and Technical Sciences. Last summer she attended a three-day trial practice seminar at the Harvard Law School. During her brief career, she has tried five cases--two jury trials and three bench trials. She won both of the jury trials and two of the three judge trials. Anderson placed an ad under the subject heading "Trial Lawyers" in the classified pages of the local phonebook. Her ad states in relevant part: Arlene Anderson, Trial Attorney Harvard Trained/Never Lost a Jury Trial/Which of the following make Anderson subject to discipline? I. Placing her ad under the heading "Trial Lawyers" in the phonebook classified pages. II. Describing herself as a "Trial Attorney" in her ad. III. Describing herself as "Harvard Trained" in her ad. IV. Stating "never lost a jury trial" in her ad.

(D)III and IV. only.

1-20. Author Arthur wrote a best-selling novel based on the life and crimes of John Dillinger, the famous bank robber. Arthur sold the movie rights to film producer Prosser, who promised to pay Arthur a lump-sum royalty of $5 million upon release of the movie. After Prosser hired actor Clint Nickleman to play the lead role and made other expensive preparations for filming, Arthur repudiated the contract. Prosser hired lawyer Laine to sue Arthur for a declaratory judgment that the contract was valid and enforceable. At Prosser's request, Laine agreed to do the legal work on a contingent fee basis: If Prosser wins, Laine will be paid 1.75% of the gross receipts from the movie, but if Prosset loses, Laine will be paid nothing. Prosser and Laine entered into a written fee agreement that contains all the details required by the rules of legal ethics. Which of the following statements is true?

(D)Laine's fee agreement is proper, even though it gives Laine a personal interest in the subject of the litigation.

1-8. Seymour is applying for admission to the State A Bar. When Seymour was in high school, he and his parents lived in State B, next door to attorney Azevedo. Azevedo is admitted to practice in State B, but not in State A. Seymour seemed to be a promising lad, and Azevedo was disappointed to learn that during his senior year in high school he was convicted of burglarizing a liquor store. After serving his sentence, Seymour went to college and later to law school. Azevedo has had no contact with Seymour since his high school years, but so far as Azevedo knows, Seymour has not done anything since high school that would reflect badly on his character. The Bar of State A sent Azevedo a routine questionnaire, asking a series of questions about Seymour's character. Azevedo does not know whether Seymour disclosed the burglary conviction on his bar application, and she does not know where to contact him to find out. How should Azevedo respond to the questionnaire?

(D)She should state what she knows about Seymour, including mention of his burglary conviction.

2-1.For the past 40 years, solo practitioner Febell has practiced municipal bond law in State A. Because he is nearing retirement age, Febell takes in young attorney Spryte as a partner. Their partnership agreement provides that Febell will train Spryte in municipal bond law, that Febell will receive 75% of the partnership's net earnings during the first three years, and that Spryte will receive the remaining 25%. The agreement further provides that if Spryte leaves the partnership before the end of the first three years, he will remit to Febell 75% of all tees he earns thereafter from municipal bond work he does in State A. Finally, the agreement provides that if Febell and Spryte are still partners when Febell retires, Spryte will pay Febell retirement benefits of $3,000 per month until Febell's death; in return, upon his retirement, Febell will turn over to Spryte all of the partnership assets (including good will) and will not thereafter practice municipal bond law in State A. Are Febell and Spryte subject to discipline for entering into this partnership agreement?

(D)Yes, because of the restriction on Spryte's right to practice if he leaves the partnership within the first three years.

1-18. State Alpha lawyer LaFrank represents Inventex Corp., a State Alpha corporation that owns a valuable United States patent. State Beta lawyer Levin represents Demonics, Inc., a State Beta corporation. Inventex believes that Demonics is infringing tile Inventex patent, and for the past three months LaFrank has been negotiating with Levin, seeking an amicable resolution of the dispute. The negotiations have broken down, and Inventex has decided to sue Demonics for infringement. Venue in an action for patent infringement is proper in any United States judicial district where the defendant or its agent resides or can constitutionally be served with process. For tactical reasons, LaFrank wants venue in State Alpha. She believes, but is not certain, that Demonics has a branch sales office somewhere in State Alpha. To find out for sure, LaFrank telephones Demonics's headquarters in State Beta and asks to speak with the vice president for sales. Without identifying herself, she asks the vice president where in State Alpha she can find a sales office for Demonics's products. The vice president politely tells her the address of the office and the name of Demonics's head sales agent in State Alpha. Is LaFrank subject to discipline?

(D)Yes, because she talked with the vice president about the matter without getting Levin's consent.

4-48. The legislature of State A conducted open hearings concerning a bill pending in the state legislature that would make it much more difficult for corporations chartered in State A to be taken over by corporate raiders. By making unfriendly takeovers more difficult, the proponents of the bill hope to save jobs in State A and to encourage corporations not now chartered in State A to obtain State A charters which would bring added revenue to the state. Attorney Armor, a senior partner in the prestigious firm of Armor, Baldwin & Chase, asked to testify at the hearings. Armor had been retained by Tentacle Corporation and was asked by Tentacle's president to testify against the pending legislation at the legislative hearings. Tentacle's president also told Armor that under no circumstances was he to tell the legislature that he was working for Tentacle or to mention the name of Tentacle in his testimony. Armor complied with Tentacle's strictures and never mentioned that he was being retained to give testimony against the pending legislation. Armor's testimony before the legislative committee was effective and hard-hitting. After his testimony, Armor was asked a few questions by committee members, but he was never asked if he was appearing on some other party's behalf. Was Armor's conduct at the hearing proper?

(c) No, because he did not disclose that he was appearing at the hearings in a representative capacity.

4-4 Alfonso is the mayor of the city of Dustbowl. Alfonso is also a licensed attorney who has a law partnership with Bella. Under the city charter of Dustbowl, the mayor has the authority to determine what issues are to be placed upon the agenda of the city council. Several council members have told Alfonso that they would like to see a particular zoning measure placed upon the agenda. This proposed ordinance would ban commercial development of a certain area within the city limits. Bella has been retained as attorney for Octopus Development Corp. Octopus has acquired land in the proposed noncommercial zoning area and has plans to construct a large shopping center there. May Bella represent Octopus in this matter?

(c) No, because of Alfonso's position as mayor

A lawyer would be subject to discipline for having their nonlawyer employee do which of the following? 1. Negotiate a settlement for a client 2. Interview a client without the lawyer present 3. Research and draft a motion for the lawyer's review 4. Appear in court on a client's behalf 5. Advise a client on the legal consequences of what they intend to do

1, 4 & 5

Two lawyers from different law firms want to work together on a matter for a client. Which of the following are ALWAYS required for a proper fee splitting arrangement between lawyers at different firms? 1. The client must agree to the split in writing 2. The client must be informed of the share each lawyer will receive 3. The total fee must be reasonable 4. The split must be in proportion to the services provided by each lawyer

1-3

Select the TWO conflict situations where the client must provide informed consent in a SIGNED writing: 1. Direct adversity conflicts 2. Representing co-parties in the same case 3. Business transactions with client 4. Third party paying client's legal fees 5. Aggregate settlement agreements

3 & 5

Valerie meets with her attorney, Anissa, to discuss a tax matter. Valerie's accountant was present to help explain Valerie's finances, Valerie's friend was present because they had dinner plans after the meeting, and Anissa's secretary was present to take notes. The presence of which of the following people will defeat the attorney-client privilege? 1. The accountant 2. The secretary 3. The friend 4. None, because all parties are associated with either Valerie or Anissa

3.

Which of the following arrangements in connection with pending or contemplated litigation are proper? 1. Paying an indigent client's expert witness fees outright, without any provision for repayment 2. Paying a non-indigent client's court costs outright, without any provision for repayment 3. Advancing a non-indigent client's court reporter fees, and telling the client that they do not have to repay the money if they lose their case 4. Guaranteeing a third person's loan to an indigent client for living expenses

3.

What is USUALLY NOT privileged? 1. An authenticated e-mail exchange between a lawyer and client 2. The client's admission to the lawyer that he committed a crime 3. The identity of the client 4. Information about the fee arrangement

4.

Yolanda regularly reviews vendor contracts for ComCorp. A few months ago, they agreed that Yolanda would bill $500 per contract, but they have not had any conversations about fees lately. On Wednesday, Yolanda receives a contract from ComCorp, does the work, and then bills them $500. On Thursday, Yolanda receives a contract from ComCorp that is twice the normal length. She does the work and then bills ComCorp $1,000, with an explanation that the contract was unusually long. Assuming both amounts are reasonable, is Yolanda subject to discipline? A. No, because ComCorp is a regularly represented client B. Yes, for her actions on Thursday only C. Yes, for her actions on both Wednesday and Thursday

A lawyer must, before or within a reasonable time after commencing a representation, communicate the basis or rate of the fee and the expenses for which the client will be responsible. However, if the lawyer regularly represents the client and will be charging the same basis or rate as in other matters, the lawyer need not communicate the fee arrangement each time. A lawyer also has an ongoing duty to communicate any changes regarding the fee arrangement. On Thursday Yolanda doubled her usual charge. Even though the total amount was reasonable, she was required to reach out to ComCorp, explain the change in circumstances, and come to an agreement before proceeding. Because she only explained the change after doing the work, she is subject to discipline.

Two years ago, when a couple divorced in State A, the court awarded the wife custody of the three children and ordered the husband to pay the wife $3,000 per month in child support and alimony payments. The husband failed to make the $3,000 payments for 17 months in a row. In desperation, the wife hired a new attorney to represent her in a proceeding to collect the past due payments from the husband. State A has no law or court rule that requires the loser to pay the winner's attorneys' fees in domestic relations matters. Because the wife had no money to pay her new attorney a regular fee, the new attorney agreed to do the work on a contingent fee basis for 10% of whatever amount the wife was ultimately able to recover. The new attorney won an award for the wife of the entire amount due ($51,000), and by tracking down and attaching the husband's secret bank account, he got the full amount paid to the wife. He then sent the wife a bill for his share, $5,100. Is the wife's new attorney subject to discipline? A) No, as long as $5,100 is a reasonable fee for the work he did. B) No, because the wife had no money to pay a regular fee. C) Yes, because the new attorney used a contingent fee in a domestic relations matter. D) Yes, because the new attorney took a portion of the money that was intended for support of the wife and the children.

A) No, as long as $5,100 is a reasonable fee for the work he did.

A bank and trust company maintains a list of approved estate and trust lawyers as a service to their customers who seek advice on estate planning matters. When a young lawyer opened her trust and estate practice in town, she asked other lawyers how she could get on the bank's approved list. They explained that the bank lists lawyers who always name the bank in wills and trust agreements they draft for clients who need an institutional executor or trustee. The bank is one of the most stable and reputable banks in the state, and its fees for executor and trustee services are competitive with those of similar institutions. In light of what she has been told by the other lawyers, may the young lawyer seek to have her name included on the bank's list? A) No, because a tacit condition of being on the list is always to name the bank as executor or trustee. B) No, because a lawyer must not solicit business through an intermediary. C) Yes, because naming the bank causes no harm to clients who need an institutional executor or trustee. D) Yes, because those who use the bank's list are already bank customers.

A) No, because a tacit condition of being on the list is always to name the bank as executor or trustee.

A potential client consulted an attorney, hoping to hire her to represent him as plaintiff in a medical malpractice action against his doctor. Without mentioning the doctor's name, the client described the alleged acts of malpractice and said that they happened more than two years ago. Only at that point did the potential client mention his doctor's name. The attorney immediately stopped the potential client and said she could not represent him because she was already representing the doctor in an unrelated matter, and she urged him to consult another lawyer. That was the end of the conversation. The potential client did nothing further for 15 months, at which point he consulted another lawyer. By that time, the statute of limitations had run on the potential client's claim against the doctor. The potential client then sued the first attorney for legal malpractice, alleging that the attorney was negligent in not warning him about the statute of limitations. Is the attorney subject to civil liability in the potential client's malpractice case? A) No, because the attorney did what a reasonably prudent lawyer would do in the circumstances—decline to represent the potential client and suggest that he consult other counsel. B) No, because the potential client never became the attorney's client and is therefore not a proper plaintiff in a malpractice action against the attorney. C) Yes, because the attorney had no legal or ethical reason to reject the potential client as a client and therefore had a duty to warn him about the statute of limitations. D) Yes, because a reasonably prudent lawyer would have foreseen that the potential client might delay in consulting another lawyer.

A) No, because the attorney did what a reasonably prudent lawyer would do in the circumstances—decline to represent the potential client and suggest that he consult other counsel.

An insurance company offers a legal services insurance policy. In return for a yearly premium, an insured will be reimbursed by the insurance company for a specified amount for legal services during the year. The insured selects a lawyer from a list of "authorized providers" supplied by the insurance company. Any lawyer who agrees to follow a maximum fee schedule set by the insurance company can become an "authorized provider." The insurance company solicits insurance sales by in-person and live telephone contact with potential insurance buyers, working systematically through local telephone directories. Will an attorney be subject to discipline if he becomes an "authorized provider" and receives clients through the insurance company's insurance plan? A) No, because the insurance company does not specifically target persons whom it knows are in need of legal services in a particular matter covered by its insurance plan. B) No, because the insurance company's insureds are allowed to select whatever lawyer they wish from among the "authorized providers." C) Yes, because the insurance company uses a specified maximum fee schedule. D) Yes, because the insurance company uses in-person and live telephone solicitation to get business.

A) No, because the insurance company does not specifically target persons whom it knows are in need of legal services in a particular matter covered by its insurance plan.

A lawyer is a partner in a private law firm. That firm regularly provides legal services to three major banks and two other important lending institutions in the community. The lawyer has been invited to become a member of the board of directors of the local legal aid society, the group that sets overall governing policies for the local legal aid office. One of the major issues that will soon face the board of directors is whether to amend the case intake guidelines to allow the legal aid office to represent clients in disputes with banks and other lending agencies. Which of the following statements is correct? A) The lawyer may join the board of directors, but she must refrain from participating in the decision about the case intake guidelines. B) The lawyer will be subject to discipline if she joins the board of directors because service on the board is in conflict with the interests of her firm's bank and lending institution clients. C) It would be proper for the lawyer to join the board of directors, and it would be proper for her to participate in the decision about the case intake guidelines. D) The lawyer may join the board of directors to help discharge her pro bono obligation, and she may vote in favor of amending the case intake guidelines in order to make it easier for low income persons to sue banks and other lending institutions.

A) The lawyer may join the board of directors, but she must refrain from participating in the decision about the case intake guidelines.

A retired lawyer practiced admiralty and maritime law for 45 years in Maine. He stopped paying his bar dues in Maine when he retired, and he is no longer licensed to practice there. He and his wife moved to a retirement village in New Mexico, but he did not seek to become licensed to practice law in New Mexico. After a few months of playing golf and puttering in the garden, the retired lawyer got bored and started missing the challenges of law practice. He therefore joined the unpaid staff of volunteer lawyers at the Rio Grande Walk-In Legal Advice Clinic, which is run by a nonprofit organization. The clinic's purpose is to offer free, quick, accurate, compassionate legal advice to walk-in clients who cannot afford ordinary legal service and who have legal problems that can be solved quickly, without litigation or other time-consuming procedures. Before they ever see one of the clinic's lawyers, all of the clients must give informed consent to the limited nature of the legal services they will receive. The retired lawyer works at the clinic three days a week, and he dispenses legal advice on all sorts of matters—although he has yet to find a client who needed admiralty or maritime advice. The retired lawyer enjoys the work because it makes him feel useful again, and because it gives him a cornucopia of interesting stories to tell his wife about his clients' various legal troubles. Which of the following statements is correct? A) The retired lawyer is subject to discipline for practicing law without a license. B) The retired lawyer is subject to discipline for failing to pay his bar dues in Maine. C) The retired lawyer's volunteer work is proper because one does not need to be licensed to dispense legal advice at a quick-service clinic like this one. D) The retired lawyer's conversations with his wife are proper because no confidential lawyer-client relationship is formed at a quick-service clinic like this one.

A) The retired lawyer is subject to discipline for practicing law without a license.

An attorney received her law degree two years ago from a small local college of law and technical sciences. Last summer she attended a three-day trial practice seminar at the Harvard Law School. During her brief career, she has tried five cases—two jury trials and three bench trials. She won both of the jury trials and two of the three bench trials. The attorney placed an ad under the subject heading "Trial Lawyers" in the classified pages of the local phone book. Her ad states in relevant part: "Trial Attorney Harvard Trained Never Lost a Jury Trial" Which of the following is correct? A) To make the ad proper, the references to "Harvard Trained" and "Never Lost a Jury Trial" must be deleted. B) To make the ad proper, the references to "Trial Attorney" and "Harvard Trained" must be deleted. C) To make the ad proper, the references to "Trial Attorney" and "Never Lost a Jury Trial" must be deleted. D) The ad is proper as written.

A) To make the ad proper, the references to "Harvard Trained" and "Never Lost a Jury Trial" must be deleted.

An attorney is representing a defendant on trial for armed robbery of a liquor store. The defendant tells the attorney in confidence that at the time in question, he was sitting at home watching television with his aged mother, and that his mother can confirm his alibi. The attorney interviews the mother, who solemnly confirms the defendant's story. After talking with her, the attorney strongly suspects that she is lying to protect the defendant. The attorney does not know for sure that the defendant and his mother are lying, but every instinct tells him that they are. The attorney has warned both of them about the dangers of perjury, but both have insisted that they want to testify to the alibi at trial. May the attorney call the defendant, or his mother, or both, as trial witnesses? A) Yes, as to both the defendant and his mother. B) Yes, as to the defendant, but no, as to his mother. C) No, as to both the defendant and his mother. D) No, as to the defendant, but yes, as to his mother.

A) Yes, as to both the defendant and his mother.

A judge serves on a state trial court that has nine other judges. Her husband is a life insurance salesman for a large life insurance company. The life insurance company is occasionally a litigant in the court on which the judge sits. Every year the life insurance company runs a national sales contest in which the person who sells the most life insurance during the year receives a valuable prize. The judge's husband won this year and took the judge on an all-expense-paid vacation in Europe. She did not make a public report of the prize. Was it proper for the judge to allow her husband to accept the prize and take her on the European vacation? A) Yes, because acceptance of the prize cannot reasonably be perceived as undermining the judge's integrity or impartiality. B) Yes, because the prize was won by her husband, not by the judge. C) No, because the judge did not make a public report of the prize. D) No, because the life insurance company may later appear as a litigant in the court on which the judge sits.

A) Yes, because acceptance of the prize cannot reasonably be perceived as undermining the judge's integrity or impartiality.

A lawyer represents the defendant in a criminal case. The defendant is charged with vehicular homicide, a felony. Under the criminal statute in question, a defendant is guilty if he caused the victim's death by driving a motor vehicle either intentionally or recklessly in disregard of the safety of others. In the defendant's case, the critical issue is whether the traffic light facing the defendant's traffic lane was green at a specified moment. If the light was green, then the defendant is not guilty, but if it was red, then the defendant is guilty. The defendant himself has blocked the entire event from memory and has no idea whether the light was green or red. Five bystanders were in a position to see the light at the time in question. The lawyer interviewed four of them. With varying degrees of uncertainty, all four of them told the lawyer that they believe the light was red but that they are not positive. Based on their recollections, as well as certain physical evidence in the case, the lawyer herself believes that the light was probably red, but of course she was not present at the scene and cannot be certain. Then the lawyer interviewed the fifth bystander, who said that he simply could not remember what color the traffic light was. The lawyer replied: "My client is facing 20 years in jail, and the whole case against him turns on the color of that light. My client and I would both be eternally grateful to you if you could testify that the light was green. Would you help us out?" After thinking it over, the fifth bystander said he would be glad to help by testifying that the light was green. At the trial, the lawyer presented the fifth bystander's testimony that he saw the light, that he remembers what color it was, and that it was green. The jury believed the fifth bystander, and the defendant was acquitted. Is the lawyer subject to criminal liability for inducing the fifth bystander to testify falsely? A) Yes, because both the bystander and the lawyer knew that the bystander did not remember what color the light was. B) Yes, because neither the bystander nor the lawyer was certain that the light was green. C) No, because neither the bystander nor the lawyer was certain what color the light was. D) No, because the defense lawyer in a criminal case must resolve all doubtful facts in her client's favor when she presents evidence on her client's behalf.

A) Yes, because both the bystander and the lawyer knew that the bystander did not remember what color the light was.

A young lawyer, three years out of law school, had never set foot in a courtroom. The lawyer was on the board of directors of a nonprofit preschool. One of the preschool's teachers was charged with felony child abuse for allegedly molesting three pupils. After conducting its own careful investigation, the preschool's board of directors concluded that the criminal charge was totally unfounded, and the board resolved to provide defense counsel for the teacher. The young lawyer volunteered to do the work without a fee. A few days before the trial was to begin, the lawyer became convinced that he was not competent to serve as the teacher's trial counsel. He asked the trial judge for permission to withdraw. After thoroughly questioning the lawyer about his preparation for trial, the judge said that while he understood the lawyer's anxiety, he believed that the lawyer was perfectly competent to handle the case. The judge denied the lawyer's motion to withdraw but postponed the trial for seven days to allow him to complete his preparation. Instead of doing what the judge ordered, the lawyer advised the teacher that he would not defend her. He handed her all of the files in the case and advised her to retain another lawyer. Is the lawyer subject to discipline? A) Yes, because he abandoned his client in direct violation of the trial judge's order. B) Yes, because he undertook a case that he was not competent to handle. C) No, because he believed that he was not competent to represent his client at trial. D) No, because he was working pro bono, not for a fee.

A) Yes, because he abandoned his client in direct violation of the trial judge's order.

A lawyer assigned his secretary to manage his client trust account. The lawyer gave the secretary extensive, detailed instructions about the kinds of records to keep, the kinds of funds that she must deposit, and the kinds of permissible withdrawals that she could make. The lawyer had complete faith in the secretary's ability and honesty, and therefore did not supervise the secretary's management of the account. Three years later, during an audit, it was discovered that on 18 different occasions during that period, the account balance fell below the amount that should have been there. The lawyer was unaware of these occasions until he received a copy of the audit. Is the lawyer subject to discipline? A) Yes, because he did not adequately supervise the secretary. B) Yes, because a lawyer must manage his client trust account himself. C) No, because he took reasonable steps to train the secretary and did not realize that the account balance had fallen below the proper level. D) No, because he did not have actual knowledge that the secretary was not performing the account management function properly.

A) Yes, because he did not adequately supervise the secretary.

A full-time judge lives in State A. Her father lives in a retirement home in State B. The judge's father told her that several of his friends in the retirement home had employed an attorney to write wills for them, and that in each will the attorney had included a bequest to himself. Each bequest was approximately 50% of the estimated total value of the person's probable estate. The friends told the judge's father that they did not really want to leave the attorney anything, but they had assumed it was merely a matter of routine, a part of the attorney's compensation for drafting the will. The attorney is admitted to practice in State B, but not in State A. The judge did not talk personally with any of her father's friends, but she believes that her father's rendition of the story is entirely accurate. Would it be proper for the judge to communicate directly with the attorney about the matter, and if that does not satisfy her, to communicate with the attorney disciplinary authority in State B about the matter? A) Yes, because she has received information indicating a substantial likelihood that the attorney has violated a legal ethics rule. B) Yes, because she has personal knowledge that the attorney has violated a legal ethics rule. C) No, because legal ethics violations that take place outside State A are not her concern. D) No, because she is not allowed to communicate directly with the attorney about the supposed legal ethics violation.

A) Yes, because she has received information indicating a substantial likelihood that the attorney has violated a legal ethics rule.

After graduating from law school, an attorney was admitted to practice in one state and not in any other jurisdiction. She joined the United States Army Judge Advocate General's ("JAG") Corps—the corps of lawyer-soldiers who provide legal services to the Army throughout the world. After completing her officer training and her training in military law, she was assigned to the JAG office at a military base in a different state. Even though she was not admitted to practice in that state, she was assigned to the legal assistance desk. According to Army regulations, her job is to provide legal services to military personnel and their dependents concerning a wide range of personal legal problems, including civil, domestic, and financial matters. An officer and his wife ask the attorney for legal advice about financing a mobile home, which they plan to put in a mobile home park located in the town closest to the military base. The attorney knows absolutely nothing about the business and legal issues involved in financing a mobile home, but she is willing to undertake additional research to learn about these issues. Would it be proper for the attorney to give the requested advice to the officer and his wife? A) Yes, because she is willing to do the research necessary to give competent advice on mobile home financing. B) No, because she is not knowledgeable about these business and legal issues. C) No, because she is not admitted to practice general civil law in the new state. D) No, because mobile home financing is not directly related to the Army's mission.

A) Yes, because she is willing to do the research necessary to give competent advice on mobile home financing.

An attorney represented a defendant in a criminal trial. After the jury returned a guilty verdict, the defendant was taken to jail and the jury was discharged. While walking to his car, the disappointed attorney spotted one of the courtroom spectators in the parking lot. The attorney recalled that the spectator had been a member of the jury pool, but he had exercised a peremptory challenge against her because he instinctively felt that she would vote against the defendant. Despite not being selected as a juror, the spectator developed an interest in the case and had attended the entire trial. In an attempt to determine whether his instinct during jury selection was correct, the attorney approached the spectator and asked her whether she would have voted to convict the defendant. The spectator said, "I'd rather not talk about it." When the attorney explained that he was simply looking for constructive feedback, the spectator changed her mind and agreed to a brief interview. The attorney and spectator spoke for a few minutes, and the communication did not involve misrepresentation, coercion, duress, or harassment. Is the attorney subject to discipline? A) Yes, because the spectator initially declined to speak with the attorney. B) Yes, because post-trial contact with prospective jurors is prohibited. C) No, because the communication did not involve misrepresentation, coercion, duress, or harassment. D) No, because the spectator was not chosen for the jury.

A) Yes, because the spectator initially declined to speak with the attorney.

Generally, a lawyer must not share fees with a nonlawyer. Which of the following is NOT an exception to this rule? A. Paying a paralegal a portion of a fee based on how much work they did on that matter B. Paying death benefits to a deceased lawyer's estate C. Sharing a court-awarded legal fee with the nonprofit organization that hired or recommended the lawyer in the matter D. Purchasing the law practice of a deceased, disabled, or disappeared lawyer E. Paying the firm's administrative assistants a salary or bonus

A. A lawyer generally can't share fees with a nonlawyer, except for the scenarios laid out in (B)-(E). The situation in (A) is a classic example of prohibited fee-splitting. A lawyer may of course pay nonlawyer employees in accordance with a regular compensation plan, profit-sharing bonus, etc. However, sharing a specific fee with a nonlawyer employee (e.g., based on their work in the case) is prohibited.

Darby, an author, hires Lee to represent her in contract negotiations with a publisher. Darby eventually stops paying Lee's bills, in violation of their fee agreement. Lee warns her that he will withdraw if she doesn't start making payments, but she refuses. Even though the negotiations are at a critical point, Lee withdraws. Darby is forced to find a new lawyer and start the process again. Is Lee subject to discipline? A. No, because he had good cause to withdraw B. Yes, because he withdrew knowing it would have material adverse effect on Darby's interests

A. A lawyer may withdraw from representing a client if the client substantially fails to fulfill an obligation to the lawyer (paying their legal bills for example) and has been warned that the lawyer will withdraw unless it's fulfilled.

Julio represents Sarah in an attempted murder case. The prosecution offers a plea deal that would require Sarah to serve five years in jail. Julio strongly believes Sarah will be acquitted at trial, and explains this to her, but she tells him she does not want to risk ending up with the maximum sentence of 30 years. What should Julio do? A. Abide by Sarah's decision B. Act in Sarah's best interests and reject the deal C. Take protective action, because Sarah is at risk of substantial harm in prison

A. A lawyer must abide by the client's decision with regard to certain substantive matters in the case, including pleas.

Jean hires Louis to negotiate the contract for her first mystery novel. Jean wants to give Louis 2% of the book royalties in return for his services. Is the arrangement proper? A. Yes, as long as Louis complies with the requirements for business transactions with a client B. No, because a lawyer must not acquire literary or media rights relating to the representation while the matter is pending

A. A lawyer must not acquire literary or media rights to a story based in substantial part on information relating to the lawyer's representation of a client (e.g., a lawyer representing a client in a pending murder trial must not acquire rights to a book about the case). However, a lawyer may acquire these rights after the client's legal matter is entirely completed, appeals and all. Furthermore, the rule doesn't apply to literary or media rights that aren't substantially based on information relating to the representation. Here, the prohibition doesn't apply because the mystery novel is a work of fiction—it isn't based on information relating to the contract negotiation. However, Louis still must comply with the requirements for business transactions with a client.

Abby wants to sell her car to one of her clients. Which condition does NOT have to be satisfied for Abby to properly enter into the transaction? A. The client must seek the advice of an independent lawyer B. The client must give informed consent in a signed writing C. The terms must be fair to the client D. The terms must be transmitted to the client in writing

A. A lawyer must not enter into a business transaction with a client unless: (i) the transaction and terms under which the lawyer acquires the interest are fair and reasonable to the client; (ii) the terms are fully disclosed and transmitted in writing to the client in a manner that can be reasonably understood by the client; (iii) the client is advised in writing that they should seek the advice of independent legal counsel on the transaction (if they don't already have counsel in the transaction), and is given a reasonable opportunity to do so; and (iv) the client gives informed consent in a signed writing. It's not required that the client actually seek advice from independent counsel. (Note that the rules regarding business transactions with clients don't apply to ordinary fee agreements, or to standard commercial transactions for products or services that the client routinely markets to others.)

William wants Lena to represent him in the sale of his home to Peter. Lena represents Peter in an unrelated matter. Lena reasonably believes that she can competently and diligently represent them both in their respective matters. What must Lena do before undertaking the representation of William? A. Get informed consent, confirmed in writing, from both William and Peter B. Nothing, because this is a transactional matter C. Nothing, because Lena represents Peter in unrelated matters

A. Direct adversity exists when a lawyer represents one client in a matter adverse to another current client, even where the lawyer represents the clients in unrelated matters. This is true in both litigation and transactional matters. Thus, Lena must address the conflict by obtaining informed consent, confirmed in writing, from both Peter and William.

Even with client consent, a lawyer litigating a matter for a client must not assert a claim against another client whom the lawyer represents: A. In the same litigation B. In the same litigation or in an unrelated matter

A. Even with client consent, a lawyer must not assert a claim on behalf of one client against another client represented by that lawyer in the same litigation (or other proceeding before a tribunal). In contrast, if the lawyer represents the second client in unrelated matters only, the conflict may be waivable.

Maddy represents Colin in a personal injury case against a defendant. During settlement negotiations, the defense offers $50,000. Maddy is certain that this is just a lowball opening offer, but she knows that Colin is a pessimist and will accept it. Maddy rejects the offer on Colin's behalf and does not tell him about it. The following week, the defense offers $75,000. Maddy presents the offer to Colin and he accepts. Is Maddy subject to discipline? A. Yes, because she failed to tell Colin about the first offer B. No, because Colin suffered no harm C. No, because Colin was likely to react imprudently to the offer

A. If an adversary offers to settle a civil case, or offers a plea bargain in a criminal case, the lawyer must promptly convey the offer to her client unless the client has previously instructed the lawyer that an offer on those terms is acceptable or unacceptable or has authorized the lawyer to accept or reject such an offer. Maddy will be subject to discipline for failing to communicate the offer, even though Colin ultimately ended up with a higher amount. A lawyer is subject to discipline for violating an ethics rule even when the client isn't harmed.

Emily previously represented building owner Sunil in various landlord-tenant disputes. Sunil e-mails Emily and says he wants her to represent him in a new dispute with a tenant. Emily and Sunil meet and spend a few hours discussing strategy. Emily never expressly accepts or declines the case during the meeting. Is there an attorney-client relationship between Emily and Sunil? A.Yes B. No

A. One way an attorney-client relationship is formed is when a person manifests an intent that the lawyer provide legal services and the lawyer agrees. The lawyer's agreement does not have to be an express statement. Sunil clearly stated that he wanted Emily to represent him in the dispute, and Emily manifested consent by meeting with Sunil and discussing strategy. Their prior relationship is not determinative but further supports the conclusion that they formed an attorney-client relationship with respect to this new matter.

Jason is charged with bank robbery. Jason meets with his attorney, Ben, and says, "I'm going to be straight with you. I did it." Is the communication privileged? A. Yes B. No

A. The attorney-client privilege doesn't apply if the client seeks the attorney's services to engage in or assist a future crime or fraud, but this crime-fraud exception doesn't apply to communications about past crimes.

Justin calls his lawyer, Vanessa, and tells her that he killed his friend the previous night and he buried the body in his backyard. Vanessa calls the authorities and passes on the information. Is Vanessa subject to discipline? A. Yes B. No

A. The ethical duty of confidentiality provides that a lawyer must not reveal any information relating to the representation unless the client has given informed consent, consent is impliedly authorized, or an exception to the duty applies. The closest exception is the "prevention of reasonably certain death or substantial bodily harm" exception, but here the friend is already dead. Consequently, Vanessa is subject to discipline for disclosing the information.

Which of the following does NOT accurately describe an exception to the duty of confidentiality? A. Preventing crimes or frauds by a client B. Preventing or mitigating substantial financial injury to a person caused by a client's crime or fraud involving the lawyer's services C. Preventing reasonably certain substantial death or bodily harm D. Obtaining legal ethics advice E. Defending a lawyer against a charge of wrongdoing

A. There is no exception to the duty of confidentiality for preventing a client's crimes or frauds in general. However, the exceptions in (B) and (C) might apply to a client's crime or fraud. The exceptions in (D) and (E) are also recognized exceptions.

Hank visited his lawyer, Gemma, and they discussed a sensitive legal matter in Gemma's office. They reasonably believed that no one else was in the office. However, Paul was hiding behind a curtain and heard everything. Can Hank assert the attorney-client privilege to prevent Paul from testifying about the conversation? A. Yes B. No

A. To be covered by the attorney-client privilege, a communication must be "confidential"; it must have been made by a means not intended to disclose the communicated information to outsiders, and the communicating person must reasonably believe that no one except a privileged person will hear the contents of the statement. For example, if a lawyer and a client had a discussion in a crowded courtroom, it would not be privileged, but under modern evidence law, a communication remains privileged if an eavesdropper is present.

Firm A and Firm B have been in merger discussions for some time, but they are concerned about possible conflicts of interest. Firm A asks Firm B to send a list summarizing each of its matters, including the name of the client involved, a brief summary of the general issues, and the estimated value of the claim. Firm B complies.Is Firm B subject to discipline? A. Yes, but only for disclosing the estimated value of each matter B. Yes, for disclosing any of the information C. Yes, but only for disclosing the client's names D. No, because the disclosure falls within an exception to the duty of confidentiality

A. When a lawyer changes firms, when two firms merge, or when a law practice is being purchased, lawyers may disclose limited client information (e.g., client names and a brief summary of the general issues involved) in order to detect and resolve conflicts of interest, subject to the following conditions: (i) the disclosure may be made only after substantive discussions regarding the new relationship have occurred; (ii) the disclosure must be limited to the minimum necessary to detect any conflicts of interest; (iii) the disclosed information must not compromise the attorney-client privilege or otherwise prejudice the clients; and (iv) the disclosed information may be used only to the extent necessary to detect and resolve any conflicts of interest. It was proper to disclose the names of the clients and a brief summary of the issues involved

Beth, an entertainment lawyer, forms a partnership with Jeff, a nonlawyer talent agent. The partnership's purpose is to provide clients with a "one stop shop" for their needs. Beth provided legal services on one side of the office, and Jeff worked as a talent agent on the other side of the office. Jeff never did any legal work, and Beth made sure to warn all clients that the protections of the attorney-client relationship did not apply to their dealings with Jeff. Is Beth subject to discipline? A. Yes B. No

A. Yes A lawyer must not form a partnership with a nonlawyer if any part of the partnership activities will constitute the practice of law. Because Beth formed a partnership with nonlawyer Jeff, and the partnership was meant to provide legal services, Beth is subject to discipline. Make sure to distinguish this rule from the law-related services rule. It would have been proper for Beth to control a separate entity that provided law-related services. However, Beth would have been subject to the rules of professional conduct with respect to the non-legal services unless she made it clear that the protections of the attorney-client relationship did not apply.

The State A Bar has established an Interest on Lawyers' Trust Accounts ("IOLTA") program, whereby lawyers deposit client trust funds into special client trust accounts that pay interest to the State A Bar, which then uses the money to help fund legal services for poor people. The program requires lawyers to deposit a particular client's funds in an IOLTA account unless the funds would earn more than $50 in interest during the time they are entrusted to the lawyer. If the client's funds would earn more than $50 in interest during that time, the lawyer must deposit them in a separate interest-bearing trust account and pay the interest to the client. A lawyer settled a personal injury case brought by her client. The defendant sent the lawyer a check for $9,000. Because she was leaving that day for a one-month vacation, the lawyer instructed her assistant to deposit the check in the lawyer's IOLTA account. The assistant is authorized to make deposits to and withdrawals from the account. The lawyer did not tell her assistant to notify the client that the check had arrived. When the lawyer returned a month later, she notified the client that the check had been received, and the client came to the lawyer's office that same day to collect the $9,000. At the prevailing rate of interest, the $9,000 would have earned $40 during the month that the lawyer was gone. Was the lawyer's handling of the matter proper? A) No, because she should have instructed her assistant to deposit the check in a separate trust account that would earn interest for the client. B) No, because she should have instructed her assistant to notify the client promptly that the check had arrived. C) Yes, because she handled the matter in accordance with the State A IOLTA program. D) Yes, because the client was not harmed.

B) No, because she should have instructed her assistant to notify the client promptly that the check had arrived.

A personal injury lawyer and an orthopedic surgeon are good friends, and they have a high mutual regard for each other's professional abilities. One day on the golf course, they made a reciprocal referral agreement: whenever the lawyer has a personal injury client with need for an orthopedic surgeon, the lawyer promised to refer the client to the surgeon. Similarly, whenever the surgeon has an injured patient with a need for a personal injury lawyer, the surgeon promised to refer the patient to the lawyer. The agreement was oral, not written, and there was no mention of an expiration date; both women simply assumed that the agreement would continue indefinitely until one or the other wanted to end it. Likewise, they did not discuss whether the agreement would be exclusive; both women simply assumed that neither of them would refer someone to a competitor of the other. Was it proper for the lawyer to make this agreement with the surgeon? A) No, because the agreement was not reduced to writing. B) No, because the agreement was of an indefinite duration. C) No, because a lawyer must not give anything of value to a person for recommending her services. D) No, because a lawyer must not enter into a reciprocal referral agreement with a nonlawyer.

B) No, because the agreement was of an indefinite duration.

A 12-year-old boy was badly injured when he was struck by a dump truck owned by a construction company and driven by the company's employee. The boy and his parents sued the construction company and the employee. The first count of their complaint alleges that the employee drove negligently while acting within the scope of his duties for the construction company, and that the construction company is therefore liable for the boy's injuries. The second count alleges that the employee drove negligently while on a frolic of his own, and that the employee is therefore liable for the boy's injuries. The construction company hired a lawyer to defend both the construction company and its employee. The lawyer conducted a careful investigation of the facts and concluded that the employee was in no way negligent; he was driving slowly and carefully when the boy suddenly ran out into traffic from between two parked cars. The lawyer further concluded that the employee was acting within the scope of his duties when the accident happened. The lawyer concluded that he could win the case because of the lack of negligence, and that he could effectively represent both the employee and the construction company. He then carefully explained the potential conflicts of interest to both of them and obtained their informed consent, confirmed in writing, to the joint representation. After exhaustive discovery proceedings, the lawyer remained convinced that the employee was not negligent, but he nonetheless explained the potential conflicts to the employee and the construction company a second time and again obtained their informed consent, confirmed in writing, to the joint representation. Three weeks before the case was scheduled for trial, counsel for the plaintiffs moved to disqualify the lawyer due to a conflict of interest between the employee and the construction company. Must the trial judge disqualify the lawyer? A) No, because there is no actual or potential conflict between the employee and the construction company. B) No, because the employee and the construction company gave informed consent, confirmed in writing, to the joint representation. C) Yes, because the potential conflict creates an appearance of impropriety. D) Yes, even though the employee and the construction company gave informed consent, confirmed in writing, to the joint representation.

B) No, because the employee and the construction company gave informed consent, confirmed in writing, to the joint representation.

A client lives in State A and is a regular client of an attorney who is admitted to practice only in State A. When the client was on vacation in distant State B, she was injured in a car accident caused by a resident of State B. The client hired the attorney to represent her in a civil action against the State B driver. For reasons of jurisdiction and venue, the case had to be filed and tried in State B. The written fee agreement between the client and the attorney provided that: (1) The attorney would assume full responsibility for the case as lead lawyer; (2) The client would pay the attorney 40% of the net recovery after deduction of litigation expenses; (3) The attorney would associate a State B lawyer to serve as trial counsel in State B; (4) The State B attorney would assume responsibility only for his work as trial counsel; and (5) The attorney would pay the State B attorney an appropriate portion of the 40% contingent fee. Would it be proper for the attorney to split his fee with the State B attorney under the circumstances described above? A) No, because the attorney is not admitted in State B) No, because the share that each lawyer will receive was not disclosed in the written fee agreement. C) Yes, because the State B attorney was assuming responsibility for his work as trial counsel. D) Yes, because there was a written fee agreement.

B) No, because the share that each lawyer will receive was not disclosed in the written fee agreement.

For many years a lawyer has done business transactions work for a wealthy client. The client was recently injured in an automobile crash, and she has asked the lawyer to represent her as plaintiff in an action against the driver who injured her. The lawyer has taken some business cases to trial, but he has never handled a personal injury case. The lawyer would like to help his client and also generate some income. Which of the following would be an improper way for him to do so? A) Take the case and, with the client's consent, associate a co-counsel who is competent in the field of personal injury law. B) Refer the client to a competent personal injury lawyer and charge that lawyer a $1,000 forwarding fee. C) Refer the client to a competent personal injury lawyer and charge the client a reasonable sum for the time spent in making the referral. D) Take the case and, with the client's consent, undertake additional research to bring himself up to speed in the field of personal injury law.

B) Refer the client to a competent personal injury lawyer and charge that lawyer a $1,000 forwarding fee.

A lawyer who limits his practice to bankruptcy law has signed up on the local court roster of attorneys who are willing to take court-appointed criminal defense matters on a pro bono basis. He has taken approximately one such pro bono criminal case each of the past 10 years, but he has won only two of them. The day after tomorrow, the lawyer will start the jury trial of a criminal defendant charged with indecent exposure. This morning, the prosecutor held a press conference, at which he told reporters that this defendant had been accused of various sex offenses on six prior occasions. The prosecutor's statements are correct, but none of the prior incidents will be admissible in evidence at the upcoming trial. The defendant's lawyer thinks that the prosecutor was simply trying to poison the jury pool by degrading the defendant. The lawyer is planning to call his own press conference at which he will give the reporters the rest of the story. The lawyer intends to explain that on all six prior occasions, the defendant was arrested but never charged, and all six arrests were made by the same police officer, who holds a personal grudge against the defendant. Which of the following is correct? A) The lawyer is subject to discipline for accepting this court appointment in light of his apparent lack of talent for criminal trial work. B) The lawyer's proposed statements at the press conference are proper in light of the prosecutor's prior statements to the press. C) The lawyer is subject to discipline for accepting court appointments in criminal matters when his active practice is limited to bankruptcy law. D) The lawyer will be subject to discipline if he holds the press conference and makes the statements described above.

B) The lawyer's proposed statements at the press conference are proper in light of the prosecutor's prior statements to the press.

A client hired an attorney to put together a complex real estate syndicate. In connection with that work, the client disclosed to the attorney a great deal of confidential information about the client's financial affairs. When the task was about half completed, the attorney's wife was killed in a car accident and his family's house burned down, all in the same week. The attorney was so emotionally and physically drained that he felt he could not competently continue with the work for his client. The client refused to allow the attorney to withdraw. The attorney begged the client to allow him to turn the files over to his law partner, an excellent real estate lawyer who was completely trustworthy and perfectly competent to handle the matter. The client refused to allow his files to be turned over to any other lawyer and insisted that the attorney himself promptly complete the work. What should the attorney do? A) Turn the files over to his partner, and remain available to assist his partner to the extent possible. B) Withdraw and turn the client's files over to the client. C) Set the client's work aside until he recovers from the ills that have befallen him. D) Continue with the matter and do the best that he can under the circumstances.

B) Withdraw and turn the client's files over to the client.

A lawyer is defending a marine supply company in a civil action brought by the state attorney general under a statute that makes it a civil offense for any person or business entity to bribe or give a kickback to a state official. The statute authorizes fines of up to $100,000 per transaction for any violation. The marine supply company has a strict corporate policy that prohibits its employees from bribing or giving kickbacks to anyone. Employees who violate the policy are subject to immediate discharge and are required to indemnify the marine supply company for any loss it suffers as a consequence of the violation. The attorney general has noticed the depositions of dozens of the marine supply company's employees. One of these employees, prior to his recent retirement, was the sales manager of the marine supply company. The lawyer met with this employee to prepare him for his deposition. At the outset of the interview, the lawyer agreed to represent the employee without charge, and the lawyer told the employee that anything said between them would be confidential. During the interview, the lawyer asked the employee whether he had ever bribed any state officials. The employee confessed that he had, but said it had been necessary because all of the company's competitors were doing it, too. What course of action may the lawyer pursue at this point? A) Withdraw from the case and inform the attorney general what the employee said. B) Withdraw from the case and keep the employee's statement in confidence. C) Withdraw from representing the employee and inform the marine supply company what the employee said. D) Continue in the case, inform the marine supply company what the employee said, and advise the marine supply company to seek prompt settlement.

B) Withdraw from the case and keep the employee's statement in confidence.

A judge sits on a federal appellate court. He and two other federal judges heard a diversity of citizenship case in which they were required to interpret a state statute concerning the marital communications privilege. The judge's two colleagues wrote the majority opinion, in which they concluded that the statute gives only the witness-spouse the right to claim the privilege. The judge wrote a vigorous and scholarly dissent, arguing that the statute gives both spouses the right to claim the privilege. Later, a state senator introduced a bill to amend the statute to reflect the judge's position. The state senate invited the judge to testify about the public policy reasons for giving both spouses the right to claim the privilege. May the judge testify? A) Yes, but only if the two judges who wrote the majority opinion are also allowed to testify. B) Yes, because a judge may engage in activities designed to improve the law. C) No, because a judge must not become involved in politics, subject to certain exceptions that do not apply here. D) No, because a judge is not allowed to make public statements about disputed propositions of law, except when acting in his judicial capacity.

B) Yes, because a judge may engage in activities designed to improve the law.

A lawyer practices real estate law in an old-fashioned jurisdiction in which almost every real estate transaction requires the services of one or more lawyers. The lawyer is also licensed by the state as a real estate broker. The lawyer conducts her law practice and her real estate brokerage business in a single office, using one secretary and one paralegal as her support staff. The lawyer specializes in small, relatively old apartment buildings that are not in peak condition. They make good investments because they can be bought cheap, fixed up, and leased at favorable rates. When the lawyer hears that an owner of a suitable building is looking to sell, she visits them in person and asks them to consider using her to find a buyer. After an owner signs her up as their real estate broker, the lawyer lets them know that she can also do the necessary legal work—the title search, the financing documents, the land transfer documents, and the like. Is the lawyer subject to discipline? A) Yes, because a person who is engaged in full-time law practice must not conduct a related business from a single office. B) Yes, because a person who offers legal services along with real estate brokerage services must not engage in face-to-face solicitation of persons known to need real estate brokerage services. C) No, because the lawyer's real estate brokerage services are ancillary to her law practice, and the two operations are conducted from a single office. D) No, so long as her face-to-face pitch to the owners of apartment buildings is truthful and not misleading.

B) Yes, because a person who offers legal services along with real estate brokerage services must not engage in face-to-face solicitation of persons known to need real estate brokerage services.

A lawyer regularly represents a manufacturer of electric kitchen appliances. One morning the president of the manufacturing company called the lawyer and asked if the lawyer had seen the newspaper story about a woman who was electrocuted when she opened the door of her dishwasher. The company president stated that he believed the dishwasher was one that his company had manufactured. The company president also stated that he found some quality control records from that period which reflected that some dishwashers left the plant without proper testing. He continued that the records should have been shredded, but somehow had been overlooked, and said that he intended to send the records to the shredder immediately unless the lawyer told him that he could not. Must the lawyer advise the president to keep the records? A) Yes, unless the company has a clearly established policy of shredding quality control records after two years. B) Yes, because the records have potential evidentiary value if the company gets sued. C) No, because at this point there is no litigation pending against the company respecting this matter. D) No, unless it was certain that the company was the manufacturer of the dishwasher in question.

B) Yes, because the records have potential evidentiary value if the company gets sued.

A Hollywood movie producer was charged under a criminal statute for unfair trade practices, and now faces a civil claim under the same statute. The producer retains an attorney to represent him in both suits. The attorney is a nationally known defense attorney who has represented many famous people. Most recently, he defended a celebrity in a notorious murder case that held the country rapt for several weeks. The attorney explains to the producer that the representation is very complex and would take a majority of his time for several months. Given the attorney's steep hourly rate, the producer's legal fees would likely be around $1 million. The producer is short on cash and makes the following proposal: If the attorney will represent him in both the civil and criminal suits, the producer will produce a movie based on the attorney's most famous past cases, told from the attorney's viewpoint. The attorney would have complete creative control and would be entitled to all of the movie's profits, which could be anything from $0 to $100 million. The producer had his personal attorney draw up a proposal to this effect and submitted it to the attorney. Assuming that the attorney receives any consent necessary from his former clients who might be portrayed in the movie, is this proposed arrangement proper? A) Yes, but only if the payment from the movie profits is for the civil suit only. B) Yes, but only if the ultimate amount paid to the attorney is not excessive in light of the work done. C) No, because any amount over $1 million is clearly excessive, and this arrangement could be worth $100 million. D) No, because a lawyer must not acquire media rights to a story concerning the lawyer's representation of a client.

B) Yes, but only if the ultimate amount paid to the attorney is not excessive in light of the work done.

An attorney was representing the plaintiff at a bench trial of a civil action pending before a judge. Midway through the plaintiff's case-in-chief, the judge called the attorney into his chambers. The judge told the attorney that he thought the attorney's case was very weak, but that he could be mistaken because he was distracted by money troubles. The judge went on to say that if he could get a $50,000 loan, he would feel much better. The attorney responded that he would be happy to loan the judge $50,000 to help him out as a friend. Later that afternoon, a messenger delivered an envelope containing $50,000 in cash to the judge's chambers. No mention was made of a promissory note, a repayment date, or an interest rate. Two days later, the plaintiff settled his lawsuit so the judge never had to decide the case. Three months later, the judge repaid the $50,000 to the attorney, together with interest at the market rate. Is the attorney subject to criminal liability for lending the money to the judge? A) Yes, if it is proven that the judge intended to induce the attorney to make the loan in return for a decision in favor of the plaintiff. B) Yes, if it is proven that, in making the loan, the attorney intended to induce the judge to decide the case in favor of the plaintiff. C) No, because as the matter turned out, the judge never had to decide the plaintiff's case. D) No, because the judge repaid the loan with interest.

B) Yes, if it is proven that, in making the loan, the attorney intended to induce the judge to decide the case in favor of the plaintiff.

A client hired a lawyer to draft a will for him. The client willed his entire estate to a 43-year-old widow. The client told the lawyer in confidence that he was neither a relative nor a friend of the widow. The client explained that he felt a moral obligation to the widow because he had killed her husband, and he had never become a suspect or confessed his sin to anyone. One day after signing the will, the client committed suicide. In due course, all of the client's assets were distributed to the widow, and the probate court closed his estate and discharged his executor. The lawyer never told the widow or anyone else that the client had confessed to killing the widow's husband. Now, a few years later, an enthusiastic young prosecutor is charging an innocent man with murdering the widow's husband in the first degree with aggravating circumstances, and the prosecutor is seeking the death penalty. May the lawyer voluntarily tell the innocent man's defense counsel what his client told him in confidence about killing the widow's husband? A) Yes, the lawyer not only may, but he must, tell the defense counsel what the client told him. B) Yes, the lawyer may tell, but he would not be subject to discipline if he decides not to do so. C) No, the lawyer would be subject to discipline if he told defense counsel because the attorney-client privilege survives the death of the client. D) No, because the client's confidential confession to the lawyer would be inadmissible hearsay if offered against the prosecution in the murder trial.

B) Yes, the lawyer may tell, but he would not be subject to discipline if he decides not to do so.

What type of fee agreements are required to be in writing? A. Hourly fees only B. Contingent fees only C. Flat fees only D. All fees

B. A lawyer generally must, before or within a reasonable time after commencing a representation, communicate the basis or rate of the fee and the expenses for which the client will be responsible. Although a writing is preferable, it is generally not required, except in contingent fee agreements (which must be in writing and signed by the client).

A court orders Joe to disclose what Joe believes to be a privileged communication with his client. The client wants the information to remain private. What should Joe do next? A. Comply with the court's order B. Assert the attorney-client privilege

B. A lawyer may reveal a client's confidential information to the extent that he's required to do so by law or court order. However, absent informed consent of the client to do otherwise, the lawyer should assert on behalf of the client all nonfrivolous claims that the order isn't authorized by other law or that the information sought is protected against disclosure by the attorney-client privilege or other applicable law.

Talia represents Gene in his divorce. She has already prepared and submitted documents relating to Gene's finances based on information from Gene. Later in the case, Gene admits to Talia that he had been hiding significant assets in offshore accounts to prevent his wife from receiving a larger settlement. Talia encourages Gene to let her amend the financial documents, but he refuses. Talia withdraws and sends an e-mail to the wife's counsel disaffirming any financial documents she had sent on Gene's behalf. Is Talia subject to discipline? A. Yes B. No

B. A lawyer may reveal the client's confidential information to the extent necessary to prevent the client from committing a crime or fraud that is reasonably certain to result in substantial financial harm to someone, if the client is using or has used the lawyer's services in the matter. The same is true if the client has already acted, and the lawyer's disclosure can prevent or mitigate the consequent financial harm. Gene was hiding significant assets and using Talia's services to do so. Talia acted properly by disclosing information reasonably necessary to prevent substantial financial injury to the wife.

A lawyer engaging in a consensual sexual relationship with their client is subject to discipline: A. Regardless of the circumstances B. Unless the relationship existed prior to the representation C. Unless the client initiated the relationship D. Only if the relationship materially limits the representation

B. A lawyer must not have a sexual relationship with a client; the sole exception is when the lawyer and client had a consensual sexual relationship before the lawyer-client relationship began.

Rena wants Jacob to draft a will that leaves a large sum of money to Jacob. Under what conditions, if any, is this proper? A. None; this is always forbidden B. If Jacob and Rena are related to each other C. If Rena is also a lawyer D. If Rena understands the inherent conflict of interest and provides informed consent, confirmed in writing

B. A lawyer must not solicit a substantial gift from a client or prepare an instrument giving the lawyer, or the lawyer's relative, any substantial gift from a client except when the client is related to the donee.

A start-up company hires Larry to incorporate its business. The start-up is low on cash and proposes giving Larry stock in the company in exchange for his services. Is this proper? A. No, because it constitutes a proprietary interest in the representation B. Yes, as long as Larry meets the requirements for a business transaction with a client

B. A lawyer must not take a proprietary interest in a cause of action or in the subject matter of litigation the lawyer is conducting for a client. This is not a litigation matter; Larry is merely incorporating the start-up company. A lawyer may accept property, such as stocks, in lieu of money as long as the requirements for a business transaction are met.

Nick is a partner at a law firm. Three years ago, Nick's then-partner Beth worked on a variety of tax and business matters for Herbert. Only Beth and an associate Alan worked on Herbert's legal matters. Beth subsequently left the firm, and Herbert followed Beth to her new practice. Alan decided to stay with Nick's firm. Sara, Herbert's wife, approaches Nick and asks him to represent her in her divorce from Herbert. May Nick represent Sara without Herbert's consent? A. Yes B. No

B. A lawyer's former firm is prohibited from representing a person with interests materially adverse to those of a client of the formerly associated lawyer if: (i) the matter is the same or substantially related to that in which the formerly associated lawyer represented the client; and (ii) a lawyer remaining in the firm has material confidential information about the matter. The conflict can be waived with informed consent, confirmed in writing, from the firm's former client. Here, the matters are "substantially related" because Beth normally would have received confidential information about Herbert's finances while representing him, and such information could materially advance Sara's position in the divorce case. Because an associate who has information about Herbert's matters is still with the firm, Nick cannot represent Sara in the divorce case without Herbert's informed consent, confirmed in writing. (Note, however, that if no lawyer remaining in the firm had material confidential information about the tax and business matters, there would have been no conflict.)

Abby is licensed to practice in State A but received "pro hac vice" permission from State B to represent a client in a lawsuit in State B court. During the State B litigation, Abby does something that violates the rules of professional conduct of State B. However, the action is permitted under State A's rules. State B's choice of law rule follows the ABA model rule. Which state's rule will apply to Abby's conduct? A. State A, because that is where Abby is licensed B. State B, because that is where the litigation took place C. Each state can apply their own rule and discipline Abby

B. Although each state can discipline Abby, a lawyer's particular conduct is governed by only one set of disciplinary rules.

Alison e-mails her lawyer, Bill, about a sensitive matter. Alison then forwards the e-mail to a friend. The e-mail is protected by: A. The attorney-client privilege only B. Bill's duty of confidentiality only C. Neither the attorney-client privilege nor Bill's duty of confidentiality D. Both the attorney-client privilege and Bill's duty of confidentiality

B. By forwarding the e-mail to a third party, Allison waived her attorney-client privilege. However, the ethical duty of confidentiality applies to information relating to the representation even if some third parties know about it.

Caitlin and Rico are partners in a law firm. A plaintiff hires Caitlin to file a lawsuit against his neighbor. When the neighbor receives the complaint, she calls Rico and asks him to represent her in the lawsuit. Caitlin and Rico both believe they can competently and diligently represent their clients despite the conflict. They notify the clients of the situation and obtain their informed consent, confirmed in writing. Can Caitlin and Rico proceed with representing their clients in the lawsuit? A. Yes B. No

B. Either Caitlin or Rico has to get out of the case. Even with client consent, a lawyer must not represent a client if it would involve asserting a claim against another client represented by that lawyer in the same litigation (or other proceeding before a tribunal). In other words, the lawyer can't be on both sides of the same proceeding. Lawyers within a firm are treated as a single unit for conflicts purposes. Therefore, Caitlin and Rico can't be on opposite sides of the same litigation.

Ted is on trial for burglary. Ted's uncle has hired Ellen to represent Ted and is paying for Ted's legal fees. Ted's uncle wants to approve any major decisions with regard to Ted's defense. Ellen obtains Ted's informed consent to this arrangement. Is Ellen subject to discipline? A. Yes, because a lawyer must not represent a client if a third party is paying the client's legal fees B. Yes, because the uncle is seeking to interfere in the case C. No, because the uncle is Ted's family member D. No, because Ted gave informed consent to the arrangement

B. Ellen is subject to discipline for agreeing to represent Ted under these conditions. A lawyer may accept compensation from a third person for representing a client, as long as: (i) the client gives informed consent, (ii) the third person doesn't interfere with the lawyer's independence or the representation of the client, and (iii) the arrangement doesn't compromise the client's confidential information. By requiring his approval of all major decisions, Ted's uncle seeks to interfere with Ellen's independence in representing Tom, and this is unacceptable.

Lyle represents Rebecca and BigCorp in separate lawsuits. Rebecca wants Lyle to take a certain position on a legal issue in her case, and BigCorp wants Lyle to take the opposite position in their case. Which statement is most accurate regarding whether there is a conflict of interest? A. Taking inconsistent legal positions in separate cases always poses a conflict that must be addressed by getting informed consent, confirmed in writing, from each client B. Taking inconsistent legal positions in separate cases poses a conflict in certain circumstances, such as when a decision favoring one client will create a precedent that will seriously weaken the other client's position C. A lawyer must never take inconsistent legal positions in separate cases, even if the clients consent

B. Generally, a lawyer may take inconsistent legal positions in different tribunals at different times on behalf of different clients, and that on its own doesn't create a conflict of interest. However, a conflict exists if there is a significant risk that a lawyer's action on behalf of one client will materially limit the lawyer's representation of another client in a different case: for example, when a decision favoring one client will create a precedent likely to seriously weaken the position taken on behalf of the other client.

Arie, a tax attorney, meets with client Chloe to prepare her taxes and provide legal advice. Arie makes a serious error on the tax return, resulting in a costly audit. Could Arie be subject to discipline for incompetently completing Chloe's tax return? A. No, because preparing a tax return is a nonlegal service B. Yes, because the legal and nonlegal services were provided together

B. If a lawyer provides nonlegal services in circumstances that aren't distinct from the provision of legal services, then the rules of professional conduct apply to both the legal and nonlegal services.

Maria is working on a class action lawsuit against a pharmaceutical company. Which members of the class could Maria sue on behalf of another client in an unrelated matter without obtaining any type of consent? A. Both named and unnamed class members B. Unnamed class members only C. Neither named nor unnamed class members

B. In class action litigation, the unnamed members of a class ordinarily are not regarded as clients for purposes of the "direct adversity" conflicts rule. Therefore, Maria can sue an unnamed class member in an unrelated matter without obtaining their consent.

Becca is a criminal defense attorney who also owns a dog grooming business. On one occasion, Becca failed to supervise a new employee, and a dog was seriously injured. Is Becca subject to discipline? A. Yes, unless she had warned the dog's owner that the protections of the attorney-client relationship do not apply B. No, because grooming dogs is not a law-related service

B. Law-related services are services that might reasonably be performed in conjunction with (and are related to) the provision of legal services and that are not prohibited as unauthorized practice of law when provided by a nonlawyer. If a lawyer provides nonlegal services through an entity that isn't her law office but that she controls (either alone or with other lawyers), that lawyer must take reasonable steps to assure that people who receive the nonlegal services understand that those services aren't legal services and that the rules of professional conduct don't cover those services. If the lawyer doesn't take those reasonable steps, then the lawyer is subject to the rules of professional conduct with respect to the nonlegal services.

Lawyer Winnie receives an e-mail from a witness in a case. The e-mail contains details of the witness's private conversation with Winnie's client. The e-mail is protected by: A. The attorney-client privilege only B. Winnie's duty of confidentiality only C. Neither the attorney-client privilege nor Winnie's duty of confidentiality D. Both the attorney-client privilege and Winnie's duty of confidentiality

B. The attorney-client privilege applies only to communications between the lawyer and client (and their respective agents). It wouldn't cover this witness's conversation with Winnie's client. By contrast, the duty of confidentiality applies to all information relating to the representation, regardless of its source, so Winnie may be subject to discipline if she discloses the information.

Angela, a prospective client, visits John's office and says, "I want advice on how to embezzle money and get away with it." John advises Angela not to carry out her plan and they do not move forward with an attorney-client relationship. Angela embezzles money from her employer, and John is subpoenaed to testify at Angela's criminal trial. Is John's conversation with Angela privileged? A. Yes B. No

B. The attorney-client privilege applies to consultations with prospective clients, but the attorney-client privilege doesn't apply if the client is seeking the attorney's services to engage in or assist a future crime or fraud.

Holly is representing Deena in a murder case. Deena brings her journal to Holly's office and tells her it contains entries from the days before the murder. She asks Holly to keep it safe during the trial. Is the journal privileged? A. Yes B. No

B. The attorney-client privilege covers both oral and written communications, but the client can't protect a preexisting document or thing from discovery simply by turning it over to the attorney. If the document or thing would be discoverable in the client's hands, it's equally discoverable in the attorney's hands. Note, however, that their conversation about the journal would be privileged.

Michelle is an estate planning lawyer. She wants to ask her client Pierre to make her the executor of his estate, which promises to be a lucrative position. Michelle and Pierre have no family relationship. May Michelle proceed? A. No, because a lawyer must not solicit a substantial gift from a client unless they are related B. Yes, provided Michelle addresses any "material limitation" conflict based on her personal interest in obtaining the position

B. The conflicts rule for gifts from clients doesn't prohibit a lawyer from seeking to have herself or her colleague named as executor of an estate or counsel to the executor or to some other fee-paying position. There may be a general conflict, though, if the lawyer's interest in obtaining the appointment will materially limit the lawyer's judgment in advising the client concerning the choice of an executor or other fiduciary. In that situation, Michelle would need to get Pierre's informed consent, confirmed in writing, and explain her financial interest as well as the availability of alternative candidates for the position.

A lawyer MUST withdraw from a case when: (1) the client fires the lawyer; (2) the representation would require the lawyer to violate the law or the rules of professional conduct; or (3) __________________. A. The lawyer has a fundamental disagreement with the client's position B. The lawyer's physical or mental condition materially impairs their ability to represent the client C. The client previously used the lawyer's services to commit a crime or fraud D. The client persists in a course of action involving the lawyer's services that the lawyer reasonably believes is criminal or fraudulent

B. The other grounds listed are some of the permissive grounds for withdrawal.

Taylor and Scott are partners in different offices of the same firm. Taylor represents Books-R-Us in real estate matters. Knowing this, Scott accepts a representation of The Book-Bin, a direct competitor of Books-R-Us, in a copyright matter. None of Taylor's matters for Books-R-Us involve The Book-Bin, and Scott's matter for The Book-Bin does not involve Books-R-Us. Neither Scott nor Taylor obtain informed consent from their client.Are Scott and Taylor likely subject to discipline? A. No, because they work in different offices B. No, because the clients not legally adverse C. Yes, because the clients are economically adverse

B. The two clients are head-to-head adversaries in an economic sense, but their interests are not adverse in any legal sense—Taylor is not representing Books-R-Us against The Book-Bin, and Scott is not representing The Book-Bin against Books-R-Us. If Taylor and Scott can disclose the situation to their respective clients without revealing confidential information, they may do so for the sake of client goodwill, but they wouldn't be subject to discipline for failing to do so. Therefore, (B) is correct and (C) is wrong. (A) is wrong. If a lawyer does have a conflict, it is generally imputed to everyone in the firm, even to lawyers in different offices.

What is the most accurate definition of "confidential information" under the ethical duty of confidentiality? A. Any communication between the lawyer and the client B. All information relating to the representation C. All information relating to the representation that the attorney receives from the client D. All information relating to the representation that the client asked to keep private

B. Unlike the attorney-client privilege, the ethical duty of confidentiality protects more than confidential communications. It applies to any information relating to the representation, from any source. This is true regardless of whether the information is privileged, whether the client asked for it to be kept in confidence, and whether revealing it might harm or embarrass the client.

Henry hires Lola to represent him on a contingent fee basis. They agree that Lola will take 30% of any recovery. Midway through the case and after Lola had done hundreds of hours of work, Henry fires Lola without cause and gets another lawyer. What fee, if any, is Lola entitled to? A. None, because she assumed the risk of a contingent fee agreement B. The reasonable value of her services, once Henry prevails in the case C. The reasonable value of her services, immediately D. 30% of whatever Henry recovers

B. When a client hires a lawyer on a contingent fee basis and then fires the lawyer before the case is over, the lawyer is still entitled to quantum meruit recovery for the reasonable value of the work done before the firing. However, the lawyer's claim doesn't arise until the contingency comes to pass.

The attorney-client privilege applies to: A. Confidential communications between the attorney and anyone involved in the case B. Confidential communications between the attorney and client and their respective agents C. All information relating to the representation D. All information relating to the representation that would harm the client

B. The attorney-client privilege protects only confidential communications between the attorney and client (or the agents of either of them). The ethical duty, in contrast, covers not only confidential communications, but also any other information that the attorney obtains relating to the representation of the client, from any source.

Jin is a well-known patent lawyer with a busy schedule. Harold pays Jin $20,000 so that Jin will be available to represent him in an upcoming patent dispute. Subsequently, Harold and Jin enter into a "nonrefundable retainer agreement." In accordance with the agreement, Harold pays Jin $100,000 for legal fees up front, to be credited against Jin's hourly rate. Mid-way through the case, Jin has done $50,000 worth of work. Harold decides he wants a new lawyer, fires Jin, and demands a refund. What, if anything, is Jin required to give back to Harold? A. Nothing B. $20,000 C. $50,000 D. $70,000 E. $100,000 F. $120,000

C A lawyer may require their fee to be paid in advance, but they must refund any unearned part of the advance if they withdraw or are fired. Harold has made an advance payment of $100,000 for legal fees. Jin has done $50,000 of work, which means that $50,000 of the payment is unearned. This is the amount that Jin must refund to Harold. It is immaterial that their agreement was deemed "nonrefundable" and a "retainer"—the terminology is not controlling. A true retainer fee is something different—it is money paid solely to ensure the availability of the lawyer. A lawyer who is fired or withdraws generally need not refund a retainer fee. The $20,000 that was initially paid solely to ensure Jin's availability was a retainer fee, and Jin is not required to return it to Harold.

A state university receives 45% of its annual budget from the state. The other 55% of the budget comes from private sources. The university is chartered by the state constitution, and it is regarded for all purposes as a unit of the state government. The governing body of the university is its board of overseers, a group of 17 citizens. The chief executive officer of the university is the chancellor, and the chief legal officer is the general counsel. The university has always strived for a student body and faculty that are diverse in age, politics, wealth, race, nationality, religion, sex, and sexual orientation. One year ago, the voters passed a ballot initiative that prohibits all units of the state government, including the university, from considering a person's race when offering employment or admission to school. The initiative prohibits giving any state funds to a governmental unit that violates the initiative. With reluctance, the university board of overseers adopted a new university-wide regulation that requires all admissions officers and hiring committees to obey the initiative. The state supreme court sustained the constitutionality of the initiative, and the United States Supreme Court denied certiorari. A lawyer is one of 15 attorneys in the university general counsel's in-house law office. The general counsel assigned the lawyer to work with the university's admissions office to develop new admissions criteria that will comply with the initiative. At the outset, the lawyer reminded the admissions director that she was not his lawyer, but rather the university's lawyer. The admissions director told the lawyer that despite any new admissions criteria, he would continue to consider race because he believed that was the right thing to do. Deep in her heart, the lawyer agrees with the admissions director. Which of the following may the lawyer do in responding to this situation? A) Keep the admissions director's statement in confidence, even if she reasonably believes that the university is likely to lose its state funding as a consequence. B) Promptly disclose the admissions director's statement to the state attorney general, who is the official in charge of enforcing the voter initiative. C) Attempt to convince the admissions director to obey the voter initiative, and if he refuses, then disclose the situation to the university's general counsel. D) Anonymously leak the admissions director's statement to the university's board of overseers.

C) Attempt to convince the admissions director to obey the voter initiative, and if he refuses, then disclose the situation to the university's general counsel.

A new associate at a law firm was asked to help a partner advise a state university on how to comply with a federal statute that requires colleges and universities to make many changes in their facilities to accommodate students with disabilities. After graduating from law school, the associate had worked on the congressional staff of a United States senator. In that role, she personally drafted a bill that was ultimately enacted as the federal statute. In light of the associate's earlier role as the drafter of the federal statute, which of the two lawyers may work on the matter? A) Neither the partner nor the associate. B) The partner only, and only if the associate is properly screened off from the matter. C) Both the partner and the associate. D) The partner only, and only if the state university consents after full disclosure.

C) Both the partner and the associate.

The attorney general's office does not include any lawyers who are skilled in the field of condemnation law (the law of eminent domain). Consequently, whenever the state wants to use its power of eminent domain to condemn some private property for a public use, the attorney general must hire a private law firm to represent the state in the condemnation proceedings. In contrast to the paltry fees that the state pays to appointed defense counsel in criminal cases, the attorney general pays quite handsomely for condemnation work. The attorney general is a partisan political position that is filled by a contested election every four years. A large state law firm limits its practice to condemnation law. The founding partner is an 87-year-old multimillionaire who remains active on the firm's management committee. When it is time to elect a new attorney general, the partner makes large donations from his personal wealth to each candidate who has any reasonable chance of becoming the next attorney general. The other members of the firm's management committee know about the partner's contributions, and they have formally and informally expressed the firm's thanks for helping the firm obtain future appointments by the attorney general. May the firm accept an appointment from the new attorney general to represent the state in a condemnation case? A) Yes, because the partner makes his contributions from his personal wealth, and he has a constitutional right to participate personally in the political process. B) Yes, because the partner's personal political contributions cannot be imputed to the law firm. C) No, because a lawyer or law firm must not accept appointed legal work from a governmental official after making a political contribution for the purpose of obtaining such work. D) No, because to accept such an appointment would create an appearance of impropriety in light of the partner's political contributions.

C) No, because a lawyer or law firm must not accept appointed legal work from a governmental official after making a political contribution for the purpose of obtaining such work.

A lawyer practices environmental law. He also happens to be one of the nation's leading experts on the environmental effects of filling wetlands. The state legislature has scheduled hearings on a bill to prohibit the filling of wetlands surrounding a bay. One of the lawyer's regular clients is a development company, which owns development rights to some of the wetlands in question. The development company wants to fill its wetlands so that it can build low-cost housing for underprivileged families. The development company hired the lawyer to appear as a witness at the legislative hearings and to testify in opposition to the ban on wetland filling. The lawyer appeared as a witness, identified himself as an expert on wetlands, and testified vigorously against the proposed legislation. Was the lawyer's conduct proper? A) Yes, unless his testimony was contrary to his own beliefs about the environmental effects of filling wetlands. B) Yes, because he is a leading expert on the environmental effects of filling wetlands. C) No, unless he informed the legislators that he was appearing in a representative capacity. D) No, because a lawyer must not be a witness for his client on a contested matter.

C) No, unless he informed the legislators that he was appearing in a representative capacity.

An attorney in solo practice published a brochure regarding what one should do when injured. The brochure contains accurate, helpful information about obtaining proper medical treatment, recording details of the accident, notifying insurance companies, not making harmful statements, and the like. The attorney's name, address, and telephone number are printed on the brochure's cover. One afternoon, the attorney saw a pedestrian knocked down in a crosswalk by a hit-and-run driver. He and another bystander called 911 and gave the pedestrian emergency first aid until an ambulance arrived. The next day, the attorney visited the pedestrian in the hospital and gave the pedestrian a copy of his brochure. Which of the following is correct? A) The attorney is subject to discipline, both for publishing the brochure and for giving the brochure to the pedestrian in the hospital. B) The attorney is subject to discipline for publishing the brochure. C) The attorney is subject to discipline for giving the pedestrian a copy of the brochure at the hospital. D) The attorney's conduct was proper because the brochure's contents are neither false nor misleading.

C) The attorney is subject to discipline for giving the pedestrian a copy of the brochure at the hospital.

Two sisters are partners in a bakery. Their partnership agreement says that they will share the work and the profits equally. They are very close, but they constantly bicker—each claims that the other is taking an unfair share of the profits and shirking on the work. Six months ago, they hired a lawyer to act as a third-party neutral, to help them resolve their differences once and for all. At the outset, the lawyer explained that he would be strictly neutral between them; he would not be representing either one, and neither of them would be entitled to the protections afforded by an attorney-client relationship. After a long series of meetings with them (sometimes separately, sometimes jointly), the lawyer proposed a solution. The sisters liked his solution, reduced it to writing, and signed it, vowing to end their bickering forever. Six months later, the feud erupted again, worse than ever. One of the sisters asked the lawyer's law firm to represent her in a lawsuit against her partner-sister, seeking to declare the partnership at an end and to bar her partner-sister from entering the bakery premises. Which of the following is correct? A) The lawyer is subject to discipline for his failed effort to serve both sisters when their interests were patently in conflict. B) It would be proper for the lawyer to represent the sister in the lawsuit as she requested, even without the informed consent of her partner-sister. C) The lawyer's law firm partner may represent the sister in the lawsuit as she requested, but only if her partner-sister is notified in writing, and only if the lawyer is timely screened and does not share in the fee earned in the lawsuit. D) The lawyer's law firm partner would be subject to discipline for representing the sister in the lawsuit as she requested, even if the lawyer is timely screened and does not share in the fee earned in the lawsuit.

C) The lawyer's law firm partner may represent the sister in the lawsuit as she requested, but only if her partner-sister is notified in writing, and only if the lawyer is timely screened and does not share in the fee earned in the lawsuit.

A law professor was selected as the neutral arbitrator of a boundary line dispute between an elderly couple and the couple's next-door neighbors. The law professor decided the matter in favor of the elderly couple. Shortly thereafter, the law professor quit his teaching position and entered private law practice. The elderly couple's next-door neighbors brought suit to have the arbitration award set aside. The elderly couple asked the law professor to represent them in the suit. If the law professor takes the case, will he be subject to discipline? A) No, because serving as the elderly couple's lawyer is consistent with his decision as arbitrator in their favor. B) No, because by seeking to hire the law professor, the elderly couple is deemed to have consented to the conflict of interest. C) Yes, because his earlier service as neutral arbitrator creates a conflict of interest. D) Yes, because there is reasonable ground to doubt his impartiality in the case.

C) Yes, because his earlier service as neutral arbitrator creates a conflict of interest.

An attorney and a licensed real estate developer, a nonlawyer, created a partnership to serve people who want to invest in commercial real estate. The real estate developer finds promising commercial real estate projects, brings together groups of investors, and works with local planning authorities to gain approval for the projects. The attorney drafts the legal documents for the projects, assists the investors with the legal technicalities, advises the investors on their tax liabilities, and does whatever legal work the investors need in connection with management and operation of the projects. The attorney and the real estate developer charge the investors a single fee for their work, and they divide the partnership profits 50%-50%. Is the attorney subject to discipline? A) No, provided the investors give informed consent to the potential conflicts of interest, and such consent is confirmed in writing. B) No, because the real estate developer does only development work, and the attorney does only legal work. C) Yes, because the attorney and the real estate developer are partners in the business. D) Yes, because she is aiding the real estate developer in the unauthorized practice of law.

C) Yes, because the attorney and the real estate developer are partners in the business.

The state bar association has established a peer counseling program whereby lawyers who are addicted to alcohol or other drugs can receive confidential counseling from other lawyers. The bar association's ethics rule on confidential information provides that communications between the counselor lawyer and the counseled lawyer are to be treated just like confidential communications between an attorney and client. A lawyer is addicted to alcohol and is receiving peer counseling under the program from another lawyer. The lawyer is a large, strong man, and his addiction has made him subject to periodic fits of physical violence. This afternoon, during their peer counseling session, the lawyer told his peer counselor that his client had refused to pay the fees he owes, and that he intended to punch out the client the next time he got roaring drunk. From working with the lawyer over an extended period, the peer counselor believes that he may really do it. May the peer counselor disclose the lawyer's statement to the client and the police? A) No, unless the lawyer consents. B) No, unless the peer counselor is certain that the lawyer will carry out his threat. C) Yes, even if the lawyer objects. D) Yes, because he is serving as a peer counselor, not a lawyer.

C) Yes, even if the lawyer objects.

An attorney is a voting member of the legislation committee of a consumer-based law reform group that drafts and advocates the passage of proposed statutes on food safety. The law reform group is currently debating a draft statute that sets quality and safety standards for growth hormones administered to chickens, turkeys, and other poultry. The attorney is also engaged in the private practice of patent law. She regularly represents a biotechnology firm. Using the techniques of genetic engineering, the biotechnology firm invents, develops, and sells a variety of patented growth hormones. The attorney herself has obtained patents on some of these hormones for the biotechnology firm. If enacted into law, the law reform group's proposed statute on poultry hormones could materially increase the biotechnology firm's hormone sales because it is the only firm whose hormones would meet the statute's quality and safety requirements. Would it be proper for the attorney, as a member of the law reform group's legislation committee, to participate in the debate on, and to cast her vote on, the proposed statute? A) No, because the statute could materially benefit the biotechnology firm. B) No, because the attorney may not serve as a member of the law reform group while representing the biotechnology firm. C) Yes, provided that she informs the legislation committee that she represents an unnamed client whose interests could be materially benefited by the statute. D) Yes, provided that she informs the legislation committee that she represents the biotechnology firm, whose interests could be materially benefited by the statute.

C) Yes, provided that she informs the legislation committee that she represents an unnamed client whose interests could be materially benefited by the statute.

A swimming coach was charged with assault of another coach. The swimming coach hired a criminal attorney to defend him. Subsequently, the swimming coach pleaded not guilty and was released on his own recognizance. At his first trial, a jury was empaneled, and the prosecutor was almost finished presenting the testimony of her first witness when a signal from her electronic pager interrupted her. The trial judge granted her request for a short recess, at the end of which the prosecutor told the judge that her office had instructed her not to proceed with this case at this time. The judge responded that if the prosecutor stopped now, the defendant would go free. When the prosecutor indicated that she understood, the judge entered a judgment of acquittal and set the swimming coach free. Twenty days later, the prosecutor recharged the swimming coach with the same offense. The swimming coach hired his original criminal attorney to defend him. The same judge presided over the second trial. The swimming coach's attorney made no pretrial motions. This time the prosecutor did not falter, and in due course the jury at the second trial found the swimming coach guilty as charged. The judge sentenced him to prison for the period required by law, but she stayed the sentence and released him on his own recognizance pending appeal. The swimming coach reluctantly paid the criminal attorney's bill for the second trial—$5,000. However, the swimming coach hired a new lawyer for the appeal, and in due course the appellate court reversed the conviction and set aside the prison sentence. The appellate court's opinion stated it had never seen a clearer double jeopardy violation. Will the swimming coach's original criminal attorney be subject to civil liability in a legal malpractice action brought by the swimming coach for having missed the double jeopardy issue? A) No, because the swimming coach never served jail time as a result of the original attorney's error. B) No, even if the swimming coach proves by a preponderance of evidence that he did not commit the assault on the opposing coach. C) Yes, provided that the swimming coach proves by a preponderance of evidence that he did not commit the assault on the opposing coach. D) Yes, but the swimming coach can recover only nominal damages.

C) Yes, provided that the swimming coach proves by a preponderance of evidence that he did not commit the assault on the opposing coach.

Which of the following survive the client's death? A. The attorney-client privilege only B. The duty of confidentiality only C. Both the attorney-client privilege and the duty of confidentiality D. Neither the attorney-client privilege nor the duty of confidentiality

C.

For which action would a lawyer be subject to discipline? A. A client admits to her lawyer that she plans to rob a store. She asks about the different penalties for armed robbery versus unarmed robbery. The lawyer gives the client accurate information on the sentencing guidelines. B. A client comes to a lawyer for advice regarding a zoning statute that is probably unconstitutional, but the client doesn't have legal standing to challenge it. The lawyer advises the client that one way to obtain standing is to disobey the statute. C. A client instructs the lawyer to take a course of action that seems to constitute money laundering, although the client does not seem aware of this. The lawyer says no, and when the client asks why not, the lawyer refuses to discuss the matter further and tells the client to get another attorney.

C. (A) is wrong. A lawyer must not advise a client to engage in conduct that the lawyer knows is criminal or fraudulent, or assist the client in such conduct. However, the lawyer may discuss the legal consequences of any proposed course of conduct. So, the lawyer here wouldn't be subject to discipline for giving information about the sentencing guidelines for different crimes. (B) is wrong. A lawyer may also counsel or assist a client to make a good faith effort to determine the validity, scope, meaning, or application of the law. One way to do this is by violating a statute to test its validity or scope in an enforcement proceeding. Thus, the lawyer here wouldn't be subject to discipline for telling the client that one way to get legal standing is to disobey the statute. (C) is correct. If the client expects the lawyer to do something that is illegal or unethical, the lawyer must consult with the client and explain why he can't do what the client wants. The lawyer should have explained that the action amounted to money laundering and he was unable to assist the client in criminal or fraudulent conduct.

A lender and a borrower want Ashley to represent them in a loan transaction. Ashley reasonably believes that she can competently and diligently represent them both. She obtains each client's informed consent on the telephone, and then promptly sends a confirmatory e-mail. She then proceeds with the representation. Is Ashley subject to discipline? A. Yes, because she did not obtain the clients' consent in a signed writing B. Yes, because a lawyer must never be on opposite sides of the same matter C. No, because she properly handled the conflict

C. A conflict exists because Ashley will be representing two directly adverse clients in the same matter. A lawyer must never be on opposite sides of the same litigation (or other proceeding pending before a tribunal), but this is a transactional matter, so (B) is wrong. If there is a conflict between current clients, a lawyer may undertake the representation if she reasonably believes that she can competently and diligently represent each affected client despite the conflict, and each client gives informed consent, confirmed in writing. The clients' signatures are not required. Therefore, (C) is correct and (A) is wrong.

Bennie asks Ayanna to represent him in his contentious divorce from Janet. Ayanna represented Janet a few years ago in various tax matters relating to her business and personal finances. May Ayanna accept the representation of Bennie without Janet's consent? A. Yes, because Ayanna does not currently represent Janet B. Yes, because the matters are unrelated C. No, because the matters are substantially related D. No, because a lawyer must not oppose a former client

C. A lawyer must not represent one client whose interests are materially adverse to those of a former client in a matter that is substantially related to a matter in which the lawyer represented the former client, unless the former client gives informed consent, confirmed in writing. Matters are "substantially related" if: (i) they involve the same transaction or legal dispute, or (ii) there is a substantial risk that confidential factual information as would normally have been obtained in the prior representation would materially advance the new client's position.

Kai is a new associate working on a personal injury case. Kai's supervisor, Leah, tells Kai to send an important eyewitness an "attendance fee" for testifying at trial. The rules of professional conduct in the jurisdiction forbid compensating a non-expert witness for their testimony, but Kai is not aware of this. Kai makes the payment to the witness. Is Kai subject to discipline? A. No, because making the payment was a reasonable resolution of an arguable ethics question B. No, because Kai was acting on orders from Leah C. Yes, because making the payment violated the rules of professional conduct

C. A subordinate lawyer doesn't violate the rules of professional conduct by acting in accordance with a supervisor's reasonable resolution of an arguable question of professional duty. Here, however, there was no arguable question of professional duty—the rules of professional conduct state that the payment was forbidden. It's immaterial that Kai didn't know about the rule. Orders from a supervisory lawyer are no excuse for clearly unethical conduct—a lawyer has to follow the ethics rules even when acting under the direction of another person.

Bernie, a college professor accused of inappropriate conduct, hires Amber to represent him in front of his university's disciplinary board. Bernie proposes to Amber that if he keeps his job, he will pay her $5,000. If he loses his job, he will pay her $500. Amber agrees. Amber memorializes the agreement in writing and sends it to Bernie, but he never returns it. The representation is successful, and Bernie keeps his job. Is Amber subject to discipline? A. No, because Amber's fee was not based on any monetary recovery B. No, because there was a written agreement C. Yes, because Bernie did not sign the agreement D. Yes, because a contingent fee is prohibited in this type of case

C. All contingent fees must be in writing, contain various disclosures, and be signed by the client.

Which of the following apply to discussions with someone who comes in to talk about hiring the lawyer but decides not to do so? A. The attorney-client privilege only B. The duty of confidentiality only C. Both the attorney-client privilege and the duty of confidentiality D. Neither the attorney-client privilege nor the duty of confidentiality

C. For purposes of the attorney-client privilege, a "client" means a person that seeks legal services from an attorney. The privilege covers preliminary communications leading up to an attorney-client relationship, even if no such relationship develops. Similarly, the ethical duty of confidentiality applies to information gained during consultations with prospective clients.

The requirements for business transactions with clients (e.g., must be fair and reasonable to client; must obtain informed consent in a signed writing) have to be met in all of the following situations, EXCEPT: A. A lawyer accepts non-monetary property as a legal fee B. A lawyer who owns a separate title insurance business sells title insurance to the client C. A lawyer who represents a bank takes out a standard home loan from that bank D. A lawyer buys a client's boat E. A lawyer leases their second home to a client

C. The rule for business transactions and adverse financial interests does not apply to: (i) an ordinary fee agreement between a lawyer and client, or (ii) standard commercial transactions in which the lawyer buys goods or services that the client routinely markets to the public. If a lawyer takes out a standard home loan from a bank that is also a client, it would be considered a standard commercial transaction. The lawyer would have no advantage in this situation, so the usual requirements for business transactions don't need to be met. The other choices are subject to the normal requirements.

Bea's client, Sally, suffers a traumatic brain injury. Bea learns that Sally's son, Ivan, is planning to have Sally sign over her house to him. Sally previously told Bea that Ivan has a severe gambling problem and she did not want him to have any control over her affairs in the future. When Bea visits Sally at a rehabilitation center, Sally tells her to let Ivan do what he wants. However, it is clear to Bea that Sally is unaware of her surroundings and unable to make an informed decision. What can Bea do in this situation? A. Bea must do nothing, because Sally told her not to B. Bea can take protective action, but she must not disclose any confidential information about Sally's relationship with Ivan, Sally's condition, or anything else C. Bea can take protective action, and she can disclose confidential information to the extent reasonably necessary to protect Sally's interests

C. When the client has diminished capacity and faces a risk of substantial physical, financial, or other harm, the lawyer may take reasonable actions to protect the client. These actions include consulting with people or entities that can protect the client, and, when appropriate, seeking the appointment of a guardian or similar surrogate. When taking protective action, the lawyer has implied authority to reveal the client's confidential information, but only to the extent reasonably necessary to protect the client.

Janae is representing Dave in a lawsuit against his employer. Midway through the case, Dave confesses to Janae that the claim is completely baseless, and that he only filed it to annoy his employer. Janae correctly tells Dave that continuing with a frivolous claim would violate the rules of professional conduct, so she will have to file a motion to withdraw. Dave says he understands, and that he'll find a new lawyer. The judge denies Janae's motion and orders her to continue. Janae withdraws anyway. Is Janae subject to discipline? A. No, because she was required to withdraw from the case B. No, because Dave consented to the withdrawal C. Yes, because the court ordered her to continue the representation

C. Withdrawal is mandatory when continued representation would require the lawyer to violate the law or rules of professional conduct, and here, continuing to represent Dave in a frivolous claim would violate the rules of professional conduct. However, when an attorney seeks to withdraw from a case and the court denies the necessary permission, the attorney has to continue the representation. This is true even if the lawyer attempted to withdraw on a mandatory ground, as was the case here.

A client hired a lawyer to do the legal work in connection with a complex public securities offering. The lawyer agreed to do the work for a set hourly fee. The lawyer did a great deal of legal research, prepared numerous memoranda of fact and law, and drafted most of the documents needed for the public offering. At that point, the client became angry with the lawyer for no apparent reason and fired him. The client paid the lawyer at the agreed rate for the work the lawyer had done and demanded that the lawyer turn over to him the papers that the lawyer had prepared, including the legal and fact memoranda and the document drafts. What papers must the lawyer turn over to the client? A) Only the document drafts, but not the legal and fact memoranda. B) Only the legal and fact memoranda, but not the document drafts. C) None of the papers, because the client fired the lawyer. D) All of the papers, even though the client fired the lawyer.

D) All of the papers, even though the client fired the lawyer.

A building contractor and his lawyer met with a landowner to negotiate a contract for construction of an office building on the landowner's property. The contractor, the lawyer, and the landowner were the only persons present at the meeting. Ultimately, the three of them worked out a written agreement, and the contractor commenced work. However, it soon became apparent that the building site required far more preparation work than the contractor had contemplated when he agreed to the contract price. The contractor and the landowner argued about who had to pay for the additional site preparation. One important issue is whether the landowner made certain oral representations to the contractor during the contract negotiating session that the lawyer attended. The contractor contends that the landowner did make the representations, while the landowner contends that he did not. The lawyer was present during the entire negotiating session, and she is virtually certain that the landowner did not make the representations. The contractor refused to proceed with construction until the landowner paid for the extra site preparation. The landowner then sued the contractor for specific performance of the construction contract. The contractor asked the lawyer to represent him as trial counsel. The lawyer should: A) Agree to serve as trial counsel for the contractor because the contractor is entitled to the counsel of his choice. B) Agree to serve as trial counsel for the contractor because she can refuse to testify if she is called as a witness by the landowner. C) Decline to serve as trial counsel for the contractor because a lawyer is not allowed to testify in a manner that is prejudicial to her client. D) Decline to serve as trial counsel for the contractor because she can foresee that she will be called as a witness.

D) Decline to serve as trial counsel for the contractor because she can foresee that she will be called as a witness.

A police officer was charged with murder. He is alleged to have savagely beaten and ultimately killed a teenage gang member in the course of an arrest. Neither the police department nor the officer's union was willing to provide legal counsel for his defense, and the officer himself lacked funds to hire private counsel. The public defender's office could not represent him due to a conflict of interest from a related case. The trial court therefore appointed a lawyer to defend the officer. The lawyer is only three years out of law school. The lawyer practices criminal defense, but he has never handled a murder case before. For which of the following reasons may the lawyer decline the court appointment? A) Based on what he has read in the newspapers, he sincerely believes that the officer is guilty. B) He has no experience in the defense of a murder case. C) He is of the same race as the teenage victim, and he is in sympathy with the plight of young gang members. D) He recently was diagnosed with severe depression, which is affecting his ability to handle his existing caseload.

D) He recently was diagnosed with severe depression, which is affecting his ability to handle his existing caseload.

A lawyer agreed to represent a wife on an hourly fee basis in securing a divorce from her husband. The husband is also represented by an attorney. Despite repeated warnings by her attorney, the wife kept pestering her attorney with telephone calls and office visits concerning inconsequential details and trifling personal complaints. When the wife was unable to contact her own attorney on the phone or in person, she would telephone her husband's attorney, and try to put her questions and complaints to him. The husband's attorney always refused to talk to his client's wife. The wife's attorney repeatedly told her not to contact her husband's attorney, but to no avail. Finally, the wife's attorney told the wife that she would withdraw unless the wife changed her ways, but the wife did not do so. The wife's attorney withdrew and sent the wife a fee bill for the total number of hours she had spent on the case. The wife refused to pay the bill, and after futile efforts to settle the matter, the wife's attorney sued her to collect the fee, and stated that she would be holding the file until her bill was paid. Which of the following propositions is not true? A) It was proper for the wife's attorney to withdraw. B) It was proper for the husband's attorney to refuse to talk with the wife on the phone. C) It was proper for the wife's attorney to bill the wife for the total amount of time she spent on the case. D) It was proper for the wife's attorney to hold the wife's file until paid.

D) It was proper for the wife's attorney to hold the wife's file until paid.

A steel company merged with an iron corporation. The state attorney general sued the steel company and the iron corporation in federal court to enjoin the merger, alleging that it was in violation of the federal antitrust laws. The federal district judge enjoined the merger, and the steel company appealed the judge's decision. The steel company's lawyer, doing the legal research for the appeal, found a recent merger decision rendered by the Federal Trade Commission ("FTC") that is directly adverse to the steel company's position. FTC decisions do not control in the United States Courts of Appeal, but they are persuasive. The attorney general failed to cite the FTC decision. Must the steel company's attorney disclose it to the court? A) Yes, because it is persuasive authority. B) Yes, because the FTC decision is directly adverse to the steel company's position. C) No, because an attorney has no obligation to volunteer facts harmful to his client's case. D) No, because the court of appeals is not obliged to follow the FTC ruling.

D) No, because the court of appeals is not obliged to follow the FTC ruling.

An attorney has organized his law practice as a professional corporation. The attorney is the sole shareholder. The sign on the office door states: "Professional Corporation— Attorney at Law Corporate and Business Law, Torts and Domestic Relations" The attorney has one lawyer-employee, who was admitted to practice two years ago. The attorney pays his employee a modest monthly salary plus 60% of the fees collected in cases that the employee handles by herself. The attorney has a general business practice and is not a certified specialist in any practice area. When a client needs representation in a tort or domestic relations matter, the attorney turns the case over to his lawyer-employee. When the attorney turns a case over to the employee, he provides general guidance and is available to answer any questions she may have, but he does not supervise every step she takes. Is the attorney subject to discipline? A) Yes, because he splits fees with his employee in matters she handles by herself. B) Yes, because he does not closely supervise the work done by his employee. C) No, but he should change his sign to show the fields of practice that he personally handles. D) No, because the employee is a lawyer-employee of the attorney.

D) No, because the employee is a lawyer-employee of the attorney.

An attorney represented a landlord in a dispute with her longtime tenant, who had recently decided not to renew his lease. The landlord wanted to retain the security deposit to pay for extensive damage to the carpeting, while the tenant insisted that the damage was normal wear and tear. The attorney and the tenant's lawyer negotiated for days, but neither party would settle for less than two-thirds of the security deposit. Finally, the landlord telephoned the attorney and said: "The tenant asked if I want to talk about the security deposit. We've known each other for years, and I think we might have better luck if we work things out ourselves." The attorney encouraged the landlord to talk with the tenant if she thought it would help, but advised her not to finalize any agreement until both parties could consult with their respective counsel. The landlord and tenant had a productive discussion. The following day, the tenant's lawyer called the attorney and said the tenant would accept one-half of the security deposit. The attorney communicated the offer to the landlord, who agreed and returned the funds to the tenant. Is the attorney subject to discipline? A) Yes, because the attorney encouraged the landlord to speak to the tenant without obtaining the consent of the tenant's lawyer. B) Yes, because the attorney encouraged the landlord to speak to the tenant without notifying the tenant's lawyer. C) No, because the tenant initiated the conversation with the landlord. D) No, because the landlord and tenant spoke to each other directly.

D) No, because the landlord and tenant spoke to each other directly.

A lawyer regularly represented an older client in matters relating to the investment of the client's considerable wealth. The client told the lawyer that he wanted to put $500,000 into a sound, income-producing investment. The lawyer suggested that the two of them pool their money and talent and buy an apartment house. The lawyer would put up $75,000 and do the legal work, and the client would put up $500,000 and serve as the live-in manager of the apartment house. The client enthusiastically agreed to the arrangement and told the lawyer to draw up the papers. The lawyer drafted an agreement between himself and the client, negotiated the purchase of the apartment house, and drafted a deed from the seller to himself and the client as joint tenants with right of survivorship. The lawyer gave the client a carefully written explanation of the terms of the transaction, but he forgot to explain the significance of the joint tenancy, i.e., that upon the death of one joint tenant, the property would pass automatically to the other joint tenant. The lawyer advised the client, in the writing explaining the terms of the transaction, to have an outside lawyer look over the transaction, and he also urged him orally to do so. However, the client said that he trusted the lawyer and signed all of the papers without further ado. The lawyer and the client operated the apartment house successfully for several years, until the client died. The executor of the client's estate sued the lawyer to have the apartment house declared part of the client's estate, but the court concluded that the joint tenancy created a gift to the lawyer, effective on the client's death. Were the lawyer's actions proper? A) Yes, because the court concluded that the joint tenancy created a gift from the client to the lawyer. B) Yes, because the lawyer might have died first, thus bestowing a gift on the client. C) No, because the lawyer entered into a business transaction with the client. D) No, because the lawyer drafted the deed that bestowed a substantial gift on himself.

D) No, because the lawyer drafted the deed that bestowed a substantial gift on himself.

A man alleges that a very wealthy actor punched him in the face. He contacted a lawyer about representing him in a civil action against the actor. After several lengthy discussions with the lawyer about the merits of the case, the man decided to employ another attorney instead. The actor was later charged with criminal assault in connection with this incident, and the trial was televised. As the lawyer was watching the trial, she was astonished when the man testified to facts that the lawyer knew from their previous discussions to be false. The lawyer sent a letter with a messenger over to the court to notify the court that the man had perjured himself. Were the lawyer's actions proper? A) Yes, because her actions were necessary to prevent the man from perpetrating a fraud on the court. B) Yes, because the man committed a criminal act by testifying falsely. C) No, unless she sent copies of the letter to the prosecution and defense attorneys and they are given an opportunity to respond. D) No, because the lawyer's information was gained during her discussions with the man.

D) No, because the lawyer's information was gained during her discussions with the man.

A patent attorney focuses her practice on patents that involve genetically engineered medicines. Representatives of a bioengineering firm had a preliminary conversation with the attorney about representing the bioengineering firm in a patent infringement action against a pharmaceutical corporation. The attorney had never represented either company previously. The bioengineering firm's representatives talked to the attorney for more than an hour about the bioengineering firm's patent and about the pharmaceutical corporation's supposedly infringing product. This conversation covered only public information, nothing confidential. The bioengineering firm's representatives detected a distinct lack of enthusiasm from the attorney, and they ended the conversation cordially but without hiring her. In due course, the bioengineering firm hired a different patent attorney and sued the pharmaceutical corporation for patent infringement. The pharmaceutical corporation hired the attorney as defense counsel in the infringement case. The bioengineering firm's attorney promptly made a motion in the trial court to disqualify the attorney because of her earlier conversation with the bioengineering firm's representatives. Is the attorney subject to disqualification? A) Yes, because the bioengineering firm had previously consulted the attorney on the same matter. B) Yes, because the infringement suit is substantially related to the earlier conversation between the attorney and the bioengineering firm's representatives. C) No, because the bioengineering firm was never the attorney's client. D) No, because the prior conversation between the attorney and the bioengineering firm's representatives did not involve confidential information.

D) No, because the prior conversation between the attorney and the bioengineering firm's representatives did not involve confidential information.

For the past five years, an attorney has represented an art dealer in the sale of many valuable paintings. One of the major transactions occurred three years ago, when the art dealer sold a landscape purportedly painted by Vincent van Gogh to an art museum for $23 million. The museum subsequently resold the painting for $35 million. Now the art dealer has asked the attorney to do the legal work in connection with the sale of another landscape, also a purported van Gogh. The proposed purchase price is $12 million, and the prospective purchaser is a wealthy television personality who knows nothing about art. During a confidential conversation in the attorney's office, the attorney asked the art dealer if he had appraisal letters certifying the painting as a genuine van Gogh. The art dealer replied that he indeed had letters—letters he had forged himself—and that he had also forged the letters for the purported van Gogh sold to the art museum. When the attorney inquired further, the art dealer told him in confidence that both of the purported van Gogh paintings were in fact counterfeits created by a clever art student. Which of the following must the attorney do at this point? A) Report the art dealer to the law enforcement authorities. B) Warn the prospective purchaser about the proposed sale. C) Inform the art museum of the truth about the first painting. D) Refuse to represent the art dealer in the present transaction.

D) Refuse to represent the art dealer in the present transaction.

A law student is applying for admission to the State A Bar. When the law student was in high school, he and his parents lived in State B. His next door neighbor was an attorney admitted to practice in State B, but not in State A. The attorney knew that during the law student's senior year in high school, he was convicted of burglarizing a liquor store. After serving his sentence, the law student went to college and later to law school. The attorney has had no contact with the law student since his high school years, and as far as she knows, the law student has not done anything since high school that would reflect badly on his character. The Bar of State A sent the attorney a routine questionnaire, asking a series of questions about the law student's character. The attorney does not know whether the law student disclosed the burglary conviction on his bar application, and she does not know where to contact him to find out. Which of the following would be a proper response to the questionnaire? A) She should not respond at all because she has no relevant information to provide. B) She should not respond at all because as a State B lawyer she is not obligated to provide information to the Bar of State A. C) She should not mention the law student's burglary conviction in her response unless she first contacts him and obtains his permission to do so. D) She should state what she knows about the law student, including mention of his burglary conviction.

D) She should state what she knows about the law student, including mention of his burglary conviction

A plaintiff brought a civil action to recover damages for personal injuries he suffered as the victim of alleged police brutality inflicted by three defendant police officers. The trial was widely reported by the media. The jury returned a verdict in favor of the plaintiff and against the three police officers for $500 million. When the trial judge received the verdict, he was shocked by the size of the award. Before dismissing the jurors, the judge told the jurors that when they were sworn in, they had promised that they would deliver a verdict based on the evidence and that they would not be swayed by passion or prejudice. The judge further admonished the jurors that they had failed in those duties, that they had made a mockery of justice, and that they should be ashamed of themselves. He then dismissed the jury, and the defense lawyers renewed their motion for judgment as a matter of law and, alternatively, moved for a new trial. The judge announced that he would rule on the motions the following Monday at 10 a.m. in open court. The press reports of the verdict and the judge's comments to the jury created a great public tumult in the city where the case was tried. On the following Monday, the courtroom was jammed with reporters. Primarily for the purpose of educating the reporters, the judge first gave a detailed explanation of the legal requirements for granting a renewed motion for judgment as a matter of law and for granting a new trial motion. He then granted the renewed motion for judgment as a matter of law and, alternatively, the motion for a new trial. Were the judge's actions proper? A) Both the statements to the jury and the communication with the reporters were proper. B) Neither the communication with the reporters nor the statements to the jury were proper. C) The statements to the jury were proper, but the communication with the reporters was not. D) The communication with the reporters was proper, but the statements to the jury were not.

D) The communication with the reporters was proper, but the statements to the jury were not.

A lawyer was a widely admired, highly compensated trial attorney in solo practice. He represented clients in all types of civil and criminal litigation, mostly in high-profile cases that drew a lot of media attention. The governor of the state where the lawyer practiced had been harshly criticized for appointing appellate judges who lacked significant experience as trial counsel. Hoping to silence his critics, the governor appointed the lawyer to serve out the remaining seven years of a recently deceased supreme court justice's 12-year term. After the seven years, the lawyer can run for election to a new 12-year term. Before taking the oath as judge, the lawyer sold his entire law practice—books, client files, office lease, furniture, and goodwill—to another attorney. The lawyer gave appropriate advance notice to the clients, and the purchasing attorney covenanted that he would not raise their legal fees. A few years later, one of the cases that the lawyer transferred to the purchasing attorney came before the state supreme court on appeal. Which of the following propositions is false? A) The lawyer's sale of his law practice was proper. B) The purchaser's covenant not to increase the fees paid by the lawyer's clients was proper. C) The lawyer must disqualify himself from the case involving his former client. D) The lawyer may participate in the decision of the case involving his former client, provided that all of the other supreme court justices give their informed consent.

D) The lawyer may participate in the decision of the case involving his former client, provided that all of the other supreme court justices give their informed consent.

An author wrote a best-selling novel based on the life and crimes of John Dillinger, the famous bank robber. The author sold the movie rights to a film producer, who promised to pay the author a lump-sum royalty of $5 million upon the release of the movie. After the producer hired an actor to play the lead role and made other expensive preparations for filming, the author repudiated the contract. The producer hired a lawyer to sue the author for a declaratory judgment that the contract was valid and enforceable. At the producer's request, the lawyer agreed to do the legal work on a contingent fee basis: If the producer wins, the lawyer will be paid 1.75% of the gross receipts from the movie, but if the producer loses, the lawyer will be paid nothing. The producer and the lawyer entered into a written fee agreement that contains all the details required by the rules of legal ethics. Which of the following statements is true? A) The lawyer is subject to discipline for entering into a publication rights contract with his client. B) The lawyer is subject to discipline for acquiring a personal interest in the subject of the litigation. C) The lawyer's fee agreement is proper, but only if the author gives informed consent. D) The lawyer's fee agreement is proper, even though it gives the lawyer a personal interest in the subject of the litigation.

D) The lawyer's fee agreement is proper, even though it gives the lawyer a personal interest in the subject of the litigation.

A prospective client comes to a law office seeking a lawyer to defend him in a civil action for aggravated assault and battery. A lawyer agrees to talk preliminarily with the client, just to obtain enough background information to decide whether she can defend him. The client explains that he has an alcohol problem; indeed, he gets roaring drunk about three nights a week. On the night in question, the client said that a loud-mouthed stranger in his neighborhood tavern made a derogatory comment about the client's favorite basketball team. The client responded by "tapping" the stranger over the head with a pool cue, not once but four times. At that point, the lawyer suddenly realizes that the client must be the rotten husband in the hotly disputed divorce and child custody case in which her law partner is representing the aggrieved wife. The lawyer stops the client and tells him that she cannot defend him in the assault and battery case because of her partner's work for the client's wife. Which of the following is true? A) The partner must withdraw from representing the wife because the lawyer has received confidential information from the client that would be harmful to the client if used in the divorce and child custody case. B) It would be proper for the partner to represent the wife and for the lawyer to represent the client in the assault and battery case because the two matters are not substantially related. C) The partner may continue representing the wife, but only if the wife gives informed consent, confirmed in writing. D) The partner may continue representing the wife if the lawyer is screened off from participation in the case and obtains no part of the fee in the case, and if the firm promptly sends the client written notice of the situation.

D) The partner may continue representing the wife if the lawyer is screened off from participation in the case and obtains no part of the fee in the case, and if the firm promptly sends the client written notice of the situation.

An elderly widower has one living child, a daughter. The widower's main asset is a 51% partnership interest in a wealthy real estate syndicate that owns and operates mobile home parks throughout the state. The daughter's husband is an attorney. One of the husband's regular clients asks the husband to represent him in negotiating the sale of 3,000 acres of roadside property to the real estate syndicate. The real estate syndicate is represented by its own lawyer in the matter. May the husband represent his regular client in a sale with the real estate syndicate? A) No, even if the client gives informed consent, confirmed in writing. B) No, because to do so would create an appearance of impropriety. C) Yes, because the husband has no significant personal interest in the real estate syndicate. D) Yes, but only if the client gives informed consent, confirmed in writing.

D) Yes, but only if the client gives informed consent, confirmed in writing.

Two years ago, an attorney represented his client when he sold his property. Unbeknownst to the attorney, the client made some fraudulent statements to the buyer about the value of some mineral deposits on the property. The buyer recently discovered the fraud and is now in the attorney's office threatening to immediately file a civil fraud suit against both the client and the attorney. The buyer accuses the attorney of engineering the fraud and helping his client carry it out. The only way that the attorney can convince the buyer that he had no part in the fraud is to tell the buyer a fact that the client disclosed to him in the deepest confidence when he was working on the property transaction. May the attorney disclose the fact without the consent of the client? A) No, if doing so will harm the client. B) No, because doing so would breach his duty of confidentiality to the client. C) Yes, but only after the buyer files the civil fraud suit against him. D) Yes, even if doing so will subject his client to civil or criminal liability.

D) Yes, even if doing so will subject his client to civil or criminal liability.

A solo practitioner is one of only three lawyers in a small town. The solo practitioner is presently defending a client in a criminal action for assault and battery. This morning one of the solo practitioner's regular clients, a gas and grocery store, asked the solo practitioner to sue the same client to recover a past due amount on a gasoline and grocery charge account. Would it be proper for the solo practitioner to represent the gas and grocery store in the charge account case? A) No, because it is presumed that a lawyer obtains confidential information in the course of representing a client. B) No, unless the other two lawyers in town are disqualified from representing the gas and grocery store. C) Yes, because there is no substantial relationship between the charge account case and the assault and battery case. D) Yes, if both the client and the gas and grocery store consent after full disclosure of the conflict, and such consent is confirmed in writing.

D) Yes, if both the client and the gas and grocery store consent after full disclosure of the conflict, and such consent is confirmed in writing.

A plaintiff, represented by his lawyer, brought suit in federal district court against a pest control company and nine chemical companies for physical and emotional injuries the plaintiff suffered after accidentally inhaling cockroach spray emanating from an apartment that had recently been fumigated by the pest control company. The lawyer's theory for suing the nine chemical companies was that the pest control company had probably purchased its cockroach spray from at least one of the nine chemical companies. A large law firm represented one of the nine chemical company defendants. By using depositions and document demands early in the discovery phase of the case, the law firm established that the chemical company it represented had never at any time sold any type of chemical to the pest control company. The law firm then moved for summary judgment as to its client. The plaintiff's lawyer offered no substantive response to that motion, but rather filed a countermotion to disqualify the law firm on the ground that the firm was biased against the plaintiff. The trial judge denied the motion to disqualify the law firm and granted the chemical company's summary judgment motion, whereupon the plaintiff's lawyer immediately moved for a rehearing, moved to stay the trial judge's two orders, and moved to disqualify the trial judge for bias and prejudice against the plaintiff and in favor of the defendant chemical company, the nature of the bias and prejudice being unspecified. The disposition of these motions consumed an entire year, due to the lawyer's obstreperousness and his repeated requests for postponements and extensions of time. Meanwhile, the law firm had to stay actively involved in the case to protect the chemical company's position. This year-long ordeal ended up costing the chemical company $14,500 in attorneys' fees and $6,750 in litigation costs. Is the plaintiff's lawyer subject to litigation sanction in the form of an order against the lawyer personally to pay the $14,500 in attorneys' fees and the $6,750 in litigation costs? A) No, because the lawyer was representing his client zealously within the bounds of the law as he was required to do by the rules of legal ethics. B) No, because litigation sanctions can be imposed only on parties to the litigation, not on their lawyers personally. C) Yes, even if the lawyer was acting in good faith, mistakenly but genuinely believing in the validity of the legal positions he took. D) Yes, provided that the chemical company can show that the lawyer either intentionally or recklessly took frivolous legal positions in order to harass the chemical company.

D) Yes, provided that the chemical company can show that the lawyer either intentionally or recklessly took frivolous legal positions in order to harass the chemical company.


Set pelajaran terkait

Section 15 unit 4 Maryland Property Disclosures

View Set

Erikson, Freud, Piaget, Vygotsky etc

View Set

Makanan dan Wadahnya -Yr6 Intermediate1-Term1 2018

View Set